You are on page 1of 100

MTH 103 LECTURE NOTES

CALCULUS I
DEPARTMENT OF MATHEMATICS
FACULTY OF NATURAL SCIENCES
UNIVERSITY OF JOS
NIGERIA
mathslist@unijos.edu.ng

July 22, 2013


Contents

1 Functions 4
1.1 Objectives: . . . . . . . . . . . . . . . . . . . . . . . . . . . . . . . . . . . . . . . . . . 4
1.2 Learning Outcomes . . . . . . . . . . . . . . . . . . . . . . . . . . . . . . . . . . . . . 4
1.3 Learning Activities . . . . . . . . . . . . . . . . . . . . . . . . . . . . . . . . . . . . . 5
1.4 Introduction . . . . . . . . . . . . . . . . . . . . . . . . . . . . . . . . . . . . . . . . . 5
1.5 Some Classes or types of Functions . . . . . . . . . . . . . . . . . . . . . . . . . . . . 7
1.6 Determination of Domain and Range of Functions . . . . . . . . . . . . . . . . . . . 11
1.7 Monotone (or Monotonic) Functions . . . . . . . . . . . . . . . . . . . . . . . . . . . 14
1.8 Zeros and Singularities of Functions. . . . . . . . . . . . . . . . . . . . . . . . . . . . 15
1.9 Injective, Surjective and Bijective Functions . . . . . . . . . . . . . . . . . . . . . . . 16
1.10 Composite Functions . . . . . . . . . . . . . . . . . . . . . . . . . . . . . . . . . . . . 19
1.11 Inverse Functions . . . . . . . . . . . . . . . . . . . . . . . . . . . . . . . . . . . . . . 20
1.12 Guidelines for finding the inverse of some functions . . . . . . . . . . . . . . . . . . 20

2 Limits and Continuity of Functions 22


2.1 Objectives . . . . . . . . . . . . . . . . . . . . . . . . . . . . . . . . . . . . . . . . . . 22
2.2 Learning outcomes . . . . . . . . . . . . . . . . . . . . . . . . . . . . . . . . . . . . . 22
2.3 Learning Activities . . . . . . . . . . . . . . . . . . . . . . . . . . . . . . . . . . . . . 22
2.4 Limit of a function . . . . . . . . . . . . . . . . . . . . . . . . . . . . . . . . . . . . . 23
2.5 Properties of Limits (Theorem) . . . . . . . . . . . . . . . . . . . . . . . . . . . . . . 27
2.6 Bounded Functions . . . . . . . . . . . . . . . . . . . . . . . . . . . . . . . . . . . . . 28
2.7 Limit of Indeterminate Forms . . . . . . . . . . . . . . . . . . . . . . . . . . . . . . . 29
2.8 Limits that arise Frequently . . . . . . . . . . . . . . . . . . . . . . . . . . . . . . . . 30

1
2.9 Continuity . . . . . . . . . . . . . . . . . . . . . . . . . . . . . . . . . . . . . . . . . . 30

3 The Derivative 34
3.1 Objectives . . . . . . . . . . . . . . . . . . . . . . . . . . . . . . . . . . . . . . . . . . 34
3.2 Learning Outcomes . . . . . . . . . . . . . . . . . . . . . . . . . . . . . . . . . . . . . 34
3.3 Learning Activities . . . . . . . . . . . . . . . . . . . . . . . . . . . . . . . . . . . . . 34
3.4 Introduction . . . . . . . . . . . . . . . . . . . . . . . . . . . . . . . . . . . . . . . . . 35
3.5 Differentiability of a function at a point . . . . . . . . . . . . . . . . . . . . . . . . . 36
3.6 Methods of Differentiation . . . . . . . . . . . . . . . . . . . . . . . . . . . . . . . . . 37
3.6.1 Differentiation of Sums ( Difference) and Powers . . . . . . . . . . . . . . . 37
3.6.2 The Product Rule . . . . . . . . . . . . . . . . . . . . . . . . . . . . . . . . . . 38
3.6.3 The Quotient Rule . . . . . . . . . . . . . . . . . . . . . . . . . . . . . . . . . 39
3.6.4 The Chain Rule . . . . . . . . . . . . . . . . . . . . . . . . . . . . . . . . . . . 40
3.6.5 Higher Derivatives and the Leibnitz Rule . . . . . . . . . . . . . . . . . . . . 41
3.6.6 Implicit Differentiation . . . . . . . . . . . . . . . . . . . . . . . . . . . . . . . 42
3.7 Derivatives of Logarithmic Functions . . . . . . . . . . . . . . . . . . . . . . . . . . 44
3.8 Differentiation of Exponential Functions . . . . . . . . . . . . . . . . . . . . . . . . . 47
3.9 Derivatives of Hyperbolic Functions . . . . . . . . . . . . . . . . . . . . . . . . . . . 49
3.10 Derivatives of Inverse Trigonometric Functions . . . . . . . . . . . . . . . . . . . . . 50
3.11 Derivatives of Inverse Hyperbolic Functions . . . . . . . . . . . . . . . . . . . . . . 53
3.12 Applications of Differentiation . . . . . . . . . . . . . . . . . . . . . . . . . . . . . . 56
3.12.1 Limit of Indeterminate Forms . . . . . . . . . . . . . . . . . . . . . . . . . . 56
3.12.2 Critical Point . . . . . . . . . . . . . . . . . . . . . . . . . . . . . . . . . . . . 58
3.12.3 First Derivative Test . . . . . . . . . . . . . . . . . . . . . . . . . . . . . . . . 58
3.12.4 Maximum and Minimum Problems . . . . . . . . . . . . . . . . . . . . . . . 60
3.12.5 Second Derivative Test . . . . . . . . . . . . . . . . . . . . . . . . . . . . . . . 62

4 Integration and the Indefinite Integral 67


4.1 Objectives . . . . . . . . . . . . . . . . . . . . . . . . . . . . . . . . . . . . . . . . . . 67
4.2 Learning Outcomes: . . . . . . . . . . . . . . . . . . . . . . . . . . . . . . . . . . . . 67
4.3 Learning Activities: . . . . . . . . . . . . . . . . . . . . . . . . . . . . . . . . . . . . . 67

2
4.4 Introduction . . . . . . . . . . . . . . . . . . . . . . . . . . . . . . . . . . . . . . . . . 68
4.4.1 The Indefinite Integral . . . . . . . . . . . . . . . . . . . . . . . . . . . . . . . 68
4.5 Methods of Integration . . . . . . . . . . . . . . . . . . . . . . . . . . . . . . . . . . . 70
4.5.1 Integration by substitution or change of variable . . . . . . . . . . . . . . . . 70
4.5.2 Integration by Parts . . . . . . . . . . . . . . . . . . . . . . . . . . . . . . . . 72
4.5.3 Integration by partial fractions . . . . . . . . . . . . . . . . . . . . . . . . . . 73
4.5.4 Integration by Reduction Formula . . . . . . . . . . . . . . . . . . . . . . . . 76

5 The Definite Integral and Its Applications 78


5.1 Objectives: . . . . . . . . . . . . . . . . . . . . . . . . . . . . . . . . . . . . . . . . . . 78
5.2 Learning Outcomes . . . . . . . . . . . . . . . . . . . . . . . . . . . . . . . . . . . . 78
5.3 Learning Activities . . . . . . . . . . . . . . . . . . . . . . . . . . . . . . . . . . . . . 78
5.4 Introduction: . . . . . . . . . . . . . . . . . . . . . . . . . . . . . . . . . . . . . . . . . 79
5.5 Properties of the definite Integral . . . . . . . . . . . . . . . . . . . . . . . . . . . . . 81
5.6 Area between two curves . . . . . . . . . . . . . . . . . . . . . . . . . . . . . . . . . 83
5.7 Arc Length . . . . . . . . . . . . . . . . . . . . . . . . . . . . . . . . . . . . . . . . . . 84
5.8 Volumes of Solids of Revolution . . . . . . . . . . . . . . . . . . . . . . . . . . . . . . 84
5.9 Mean or Average Value of a Function . . . . . . . . . . . . . . . . . . . . . . . . . . . 85
5.10 Moment of Inertia . . . . . . . . . . . . . . . . . . . . . . . . . . . . . . . . . . . . . . 86
5.10.1 Thin Rod . . . . . . . . . . . . . . . . . . . . . . . . . . . . . . . . . . . . . . 90
5.10.2 Circular Disc . . . . . . . . . . . . . . . . . . . . . . . . . . . . . . . . . . . . 90
5.10.3 Sphere . . . . . . . . . . . . . . . . . . . . . . . . . . . . . . . . . . . . . . . . 92
5.11 Differential Equations . . . . . . . . . . . . . . . . . . . . . . . . . . . . . . . . . . . 93
5.12 Application of Integration to Problems in Physical Sciences & Engineering. . . . . 95
5.12.1 Displacement/Distance and Velocity/Speed . . . . . . . . . . . . . . . . . . 95
5.12.2 Work . . . . . . . . . . . . . . . . . . . . . . . . . . . . . . . . . . . . . . . . . 96
5.13 Application of Integration to Laws of Growth and Decay . . . . . . . . . . . . . . . 97
5.14 Application of Integration to Problems in Economics . . . . . . . . . . . . . . . . . 98

3
Chapter 1

Functions

1.1 Objectives:

In this module:

1. We introduce with examples, the concept of a function and study some classes of functions.

2. We discuss methods of finding the domain, range, zeros and singularities of some real-
valued functions.

3. We look at injective (one to one) and surjective (onto) functions and end the module by
exploring the inverses of some functions and the composition of functions.

4. We stimulate students through classroom and other interactive activities to understand the
concept of a function and then discover the differences and similarities between the several
types of functions introduced.

1.2 Learning Outcomes

At the end of this module, students should be able to

1. Define a function; identify different types of functions and identify relations which are not
functions;

2. Identify different classes of functions, find the domain and range that makes a rule f (say),
a function;

3. Investigate the injectiveness, surjectiveness and the inverse of different functions;

4. Establish the composition of two or more functions.

4
1.3 Learning Activities

Students should:

1. Explore notes and exercises, individually and in groups;


2. Explore and use related materials on the Intranet, especially the e-granary and Massa-
chusetts Institute of Technology open course ware ( MITOCW );
3. Explore and use related materials on the Online Learning Initiatives Calculus course on
the Internet
(http://www.cmu.edu/OLI/courses/);
4. Solve relevant questions from past MTH103 Examinations.

1.4 Introduction

A function is the means by which the real world can be described in mathematical terms. For
instance, the area of a circle C depends or is a function of its radius r. A function f is an input-
output relation in which f ( x ), is the output with the variable x as the input. Thus a function f ,
with the input x, can be expressed as f ( x ), which is the value of the function at the point x. A
function can also be referred to as a mapping or a transformation.
We shall consider the definition and types of functions as well as examine some of their proper-
ties.

Definition 1.1 Function


Let X and Y be two nonempty sets. A function f :X ! Y is a rule which assigns or associates to each
element x of the set X, a unique (one and only one) element y = f ( x ) in Y.
Alternatively, a function can be defined as a relation R from a set X to another set Y such that (i) to every
x 2 X, 9 y 2 Y such that x is related to y written as xRy (ii) if xRy1 and xRy2 then y1 = y2 ( in other
words the relation is called a function from X to Y if to each x 2 X there corresponds exactly one y 2 Y).

Definition 1.2 Domain, Codomain and Range of a Function


The set X of all x-values allowed by the function f : X ! Y, is the domain of the function f . It is denoted
by Dom( f ) or D ( f ). x which may take any value in Dom( f ), is the independent variable. y is called the
dependent variable since its value depends on the value of the independent variable x. For a particular
value x = x1 the value y1 = f ( x1 ) is called the image of x1 under f while x1 is called the preimage of
y1 under f . The set Y in the definition of the function f : X ! Y is called the codomain of the function
f . The set of all admissible y-values as x goes through all values in its domain, is called the range of the
function. The range or image set of a function f will be denoted by

Ran( f ) = fy 2 Y : f ( x ) = y, x 2 D ( f )g

5
or simply R( f ).

p
Example 1.1 : X = X1 = f1, 2, 3g; Y = Y1 = f 73 , 1, 12 , 2, 2, 4, 5, 6g and f ( x ) = 2x. These two
sets and the given rule define a function because each element in the domain X1 has a unique image in the
codomain Y1 .The range of the function is R( f ) = f2, 4, 6g.

p p
Example 1.2 : X = X2 = f0, 4, g; Y = Y2 = fp 37 , 2, 1, 0, 12 , 2, 2, 4, 5, 6g and g( x ) = x.
These two sets X2 , Y2 and the given rule g( x ) = x do not define a function because the element 4 in
the domain X2 has two different images 2 and 2 in the codomain Y2 .This is despite the fact that 0 in the
domain X2 has a unique image 0 in the codomain Y2 .

Example 1.3 : Let h : X3 ! E with X3 =f1, 2, 3, 4, 5g and E = f2, 4, 6, ...g and h is defined by
h( x ) = 2x. 0 1
E
0 1 B 2 C
X3 B C
B 4 C
B 1 C B C
B C B 6 C
B 2 C B C
B C ! B 8 C.
B 3 C B C
B C B 10 C
@ 4 A B C
B . C
5 B C
@ . A
.
Here, the rule is multiplication of the elements of X3 by 2 giving in each case, a unique even number. In
this example the domain of h is the set X3 = f1, 2, 3, 4, 5g ,the codomain is the set E = f2, 4, 6, ...g, while
the range is the set f2, 4, 6, 8, 10g = R(h).

Remark 1.1 : Generally the range is a subset of the codomain as the examples above illustrate.

We shall consider functions f : R ! R. A function f is represented algebraically if, for each x


in the domain, f ( x ) is given by some algebraic expression involving x. Since many algebraic
operations give unique results, there is usually no problem in determining whether or not, an
algebraic expression involving x defines a function. For instance, since the square of a number
is a unique number, the expression f ( x ) = x2 is a rule that defines a function from R to R.
Also, since the sum of any two numbers is a unique number, the expression g( x ) = x + 5 is
p defines a function from R to R. Can you determine whether or not, the expression
a rule that
s( x ) = x is a rule that defines a function from R to R?

Remark 1.2 : When a function f is defined by a formula y = f ( x ) and the domain of the function is
not explicitly given, then we consider its domain to consist of all values of x for which the formula can be
evaluated as a real number. In particular, division by zero is not allowed, and square roots (or even roots)
of negative numbers are not allowed.

6
Remark 1.3 : A function can be expressed in terms of ordered pairs which in turn can be represented as
co-ordinates of points in a plane. Given S = f a, b, cg and T = f1, 2g,then f = f( a, 1), (b, 1), (c, 2)g is
a rule and defines a function that assigns elements of T to elements of S as shown. In the third example
given above, h can be written as

h = f(1, 2), (2, 4), (3, 6), (4, 8), (5, 10)g.

Example 1.4 : (Relations and Rules that are not functions)

p
a Consider f : R+ ! R defined by f ( x ) = x or f = f( x, y) : y2 = x g. This is not a
function because
p an element of the domain can be mapped to two elements of the codomain, e.g.
f (9) = 9 = 3. Thus 9 is assigned to 3 and +3 at the same time, as such, f cannot be a
function. The rule will be a function if we are allowed to take only the values in ( ∞, 0) or (0, ∞).
i.e from R or R+ at a time.

b Consider the function f : R ! [0, 2π ] defined by f ( x ) = sin 1 x. Observe that 21 is mapped to


π 5π
6 rad and 6 as such, f is not a function. The rule will be a function if we are also allowed to take
only the values in [0, π2 ].

c Let S = f a, b, cg and T = f1, 2g then f = f( a, 1), ( a, 2), (b, 1), (c, 2)g is not a function because
a is mapped to both 1 and 2 at the same time, i.e to more than one element.

1.5 Some Classes or types of Functions


1. Linear Functions:
A linear function is of the form y = f ( x ) = mx + c where m and c are constants. The
graph of such a function is a straight line with gradient or slope m and y intercept as
c. Any linear equation in x and y can be considered to be of the form y = mx + c .Thus,
lx + my + n = 0̄ can be rearranged to be y = lx m
n
m with m =
l
m, c =
n
m .For example
6x + 3y = 5 reduces to y = 2x + 35 . A double intercept equation of a straight line can be
y
written as xa + b = 1, a 6= 0, b 6= 0.

2. Constant Functions:
A constant function is of the form y = f ( x ) = c, where c is the constant taken by the
function y for every value of x in the domain of f . A constant function is a special case of a
linear function with m = 0.For example, y = 2. is a constant function.

3. Quadratic Functions:
A quadratic function is of the form y = f ( x ) = ax2 + bx + c where a, b, c are constants and
a 6= 0. Example is the function f ( x ) = y = x2 5x + 6.

7
4. Polynomial Functions:
A polynomial function of degree n in x is a function of the form,
n 1
y = Pn ( x ) = a0 + a1 x + a2 x2 + + an 1x + an x n with an 6= 0
where a0 , a1, , an are constants called the coefficients of Pn and n is a non-negative in-
teger. For example,y = P4 ( x ) = x4 + 3x3 + 2x + 3 is a polynomial of degree 4. Note that
constant, linear and quadratic functions are polynomials functions of degrees 0, 1 and 2
respectively.
5. Power functions:
A power or geometric function is of the form f ( x ) = xr where r is any real number. f ( x ) =
3
x3 ; g( x ) = x 4 and h( x ) = x 2 are examples of power or geometric functions.
6. Rational Functions:
P( x )
A rational function is of the form y = f ( x ) = Q( x) , where P( x ) and Q( x ) are polynomials
with Q( x ) 6= 0 . In other words, rational functions are ratios of polynomials. For example
3
the function f given by f ( x ) = xx4++2x +1
x +1
is a rational function. In a particular case when
the denominator is a constant the rational function reduces to a polynomial; the class of all
polynomials is a sub-class of all rational functions.
7. Algebraic Functions:
An algebraic function of x, of degree n, is the root of an
pequation of degree n in y whose
p
coefficients are rational functions of x.For example y = x + x becomes y4 2xy + x2
x = 0 when the roots are removed by squaring. Sometimes it is not possible to solve for
y explicitly, for example y5 + xy x3 = 0.In this case the equation is said to define y as
an implicit function of x. All functions which are not algebraic are called transcendental.
This class includes the trigonometric functions and their inverses, the logarithmic and ex-
ponential functions and the hyperbolic functions.
8. Trigonometric Functions:
Functions that involve one or more of the trigonometric ratios (sine, cosine, tangent, cose-
cant, secant, or cotangent) are called trigonometric functions. For example, y = f ( x ) =
sin x and g( x ) = cos2 x + cot x are trigonometric functions.
9. Inverse Trigonometric Functions:
Functions that involve one or more of the inverses of the trigonometric ratios (arcsine, ar-
ccosine, arctangent, arccosecant, arcsecant, or arccotangent) are called inverse trigonomet-
ric functions. For example, y = f ( x ) = sin 1 x and g( x ) = cos 1 x + tan 1 x are inverse
trigonometric functions.
10. Exponential Functions:
The exponential functions are functions of the form f ( x ) = e x or f ( x ) = a x ,where a is any
fixed positive number and x is any real number. The former type can be expressed as a
series as,

8

x2 x3 xn n
f (x) = ex = 1 + x + 2! + 3! + + n! + = ∑ ( xn! ).
n =0
Here n! = n(n 1)(n 2) (2)(1)and f ( x ) = e x is
always real and positive.
Note that the trigonometric functions can be defined in terms of the exponential function
as
eix e ix eix + e ix
sin x = , cos x =
2i 2
and
i eix e ix
tan x =
eix + e ix
p
, where i = 1 defines the imaginary unit.

11. Hyperbolic Functions:


These functions are defined in terms of the exponential function as

ex e x ex + e x ex e x
sinh x = , cosh x = , tanh x = ,
2 2 ex + e x

2 2
cschx = , sechx =
ex e x ex +e x

and
ex + e x
cothx = .
ex e x

Note that the trigonometric and hyperbolic functions are related by

sinh x = i sin(ix ),

cosh x = cos(ix ),
tanh x = tan(ix )
p
, where again i = 1 defines the imaginary unit.

12. Logarithmic Functions:


These are functions that involve logarithm, for example, y = f ( x ) = log10 x or h( x ) =
loge x = ln j x j

13. Modulus or Absolute Value Function:


This is a function of the form;

x, x 0
f (x) = jxj =
x, x <0;
or p
jxj = x2 .
Note for instance that, j 3j = 3, j0j = 0, and j2j = 2.

9
14. The Characteristic Function:
This function is defined by
1, if x 2S
χS ( x ) = .
0, if x 2
/S

15. The Floor Function b x c :


This function is defined by f ( x ) = b x c , for every real number x, and its value is the greatest
integer which is less than or equal to x. For example b2.34c = 2, and b 2.7c = 3 .

16. The Ceiling Function d x e :


This function is defined by f ( x ) = d x e , for every real number x, and its value is the small-
est integer that is greater than or equal to x. For example d2.34e = 3 and d 4.781e = 4.

17. The Square Root Function:


p p
This function is defined by f ( x ) = x, where the symbol denotes the positive square
root of x.

18. Euler’s Form of Trigonometric and Hyperbolic Functions:


The exponential function eix can be expressed as eix = cos x + i sin x. This is known as
Euler’s form (pronounced as Oiler). Also, e ix = cos( x ) + i sin( x ) = cos x i sin x.
From these two expressions, we can write the trigonometric functions in terms of the ex-
ponential function as

eix e ix eix + e ix eix e ix


sin x = , cos x = , tan x = ,
2i 2 ieix + ie ix

etc.

19. Periodic Function:


A function f is said to be periodic if there exists a real number T > 0 such that f ( x + T ) =
f ( x ) for every x in the domain of f , T is called the period of f . For example, f : R ! R
defined by f ( x ) = sin x is periodic with period T = 2π because sin( x + 2π ) = sin x, for
every x in the domain of f . The smallest T > 0 is called the primitive period of f .

20. Odd Function: A function f is said to be odd if f ( x ) = f ( x ) 8 x 2 D ( f ). Equivalently,


f is said to be odd if f ( x ) + f ( x ) = 0 For every x 2 D ( f ). For example, f ( x ) = x3 is odd
because f ( x ) = ( x )3 = x3 = f ( x ), 8 x 2 D ( f ). Similarly, sin x is odd because
sin( x ) = sin x,for all real x.

21. Even Function:


A function f is said to be even if f ( x ) = f ( x ) 8 x 2 D ( f ). .Equivalently, f is said
to be odd if f ( x ) + f ( x ) = 0. For every x 2 D ( f ).Equivalently, f is said to be even if
f ( x ) f ( x ) = 0 8 x 2 D ( f ).For example, f ( x ) = x2 is even because f ( x ) = ( x )2 =
x2 = f ( x ) 8 x 2 D ( f ).Also, cos x is even since cos( x ) = cos x 8 x 2 R.

10
Remark 1.4 : Some functions are neither odd nor even, for example,(i) f ( x ) = x2 + x + 1 (ii)
g( x ) = x 1 1 (iii) h( x ) = sin x + cos x.
The evenness or oddness of a function f is known as its parity.

Exercise 1.5
Determine which of the following functions is periodic, even, odd or neither even nor odd?
1 2 2 p
tan x, sec x, csc x, cot x, sinh x, cosh x, tanh x, eix , j x j , 2x , ln x, log10 x2 , e x , xe x , x.

1.6 Determination of Domain and Range of Functions

Example 1.5 Given the rules below with R as codomain, determine in each case, the domain so that a
function is well defined.

1. f ( x ) = x2 .
2. f ( x ) = 2x2 x + 1.
3. f ( x ) = 1x .
p
4. f ( x ) = 4x2 1.
5. f ( x ) = p1 .
+ 4 x2

6. f ( x ) = e2x+4 .
7. f ( x ) = log( x2 4).

1. For any real number x, x2 is real. Therefore, f maps every real number into a real number
and so the domain of f is the set of all real numbers. i.e. Dom( f ) = R, where R = ( ∞, ∞).
2. It is not difficult to see that 2x2 x + 1 is always real 8 x 2 R, since it is a polynomial.
D ( f ) = ( ∞, ∞) = R. In fact, all polynomial functions are real for real values of x. i.e.
D ( Pn ( x )) = R, for all polynomials Pn ( x ).
1
3. Observe that the function x is real iff the denominator is real and not zero. i.e.

D ( f ) = f x 2 R : x 6 = 0g = R f0g = ( ∞, 0) [ (0, ∞)
p
4. 4x2 1is real iff 4x2 1 0. i.e..(2x 1)(2x + 1) 0 =) either 2x 1 0 and 2x + 1
1 1
0; or 2x 1 0 and 2x + 1 0.The first pair of inequalities gives x 2 and x 2;
1 1 1
both combine to give x 2 . The second pair of inequalities gives x 2 and x 2 ; both
1
combine to give x 2.
Thus, Dom( f ) = f x 2 R : ∞<x 1
2 g [ fx 2R: 1
2 x < ∞g = R ( 1 1
2 , 2 ).

11
p
5. Observe that p 1 2 is real iff the denominator 4 x2 is real and nonzero, i.e..(2 x )(2 +
4 x
x ) > 0 =) either 2 x > 0 and 2 + x > 0; or 2 x < 0 and 2 + x < 0.The first pair of
inequalities gives x < 2 and x > 2; both combine to give 2 < x < 2. The second pair
of inequalities gives x > 2 and x < 2; there are no values of x satisfying these last two
inequalities at the same time.
Hence, Dom( f ) = f x 2 R : 2 < x < 2, g = ( 2, 2).

6. Observe that 8 x 2 R, 2x + 4 is real and the exponential e2x+4 is also real. ) Dom( f ) = R.

7. Note that loge ( x2 4) is real iff x2 4 > 0 =) ( x 2)( x + 2) > 0


=) x 2 > 0 and x + 2 > 0 or x 2 < 0 and x + 2 < 0
) x > 2 and x > 2 or x < 2 and x < 2.
This implies that x2 4 > 0 when either x < 2 or x > 2. ) Dom( f ) = f x 2 R : x < 2
or x > 2g = ( ∞, 2) [ (2, ∞)
=R [ 2, 2].

Example 1.6 : Determine the range of each of the following functions.

1. f : R ! R defined by f ( x ) = x2 .

2. f : R ! R such that f ( x ) = 2x2 x + 1.

3. f : R ! R defined by f ( x ) = 1x .
p
4. f : R ! R defined by f ( x ) = 4x2 1

5. f : R ! R defined by f ( x ) = p1 .
+ 4 x2

6. f : R ! R such that f ( x ) = e2x+4 .

7. f : R ! R such that f ( x ) = log( x2 4).

Solution 1.1 : [1]

1 Observe that f is mapped to non-negative real numbers since the square of any real number
(whether negative, zero or positive) is non-negative. Therefore, the range is the set of non-negative
real numbers i.e.. Ran( f ) = fy 2 R : 0 y < ∞g = [0, ∞).
Alternatively, to find the range of f we can let f ( x ) to be y and obtain the possible values of y for
which x is real, since f is a mapping of real numbers.
p
Let f ( x ) = y ) x2 = y ) x = y. Observe that for x to be real (as required) y has to be
non-negative real numbers. i.e. y 0 ) R( f ) = fy 2 R : 0 y < ∞g = [0, ∞).

12
2 Let f ( x ) = y ) 2x2 x + 1 = y ) 2x2 x + 1 y = 0.Using the quadratic formula for the
roots p
1 1 8(1 y )
)x=
4
Observe that x is real iff 1 8(1 y) 0 )y 7
8 ) Ran( f ) = fy 2 R : 78 y < ∞g =
[ 8 , ∞). In determining the range, it is sometimes possible to sketch the curve of the function and
7

take all the possible values of y from the graph as the range of the function.

3 Let f ( x ) = y
1
) =y
x
1
)x=
y
Again x is real iff y is real and nonzero. Therefore,

Ran( f ) = fy 2 R : y 6= 0g = R f0g = ( ∞, 0) [ (0, ∞).

Here the domain of f is the same as its range.


p
4 Let y = f ( x ), then y = 4x2 1.It should be observed that y is non-negative iff 4x2 1 0.
=) for all values of x in the Dom( f ), y takes non-negative values. Thus,

Ran( f ) = fy 2 R : y 0 g = R+ [ f 0 g

5 Let f ( x ) = y
1
)p =y
4 x2
p
) y 4 x2 = 1
) y2 (4 x2 ) = 1 ) 4y2 y2 x 2 = 1
) y2 x 2 = 1 4y2 ) y2 x2 = 4y2 1
p
4y2 1 4y2 1
) x2 = )x= , y 6= 0
y2 y
Observe that x is real if 4y2 1 0 =) (2y + 1)(2y 1) 0.
Therefore, either 2y + 1 0 and 2y 1 0 or 2y + 1 0 and 2y 1 0.This implies that
1 1 1 1 1 1
either y 2 and y 2 or y 2 and y 2 .Eithery 2 or y 2 satisfy the inequality
4y2 1 0.Thus,
1 1
y 2 ( ∞, ] [ [ , ∞)
2 2
Observe that f cannot be negative,
) Ran( f ) = fy 2 R : 1
2 y < ∞, g = [ 12 , ∞).

13
6 Let f ( x ) = y i.e. e2x+4 = y and taking the natural logarithm of both sides,
) 2x + 4 = ln y i.e. x = 21 (ln y 4) = 21 ln y 2.
Observe that x is real if y > 0 ) Ran( f ) = fy 2 R : y > 0g = (0, ∞).
7 Let f ( x ) = y ) ln( x2 4) = y and taking exponential of both sides.
p y
) x2 4= ey ) x2 = ey + 4 i.e. x = e + 4,
x is real for all ey + 4 > 0. Observe that this is true 8y 2 R,
since ey > 0 > 4 ) Ran( f ) = R = ( ∞, ∞).

Exercise 1.6
Find the domain and range of each of the following functions.
(i) g : R ! R s.t. g( x ) = 1
1 .
(36 x2 ) 2

(ii) g : R ! R s.t. g( x ) = p 1 .
x2 36

(iii) f : R ! R s.t. f ( x ) = 7x 2
2x +3 .

(iv) p : R ! R s.t. p( x ) = tan( x2 1).


(v) f : R ! R s.t. f ( x ) = tan 1 2x.

(vi) f : R ! R s.t. f ( x ) = sin 1


x.
(vii) f : R ! R s.t. f ( x ) = cos x.
(viii) f : N ! R s.t f ( x ) = 2x. Here N denotes the set of natural numbers.
(ix) f : Z ! R s.t. f ( x ) = 1
3x . Here Z denotes the set of integers.
(x) f : [ 1, 1] ! R s.t. f ( x ) = x2 .
(xi) f : R ! R s.t. f ( x ) = tanh x.
(xii) h : R ! R s.t. h( x ) = cot(3x 2)

1.7 Monotone (or Monotonic) Functions

Definition 1.3 : A function f is said to be increasing on an interval I = [ a, b] if for x1 , x2 in I,


f ( x1 ) f ( x2 ) whenever a x1 < x2 b.

Definition 1.4 : A function f is said to be strictly increasing on an interval I = [ a, b] if for x1 , x2 in I,


f ( x1 ) < f ( x2 ) whenever a x1 < x2 b.

Definition 1.5 : A function f is said to be decreasing on an interval I = [ a, b] if for x1 , x2 in I,


f ( x1 ) f ( x2 ) whenever a x1 < x2 b.

14
Definition 1.6 : A function f is said to be strictly decreasing on an interval I = [ a, b] if for x1 , x2 in I,
f ( x1 ) > f ( x2 ) whenever a x1 < x2 b.

Definition 1.7 : A function f which is either increasing ( or strictly increasing) or decreasing ( strictly
decreasing) is called a (monotone) or monotonic function.

Example 1.7 : The function f : [0, 2] ! R defined by f ( x ) = 2x is monotonically increasing on [0, 2],
since f ( x + h) = 2( x + h) = 2x + 2h > 2x = f ( x ) 8 x 2 [0, 2], h > 0.
For instance, f ( 21 ) = 2( 12 ) = 1 < 2 = 2(1) = f (1), as 1
2 < 1 where 12 , 1 2 [0, 2]

Example 1.8 : The function g : [1, 3] ! R defined by g( x ) = 1x is monotonically decreasing on [1, 3],
1 1 1 1
since g( x + k ) = x+ k < x = g ( x ) 8 x 2 [1, 3], k > 0.For example g (2) = 2 < 1 = 1 = g (1) as
1 < 2, 1, 2 2 [1, 3].

Remark 1.5 : In order to determine the monotonicity of a function f , one can either compute the
f ( x +h)
difference f ( x + h) f ( x ), h > 0 or the quotient f ( x) , h > 0. The function f is increasing if
f ( x +h) f ( x +h)
f ( x + h) f ( x ) 0 or f (x)
> 0 and decreasing if f ( x + h) f (x) 0, h > 0 or f (x)
< 0, h > 0
and 8 x 2 Dom( f ).

Exercise 1.7
Investigate the monotonicity or otherwise of the following functions:
1
(1) f ( x ) = 3x2 6x 1;(2) g( x ) = x 2 +1
; (3) h( x ) = 2 sin πx; (4) k ( x ) = sec x;(5) l ( x ) = tan πx;(6)
m( x ) = cosh x,
(7) n( x ) = e x ;(8) x 2
p p( x ) = xe ;(9) q( x ) = j x j ;(10) r ( x ) = ln( x + 1); (11) s( x ) = log10 ( x + 3x +
2);(12) t( x ) = x, on the interval [0,1].

1.8 Zeros and Singularities of Functions.

Definition 1.8 : The value of x for which a function f takes the value zero is called the zero of the
function f . For example, the zeros of f ( x ) = x2 + 3x + 2 are 2 and 1 since f ( 1) = 0 and
f ( 2) = 0.

P( x )
Definition 1.9 : Given a rational function, f ( x ) = Q( x) ,the value of x for which the denominator Q( x )
is zero is called the singularity of f . For example, the singularities of the function,

x2 25
f (x) = x2 5x +6
are 2 and 3.

15
P( x )
Remark 1.6 : For a rational function f ( x ) = Q( x) , the zeros of f are the values of x for which the
numerator P( x ) = 0 while its singularities are the values of x for which the denominator Q( x ) = 0.
2
Thus, for the rational function f ( x ) = xx2+7x +12
5x +6
, the zeros are 3 and 4 while the singularities are 2
and 3.

Exercise 1.8
1. Calculate the zeros of the following functions: (a) f ( x ) = cos(2x ). (b) g( x ) = tanx. (c) h( x ) =
sinh(2x ).
5x 3
2. Compute the zeros and singularities of the followings: (a) F ( x ) = cotx. (b) G ( x ) = x2 2x 3
.
x2 +4x x +4
(c) H ( x ) = x3 +3x2 10x
. (d) K ( x ) = x3 +3x2 10x
.

1.9 Injective, Surjective and Bijective Functions

Definition 1.10 : A function f : Dom( f ) ! R is injective (one to one) if f ( x1 ) = f ( x2 ) implies


x1 = x2 for x1 ,x2 2 Dom( f ). i.e. for each element x 2 Dom( f ),there is one and only one element y in
the codomain such that y = f ( x ). That is, two preimages in the domain of f do not have one image in
the codomain of f .

Definition 1.11 : A function f : Dom( f ) ! R is surjective (onto) if every element in the codomain of
f has a preimage in the domain of f .
That is the codomain is exhausted or the range and codomain of f are the same or equal.

Definition 1.12 : A function f : Dom( f ) ! R is bijective if f is both injective and surjective.

Example 1.9 : Which of the following functions is injective, surjective or bijective?

(a) f : R ! R; f ( x ) = 2x + 1.
(b) f : R ! R; f ( x ) = e x .
(c) f : R ! R+ ; f ( x ) = e x .
x 2 +2
(d) f : R ! R+ ; f ( x ) = x 2 +1
.

(e) f : R ! R; f ( x ) = p 2x2 .
( x +1

Solution 1.2 : (a) Let a, b 2 R such that f ( a) = f (b).Since f ( x ) = 2x + 1,then f ( a) = f (b) implies
that 2a + 1 = 2b + 1 and a = b, ) a = b whenever f ( a) = f (b). Thus f is injective.
y 1
To show that f is surjective, let 2x + 1 = y 2 R. Then x = 2 which is in R )every y in R has a
preimage x in R.

16
Thus f is surjective. Since f is both injective and surjective, it is bijective.
(b) f : R ! R; f ( x ) = e x . Let a, b 2 R such that f ( a) = f (b).
Then e a = eb .Taking the loge of both sides gives loge e a = loge eb =) aloge e = b loge e and a = b.Thus
a = b whenever f ( a) = f (b) and so f is injective.
Let e x = y 2 R. Then x loge e = loge y (since loge e = 1) or x = loge y.
But y is any real number and loge y is not defined if y is zero or negative. )For y zero or negative (i.e.
y 0) there is no real x such that e x = y.
Thus, f is not surjective because its range is R+ although its codomain is R;the range is not equal to the
codomain. The conclusion is that f is injective but not bijective.
(c) f : R ! R+ , f ( x ) = e x
Let a, b 2 R such that f ( a) = f (b) then e a = eb .Taking loge of both sides=) loge e a = loge eb =)

aloge e = b loge e =) a = b. ) a = b whenever f ( a) = f (b).


Thus, f is one to one. To show that f is a surjection, let y 2 R+ such that e x = y.Then y is a positive real
number and x loge e = loge y =) x = loge y.
Since y is a positive real number, then loge y 2 R )there exist a preimage x namely loge y for which
y = ex .
Thus, f is (onto) surjective. Since f is injective and surjective, it is therefore bijective.
x 2 +2 a2 +2 b2 +2
(d) f : R ! R, f ( x ) = x 2 +1
.Let a, b, 2 R be such that f ( a) = f (b). Then a2 +1
= b2 +1

=) ( a2 + 1)(b2 + 2) = ( a2 + 2)(b2 + 1) or a2 b2 + 2a2 + b2 + 2 = a2 b2 + 2b2 + a2 + 2 or


p
2a2 + b2 = 2b2 + a2 or a2 = b2 or a = b2 = b.Thus,a = b, where f ( a) = f (b).
It is therefore possible to have f ( a) = f (b) when a 6= b. Take for instance, 2 and 2.
+22 ( 2)2 +2
Although f (2) = 222 + 1
= f ( 2 ) = ( 2)2 +1 = 6
5 but 2 6= 2.Thus, one image, 6
5 had two preimages
namely 2 and 2.Therefore, f is not injective .
x 2 +2
Let y 2 R such that = y =)
x 2 +1
x2 + 2 = y( x2 + 1) = yx2 + 2
q
y 2
x 2 (1 y) = y 2, x = ( 1 y ).
q
y 2
Observe that ( 1 y )is not defined for all values of y. For instance if y = 0, there is no real x such that
q
y 2
x = (1 y) .
q
y 2
In fact for all y 2,there is no value of x for which x = ( 1 y ).Thus f is not a surjection since not
every y in R has no preimage in R. Hence f is not bijective.
(e) f : R ! R, f ( x ) = p 2x
, Suppose f ( a) = f (b), then
(2x +1)

17
2a 2b
p p
p = p or 2a (b2 + 1) = 2b ( a2 + 1) and on squaring both sides, we get 4a2 (b2 +
( a2 +1) ( b2 +1)
1) = 4b2 ( a2 + 1) which implies that
a2 b2 + a2 = a2 b2 + b2 or a2 = b2 or a = b.
By proceeding as in example (d), students can easily verify that f is neither one to one nor surjective.
Therefore f is not bijective.

Example 1.10 : (a) Let the absolute value relation be as defined below. Find the domain and range for this
p x, if x 0
relation to define a function. Is the absolute value function bijective? f ( x ) = j x j = + x2 = f x,if x<0 .

(b) What do you understand by j x j a, where a is a non negative real number?


(c) If A = f a, b, cg, find (i) χ A ( a), (ii) χ A (b), (iii)χ A (k ).

(d) Write the Characteristic function of the interval [ 1, 1).


ex e x x3 x5 x7
(e) Show that 2 = x+ 3! + 5! + 7! + ...

Solution 1.3 : (a) The absolute value is defined for all real
p values of x and as such the domain of the
absolute value is R. Thus, for any real number x, j x j = + x2 0.
Hence, the range is the set of all non negative real numbers, i.e. Ran( f ) = [0, ∞). Therefore, the relation
defines a function with R as its domain and [0, ∞) as its range.
Let j aj = jbj .This does not imply that a = b. For instance j 2j = j2j but 2 6= 2. The modulus function
is therefore not an injective function.
But for any non negative real number, y there is always a real number x whose modulus j x j is y.
Thus, the modulus function is surjective, if its codomain is the set of positive numbers R+ . Therefore, the
absolute value function j x j is not bijective.
(b) If x 0 then x a and if x 0 then x a. ) j x j a) a x a.
(c) (i) χ A ( a) = 1 since a 2 A. (ii) χ A (b) = 1 since b 2 A (iii) But χ A (k) = 0 since k 2
/ A.

1, if 1 x <1
(d) χ[ 1,1) ( x ) = f0, if x< 1 and x 1

x2 x3 xn
(e) Since e x = 1 + x + 2! + 3! + .... = ∑ n! ,
n =0
x )n x n ( x )n
) ex e x = ∑∞
n =0
xn
n! ∑∞
n =0
(
n! = ∑∞
n =0 n!
If n is even, then xn ( x )n = x n x n = 0. But if n is odd, then x n ( x )n = 2x n .
Considering odd values of n, we

∑∞ ∞
ex e x 2n+1 x2n+1 x3 x5 x7
) 2 = 1
2
x
n=0 2 (2n+1)! = ∑n=0 (2n+1)!
= x+ 3! + 5! + 7! + ...

18
Exercise 1.9
Verify whether or not the following functions are injective, surjective or bijective.
(i) f :R+ ! R, f ( x ) = loge x,(Ans:injective,surjective,bijective)
(ii) f : R+ ! R, f ( x ) = loge x + 2,(Ans:injective,surjective,bijective)
2x2 1
(iii) f : R ! R, f ( x ) = x 2 +1
,(Ans:not injective,not surjective,not bijective)
(iv) f : R ! R, f ( x ) = sin x,(Ans:not injective,not surjective, not bijective)
(v) f : R ! [ 1, 1], f ( x ) = sin x,(Ans:not injective,surjective,not bijective)
(vi) Sketch the graph of the modulus function between x = 3 and x = 3.What are the images
of 2 and 2? ( Ans.2)
(vii) If Peter is a Maths student but Jane is not, what is
χ(math students) (Peter)+χ(non math students) (Jane)? ( Ans.2)
e x +e x 2 4 6
(viii) Show that 2 = 1 + x2! + x4! + x6! + ...
e x +e x ex e x
(ix) If cosh x = 2 and sinh x = 2 , find cosh 0 + sinh 0.( Ans.1)
(x) What is cosh x + sinh x? x
( Ans.e ).

1.10 Composite Functions

Definition 1.13 : Let f : X ! Y and g : Y ! Z.The function h : X ! Z defined by h( x ) =


( go f )( x ) = g( f ( x )) is called the composition of g with f .

Example 1.11 : Let f : R ! R and g : R ! R such that g( x ) = 2x + 3 and f ( x ) = x2 + x. Find (i)


g( f (5)) (ii) g( f ( x )) (iii) Verify that f ( g( x ) 6= g( f ( x )) in general by evaluating the two functions at a
point x = a of your choice.

g( f (5)) = g(52 + 5) = g(30) = 2(30) + 3 = 63. g( f ( x )) = g( x2 + x ) = 2( x2 + x ) + 3 =


2x2 + 2x + 3. But f ( g( x )) = f (2x + 3) = (2x + 3)2 + 2x + 3 = 4x2 + 14x + 12.

Example 1.12 : If k ( x ) = 2x + 3 and t( x ) = x + 1, prove that t(k ( x )) = 2x + 4 and k (t( x )) =


2x + 5.For what values of x do we have k(t( x )) = t(k ( x ))?

t(k ( x )) = t(2x + 3) = 2x + 3 + 1 = 2x + 4;and k (t( x )) = k( x + 1) = 2( x + 1) + 3 = 2x +


5.
k (t( x )) = t(k ( x )), implies that 2x + 5 = 2x + 4; But there is no value of x such that 2x + 5 =
2x + 4.Therefore k (t( x )) 6= t(k ( x )) for any value of x.

19
Example 1.13 : Let g : R ! R such that g( x ) = 2x + 3 Find g( g( x )) and g( x2 ).

Now, g( g( x )) = g2 ( x ) = g(2x + 3) = 2(2x + 3) + 3 = 4x + 9 and g( x2 ) = 2x2 + 3.


Exercise 1.10
(a) Evaluate the following: (i) sin(cos π2 ) (ii)cos 1 (sin π ) : ( Ans :(i) 0;(ii) π/2)
p p
(b) If f ( x ) = x2 and g( x ) = x2 + 1, Find (i) f ( g( x ));(ii)g( f ( x )).( Ans. (i) x2 + 1; (ii) x4 + 1)
(c) Evaluate the following: (i) loge (eπ ) (ii) log(sin π2 ) (iii)ecos 2π .( Ans.(i) π, (ii) 0; (iii) e1 = e.)

1.11 Inverse Functions

Let f be a bijective function with domain X and range Y,then a function g with domain Y and
range Xis called the inverse function of f if g( f ( x )) = x for every x in X and f ( g(y)) = y for
every y in Y. The inverse function g of f if it exists, is usually denoted by f 1 .

Remark 1.7 : The symbol 1 as used in the notation f 1 of inverse of f should not be mistaken for an
exponent; that is, f 1 ( x ) does not mean [ f (1x)] . The reciprocal [ f (1x)] is denoted by [ f ( x )] 1 .

Remark 1.8 : It is absolutely essential that a function f be bijective for its inverse to be defined. This is
one reason why we studied bijective functions before inverses of functions.

1.12 Guidelines for finding the inverse of some functions

(a) Verify that f is bijective (i.e. one to one and onto ). If the function is not bijective one can only
find an inverse of a restriction of the function that is bijective.
(b) When the function is bijective, solve the equation y = f ( x ) for x in terms of y, obtaining an
equation of the form x = f 1 (y).
(c) Verify the two conditions f 1 ( f ( x )) = x and f ( f 1 ( x )) = x for every x in the domain of f
and f 1 respectively.

Example 1.14 : Find the inverse of f : R ! R defined by f ( x ) = 3x + 11.The function f is bijective.


y 11
Let y = 3x + 11 ) 3x = y 11 or x = 3 , i.e. f 1 ( x ) = x 311 .

p
Example 1.15 : It is very easy to prove that g( x ) = 1 + x 1 is the inverse of a restriction of the
function f ( x ) = x2 + 2x + 2. Hence the function g is the inverse of f .

20
Exercise 1.11
Given the rule of definition of f below, find a domain and range such that f defines a bijective
function. Find the inverse g of an appropriate restriction of the bijective function identified.
2x +3
(a) f ( x ) = 7x 2 .
1
(b) f ( x ) = (1 x )2
.
p
(c) f ( x ) = x2 1 .
p
(d) f ( x ) = x2 + 1.
(e) g( x ) = tan x
(f) h( x ) = e2x 5

(g) k ( x ) = sinh(3x 8)
(h) m( x ) = log(5x + 6)
1

1 Provided by Departments of Mathematics,UNIJOS and PLASU - 2013

21
Chapter 2

Limits and Continuity of Functions

2.1 Objectives

In this module, we give an elementary introduction to the concepts of limits and continuity of
a function, discuss methods of finding such limits and examine the properties of limits. De-
termination of limits of indeterminate forms will be considered in chapter three after exploring
derivatives of functions. We then define and explore continuous functions, state some useful
results without proof and show how to apply them. The exploration of the learning activities
will provide an opportunity for in-depth study of limits and continuity.

2.2 Learning outcomes

At the end of this module students should be able to

• Find the limit of a function at a point;

• Define and establish the continuity of some elementary functions.

2.3 Learning Activities


• Solve relevant questions from past MTH 103 examinations;

• Explore and use related materials on the Intranet, especially the e-granary and MIT open
courseware;

• Explore and use related materials on the Internet especially units 5 and 6 of Calculus of the
OLI courses.

22
2.4 Limit of a function

Definition 2.1 : Left Hand Limit( Limit from the left or limit from below): Let f be defined at all points
of the interval ( a, b) ( except possibly at the point x0 2 ( a, b) ); we say f tends to a limit l as x tends to x0
from the left or from below if given any e > 0, we can find δ(e) > 0 such that j f ( x ) l j < e provided
that x0 δ < x < x0 .We write this as f ( x ) ! l as x ! x0 ; or limx! x f ( x ) = l.
0

Definition 2.2 : Right Hand Limit( Limit from the right or limit from above ): Let f be defined at all
points of the interval ( a, b) ( except possibly at the point x0 2 ( a, b) ); we say f tends to a limit l as x
tends to x0 from the right or from above if given any e > 0, we can find δ(e) > 0 such that j f ( x ) l j < e
provided that x0 < x < x0 + δ.We write this as f ( x ) ! l as x ! x0+ ; or limx! x+ f ( x ) = l.
0

Definition 2.3 : The limit of a function: Let f be defined at all points of the interval ( a, b) ( except
possibly at the point x0 2 ( a, b) ); we say f tends to a limit l as x tends to x0 if given any e > 0, we can
find δ(e) > 0 such that if 0 < j x x0 j < δ then j f ( x ) l j < e or equivalently if x0 δ < x < x0 + δ
then l 2< f ( x ) < l + 2 .We write f ( x ) ! l as x ! x0 ; or limx! x0 f ( x ) = l.

Remark 2.1 : It should therefore be understood that when f is defined in an interval ( a, b) except
possibly for some point x0 2 ( a, b); f is said to have a limit l at x = x0 if and only if the left hand limit
and the right hand limit of f exist and are equal ( the same). The δ depends on 2 and sometimes on x.

Let us explore these definitions with some examples.

jxj
x , if x 6=0
Example 2.1 : Let f be defined by f ( x ) = f0,if x =0 .Evaluate the left and right limits at the point
x = 0.Does the function have a limit l at x = 0?

jxj j0 h j jhj
Solution 2.1 : Let h > 0,then limx!0 f ( x ) = limx!0 x = limh!0 0 h = limh!0 h =
jxj j0+ h j jhj
1. limx!0+ f ( x ) = limx!0+ x = limh!0+ 0+h = limh!0+ h = 1. Thus, the function has the
left and right limits at x = 0. For the left limit, this implies that given any 2> 0, we can a δ > 0 such
jxj jxj h
that δ < x < δ =) j f ( x ) ( 1)j <2 . But for 2> 0, x ( 1) = x +1 = h +1 =
h h 0
h = h = 0 <2 =) δ =2 . Similarly, for the right limit, this implies that given any 2> 0,
jxj jxj
we can find a δ > 0 such that δ < x < δ =) j f ( x ) 1j <2 . For 2> 0, x 1 = x 1 =
h h h 0
h 1 = h = h = 0 <2 =) δ =2 . But limx!0 f ( x ) does not exist since the left hand and

23
right hand limits though, they exist, they are not equal. The graph of the function is given below.

jxj
Fig.2.1 limx!0 ( x )

1 + x if x < 1
Example 2.2 : Let g be defined by g( x ) = .Evaluate the left and right limits at
3 x2 if x 1
the point x = 1.Does the function g have a limit l at x = 1?

24
Solution 2.2 : The graph of the function is given below.

1 + x if x < 1
Fig.2.2 g( x ) =
3 x2 if x 1

The piecewise function g has a break point at x = 1 we will compute its left and right limits.
Now, limx!1 g( x ) = limx!1 (1 + x ) = 1 + 1 = 2; and limx!1+ g( x ) = limx!1+ (3 x2 ) =
3 1 = 2.
It can be observed that the left and right limits both exist and are equal We have limx!1 g( x ) =
2 = g(1), Thus, the limit of the function g exist. Now given any 2> 0, we need to find a δ > 0
such that if 1 δ < x < δ then j g( x ) 2j <2 and if δ < x < 1 + δ then j g( x ) 2j <2 in each case
respectively. For the first case, j g( x ) 2j = j1 + x 2j = j x 1j <2 implies that 2< x
1 <2 which is equivalent to 1 2< x < 1+ 2 . This implies that 1 2< x < 1, with δ =2 . For
the second case, j g( x ) 2j <2= 3 x2 2 = 1 x2 = x2 1 = j( x 1)( x + 1)j < 3δ <2
implies that δ < x 1 < δ, Thus, δ = 23 . Here, j x + 1j j x 1 + 2j < j x 1j + 2 < 1 + 2 = 3,
where δ 1.

x2 9
Example 2.3 : Let h : R ! R be defined by h( x ) = x +3 for x 6= 3. Evaluate the limit of h at x = 3
and use 2 δ argument to justify your answer.

25
2 ( x 3)( x +3
limx! h( x ) = limx! 3 ( xx+39 ) = limx! 3
3 = limx! 3 ( x 3), x 6=
Solution 2.3 : Now, x +3
= limx! 3 ( 3 3) = 6. Thus, h has limit 6 at x = 3.
Given any 2> 0, we need to find δ > 0 such that jh( x ) h( 3)j <2 for 0 < j x ( 3)j = j x + 3j <
δ. But
x2 9 x2 9
jh( x ) h( 3)j = ( 6) = +6
x+3 x+3
x2 9 + 6x + 18
=
x+3
2
x + 6x + 9
=
x+3
( x + 3) ( x + 3)
=
x+3
= j x + 3j, x 6 = 3
= j x + 3j < ε ) Thus δ =2
Thus, δ =2 .

Example 2.4 : Let p : R ! R be defined by p( x ) = 9 4x. We shall evaluate its limit at x = 2 and
use 2 δ argument to justify the answer.

Solution 2.4 : It should be observed that the function p is defined for each real number x.Thus,
limx!2 p( x ) = limx!2 (9 4x ) = 9 4(2) = 9 8 = 1.Given any 2> 0, we need to find a δ > 0 such
that if j x 2j < δ then j p( x ) 1j <2 . Now, j p( x ) 1j = j9 4x 1j = j8 4x j = 4 j2 x j =
4 j x 2j <2 =) j x 2j < 24 . Thus δ = 24 .

Example 2.5 : Let F : R ! R be defined by F ( x ) = 3x2 4x + 5. Use 2 δ argument to show that F


has a limit at x = 2.

Solution 2.5 : Note here also that F is defined for every real value of x.Thus, limx!2 ( F ( x )) = limx!2 (3x2
4x + 5) = 3(2)2 4(2) + 5 = 12 8 + 5 = 9. Therefore the limit of F at x = 2 is 9.Given any
2> 0, we need to find a δ > 0 such that j F ( x ) F (2)j = j F ( x ) 9j <2 whenever j x 2j < δ.
Now,let j x 2j < δ < 1, then
j F ( x ) F (2)j = (3x2 4x + 5) (3(2)2 4(2) + 5)
= 3( x 2 4) 4( x 2) + 5 5 3( x 2 4) 4( x 2)
< j3( x 2)( x + 2) + 4( x 2)j < 3 j x 2j j x + 2j + 4 j x 2j
< 3δ j x + 2j + 4δ, But j x + 2j j x 2j + 4 < δ + 4 < 1 + 4 = 5
< 3(5)δ + 4δ = 19δ
2
i.e., j F ( x ) F (2)j < 19δ <2 ) δ = 19 ,

2x
p
Example 2.6 : limx!2 1 . Solution: limx!2 (2. ( x2 + 5)) = 6.
x ( x 2 +5) 2

Remark 2.2 : The last two examples show that if the function f is a polynomial and a is real number, the
limx! a f ( x ) = f ( a). This true for sine, cosine and exponential functions.

26
2.5 Properties of Limits (Theorem)

We shall state without proof some basic properties of limits of functions. Some of the proof may
be given in class but students are advised to consult some standard texts for the other proofs.

1. Suppose a function f has a limit l at a point x = a, then this limit l is unique. This implies
that a function CANNOT have two different limits at a point.

2. The limit of a constant function f ( x ) = k equals the constant k, i.e. limx!a k = k.

3. If limx! a f ( x ) = l1 and limx!a g( x ) = l2 then


(a) limx! a [ f ( x ) + g( x )] = limx! a f ( x ) + limx! a g( x ) = l1 + l2 (the limit of a sum of func-
tions is the sum of the limits) and
(b)limx! a [ f ( x ) g( x )] = limx!a f ( x ) limx! a g( x ) = l1 l2 (the limit of a difference of
functions is the difference of their limits).

4. limx! a [ f ( x ) g( x )] = limx!a f ( x ) limx! a g( x ) = l1 l2 (the limit of a product of functions is


the product of their limits.

5. In the special case when g( x ) = k, lim x!a k f ( x ) = k limx! a f ( x ),here k is a constant.


f (x) l1
6. limx! a g( x) = l2 , provided l2 6= 0, (the limit of a quotient of functions is the quotient of
their limits).

7. limx! a g( f ( x ) = g(limx!a ( f ( x )) whenever the limit of f exists.

Remark 2.3 : Properties 3 to 7 can be extended several functions whose limits exist.

Exercise 2.1
(a) Let f be defined by f ( x ) = f1x+ifx,if
x 1
x >1 Evaluate the right and left limits of f at the point x =
1.Evaluate limx!1 f ( x ) if it exists. Use 2 δ argument to justify your answer.
x2 4
(b) Evaluate the limit of x 2 at x = 2 and use 2 δ argument to justify your answer.
(c) Find the limit of 2x + 4 at x = 3 and use 2 δ argument to justify your answer.
3x2 2x 16
(d) Evaluate limx! 2 x +2 and use 2 δ argument to justify your answer.
(e) Find the limit of 5x 9 at x = 2 and use 2 δ argument to justify your answer.
2
(f) Let f be defined by g( x ) = f4x x1+ x,if
if x 1
x >1 Evaluate the right and left limits of f at the point
x = 1.Evaluate limx!1 g( x ) if it exists and use 2 δ argument to justify your answer.
if x 1
(g) Let f be defined by f ( x ) = f2x1+ x,if x >1 Evaluate the right and left limits of f at the point x =
1.Evaluate limx!1 f ( x ) if it exists and use 2 δ argument to justify your answer.

27
2.6 Bounded Functions

Definition 2.4 : A function f is said to be bounded above if 8 x 2 Dom( f ), f ( x ) K, and K is an


upper bound of f.Graphically this means that no part of the curve y = f ( x ) is above the line y = x.
Similarly, a function f is said to be bounded below if 8 x 2 Dom( f ), f ( x ) L, and L is a lower bound
of f. A function f that is both bounded above and below is said to be bounded. This implies that there is a
constant M such that j f ( x )j < M, i.e. M < f ( x ) < M 8 x 2 Dom( f ).

Definition 2.5 : A function that is not bounded is called unbounded.

Remark 2.4 : In order to show that a function is bounded above, it is necessary to find some value of K for
which f ( x ) K,but not necessarily the smallest such value. For instance sin x < 8. Infact sin x < K,for
any K 1.g( x ) = 1x is unbounded above for 0 < x < 1 but bounded below by 1 for 0 < x < 1.

Definition 2.6 : A function f ( x ) is said to tends to a value l as x sufficiently increases, if given any
2> 0,we can find a number X such that j f ( x ) l j <2, 8 x X.

Remark 2.5 : The important criterion in the above definition is that however small we take 2, we can
always find a value X for which the inequality in the definition holds.

3x2 +1
Example 2.7 : Evaluate limx!∞ x2 3
and Use 2
X argument to justify your answer. Solution: The
p
function is defined for all values of x except for x = 3.Thus, the function is defined for sufficiently
2
2 +1 3 x2 + 1
3+ 1
large x. limx!∞ 3x
x2 3
= limx!∞ x
x2
x2
3
= x2
3 = 3, since 1
x2
and 3
x2
both approach 0 as x approaches
1
x2 x2 x2
3x2 +1 10
infinity. We now prove that the limit is 3 using 2 X argument. Now j f ( x ) l j = x2 3
3 = x2 3
p 2 2 p
if x > 3. Now x2 3 > x2 x2 if x2 > 3. Hence j f ( x ) l j < 10 x2
, if x > 6.
2

20
p p p
<2 if x2 > 2 and x > 6.Thus, if x > X = max( 20/ 2, 6),then
j f (x) l j <2 .
Exercise 2.2
Evaluate the following limits:
m ), m 1
(a) limx∞ ( x > 0 ( Ans : l = 0, X = 1 + m2 )
1
(b) limx∞ (1 x) ( Ans : l = 1, X = 21 )
3 3x +6
(c) limx∞ ( 2x
x3 5x +1
) ( Ans : l = 2, X = max( p42 , 4)

Definition 2.7 : Let K be a real number and f : R ! R. Then we say that the function f ( x ) ! ∞ as
x ! a if for j x aj < δ, f ( x ) > K for any value of K > 0, however large. Similarly,we say f ( x ) ! ∞
as x ! a if for j x aj < δ, f ( x ) < K for any value of K > 0, however large. We can also say that f
diverges to ∞ (i.e. f ( x ) ! ∞ as x ! ∞) or diverges to +∞ (i.e. f ( x ) ! ∞ as x ! ∞).

28
x2 3
Example 2.8 : Show that limx∞ x +1 = ∞, that is the function is unbounded above.

2 2 x2
x2 3
Solution: Now x2 3 > x2 x2 if x2 > 3 and x + 1 < x + x if x > 1. Hence, x +1 > 2
2x = x
4 if
p 2 p
x> 6,i.e. xx+13 > K if x > X = max(4K, 6).
Exercise 2.3
Evaluate the following limits:
(a) limx!∞ ( x2 2x 1) 1 ( Ans.l = 0)
(b) limx!∞ ( x3x1 2x
x +1 ) ( Ans.l = 1)
5
(c) limx!∞ ( 3x +5
4x3 1
) ( Ans.l = ∞)

2.7 Limit of Indeterminate Forms


f (x) f (x)
Definition 2.8 : If as x ! x0 , g( x )
takes the form 00 , we say g( x )
has an indeterminate form. This is also
the case if as x ! x0 , g( x) takes the form ∞
f (x)
∞ . The limit of some functions with indeterminate form can be
evaluated using L’hospital’s rule. We shall use the L’hospital’s rule to investigate limits of such functions
after discussing derivatives in chapter 3. But for now, we shall consider some functions whose limits
can be evaluated by factorisation and then conclude with some limits that arise frequently in Science and
Engineering.

( x 3)( x +3) 2x x
Example 2.9 : Evaluate the following limits:(i) limx!4 px 4
x 2
(ii) limx!3 x 3 .(iii) limx!0 e e+x e 1 2

p p
p ( x +2 ) ( x 4) p
Solution (i) For x 6= 2, limx!4 px 4
x 2
= limx!4 ( pxx 42 . p xx+
+2
2
) = limx!4 ( x 4 ) = limx!4 x+
p
2j x6=4 = 4 + 2 = 2 + 2 = 4.

( x 3)( x +3)
(ii) Since x 6= 3, limx!3 g( x ) = limx!3 x 3 = limx!3 ( x + 3) = 6.
e2x +e x 2 (e x +2)(e x 1)
(iii) For x 6= 0, limx!0 ex 1 = limx!0 ex 1 = limx!0 (e x + 2) = e0 + 2 = 3.

Remark 2.6 : These last three examples are of indeterminate form in which substituting the correspond-
ing value of x in each of the functions gives 00 . But factorising the functions below evaluating the limits
give us the results. In cases where factorisation cannot be performed, we can employ the L’Hopital rule.
That will be considered in chapter three after treating differentiation.

29
2.8 Limits that arise Frequently

We state without proof some limits that arise very frequently in Mathematics, Science, Technol-
ogy and Engineering. Students can read their proffs in some standard texts.
(i) lim sinθ θ = 1.
θ !0

(ii) lim cos θθ 1


= 0.
θ !0
ln n
(iii) lim = 0, 8n 2 N.
n!∞ n
p
n
(iv) lim n = 1.
n!∞
1
(v) lim x n = 1, (for x > 0, 8n 2 N).
n!∞

(vi) lim x n = 0, (for j x j < 1, 8n 2 N) .


n!∞
x n
(vii) lim 1 + n = e x , (for any x, 8n 2 N) .
n!∞
xn
(viii) lim = 0, (for any x, 8n 2 N).
n!∞ n!
1
(ix) lim =0
n!∞ n
n
(x) limn!∞ en =0

Example 2.10 : (1) If j x j < 1, then lim x n+4 = lim x4 . lim x n = x4 .0 = 0.


n!∞ n!∞ n!∞
p p p
(2) lim n 2n = lim n 2 lim n n = 1.1 = 1
n!∞ n!∞ n!∞
2n n n
1 1 1
(3) lim 1+ n = lim 1 + n lim 1+ n = e.e = e2
n!∞ n!∞ n!∞
x n +1 x xn
(4) lim = lim = limn!∞ n! = 0.0 = 0.
n ! ∞ +1) !
( n n ! ∞ ( n +1)

2.9 Continuity

Definition 2.9 : A function f is continuous at a point x = a when lim f ( x ) = f ( a) if given any 2> 0
x!a
there exists a δ > 0 such that j x aj < δ implies that j f ( x ) f ( a)j <2 . This definition implies that
(1) the function f is defined at the point a,i. e. f( a) exists, (2) the limit of the function at x = a exists( i.e.
limx! a f ( x ) exists) and (3) limx!a f ( x ) = f ( a). If f is continuous at every ( each ) point in its domain
D, then f is a continuous function on D.

Remark 2.7 : Continuity of a function implies that we can draw the graph of the function without lifting

30
off our hand from the paper or board. A function which is continuous at a point a will always have a limit
at that point a. But not every function with a limit at a point is continuous at that point.

Definition 2.10 : A function f is discontinuous at a point a in its domain if it is not continuous there.

Example 2.11 Show that the functions below are continuous at the given points:

1. f ( x ) = 2x + 3 at the point x = 5.

2. g( x ) = x2 at x = 0
1
3. h( x ) = x and x = 5
p
4. s( x ) = x at x = 4

5. r ( x ) = x3 at x = 2

6. p( x ) = x4 at x = 2

7. q( x ) = 5x2 4x + 7 at x = 3.

Solution 2.6 : f ( 5) = 2( 5) + 3 = 10 + 3 = 7 = limx! 5 f ( x ). Given any 2> 0, we


need to find a δ > 0 such that j x ( 5)j = j x + 5j < δ =) j f ( x ) ( 7)j = j f ( x ) + 7)j <2 .
Now, j f ( x ) + 7)j = j2x + 3 + 7j = j2x + 10j = 2 j x + 5j <2 =) j x + 5j < 22 . This implies that
δ = 22 . Thus, for 2> 0, set δ = 22 . Then δ > 0 exists and satisfies j x + 5j < δ =) j f ( x ) + 7j =
j2x + 3 + 7j = j2x + 10j = 2 j x + 5j < 2 22 =2 . QED

Solution 2.7 : limx!0 g( x ) = limx!0 x2 = 0 = g(0). Given any 2> 0, we need to find a δ > 0 such
p
that j g( x ) 0j <2 =) if j x 0j = j x j < δ. But j g( x ) 0j = x2 = j x j2 <2 =) j x j < 2.
p p 2
Thus, for 2> 0 there is a δ > 0 namely δ = 2 for which j g( x ) 0j = j x j2 < 2 =2 QED.

1
Solution 2.8 : h(5) = 5 = limx!5 h( x ) =; limx!5 ( 1x ). Given any 2> 0, we need to find a δ > 0
1 1 1 1 5 x j x 5j
such that j x 5j < δ =) h( x ) 5 <2 .Now, h( x ) 5 = x 5 = 5x = 5j x j
<2 .
1 1
But j x 5j < δ =) j x 5j < 1 =) 1 < x 5 < 1 =) 4 < x < 6 =) 6 < x < 4
=) 16 < j x j < 14 . Thus h( x ) 15 = 1x 51 = 55xx = jx5jx5j j = jx5.45j = jx205j <2 =)
j x 5j < 20 2 . Thus, δ = minf1, 20 2g.Hence, for 2> 0, set δ = 20 2. Then δ > 0 exists and
j x 5j j x 5j j x 5j
satisfies j x 5j < δ =) h( x ) 51 = 1x 15 = 55xx = 5j xj = 5.4 = 20 < 20202 =2 . QED

p p
Solution 2.9 : limx!4 s( x ) = limx!4 ( x ) = 4 = 2 = s(4). Given any 2> 0, we needpto find a
p p p
δ > 0 such that j x 4j < δ =) js( x ) 2)j = x 2 <2 . But x 2 = ( x 2) p xx+ 2
+2
=
j x 4j p
p <2 The fraction p x1+2 is positive if x > 0 and maximum if x and x is minimum. So we take
j x +2 j j j

31
δ 4 so that j x 4j < 4 =) 0 < x < 8̇. Thus p1 2 ( p81+2 , 12 ) =) p x1+2 < 12 .Hence
j x +2 j j j j j
j x 4j j x 4j
p < 2 < 2δ <2 =) δ = 2 2 . Therefore given any 2> 0, there is a δ = minf4, 2 2g > 0
j x +2 j
p
such that for j x 4j < δ js( x ) 2j <2 . But js( x ) 2j = x 2 = pxx+42 < 2δ = 222 =2 .QED

Solution 2.10 : limx! 2 r ( x ) = limx! 2 x3 = ( 2)3 = 8 = r ( 2). Given any 2> 0, we


need to find a δ > 0 such that if j x + 2j < δ then jr ( x ) 8j <2 . But jr ( x ) 8j = x3 8 =
( ) x3 + 8 = x3 + 8 = j x + 2j x2 2x + 4 . But j x + 2j < δ =) j x + 2j < 1 =) 3 <
2
x < 1 =) j x j < 3. Thus, x2 2x + 4 j x j + 2 j x j + 4 < 32 + 2(3) + 4 = 19. Hence
2
jr ( x ) + 8j < 19δ <2 =) δ = minf1, 19 g.QED

Solution 2.11 : limx! 2 p( x ) = limx! 2 x4 = ( 2)4 = 16 = p( 2). It is easy to show that


2
δ = minf1, 65 g Student should demonstrate the result and then verify that it is indeed the value.

Solution 2.12 : q(3) = 5(3)2 4(3) + 7 = 40 = limx!3 q( x ). Given any 2> 0 we can find
a δ > 0 such that if j x 3j < δ then jq( x ) 40j <2 But jq( x ) 40j = 5x2 4x + 7 40 =
5x2 2x 33 = j x 3j j5x 11j < j x 3j j5x 11j < δ j5x 11j < (5 j x j + 11)δ < δ(5 j x 3 + 3j
11) < (5 j x 3j + 5(3) + 11)δ < (5(1) + 15 + 11)δ < 31δ <2, whenever j x 3j < δ, j x 3j < 1.
2 2
Thus jq( x ) 40j < 31 , =) δ = minf1, 31 g QED

4 x2
if x 6=2
Example 2.12 : (1) Given the function f defined by f ( x ) = fk2if xx=2 Find the value of k for the
sin x
x if x 6=0
function to be continue at x = 2.(2) Is the function s( x ) = f2 if x =0 .

2 (2+ x )(2 x )
Solution 2.13 : (1) limx!2 f ( x ) = limx!2 42 xx = limx!2 2 x = limx!2 2 + x j x6=1 = 2 +
2 = 4. Thus, f is continuous at x = 2 if k = 4, i.e. f (2) = 4. (2) Although limx!0 s( x ) = limx!0 sinx x =
0, the function is not continuous at x = 0 since limx!0 s( x ) = 1 but s(0) = 2 .The limit of the function
s and its value at x = 0 are not equal (are not the same).

Remark 2.8 : Some of the examples above illustrate the following. (1) Suppose f is a polynomial function
given by f ( x ) = an x n + an 1 x n 1 + ... + a1 x + a0 then limx! x0 f ( x ) = limx! a0 ( an x n + an 1 x n 1 +
... + a1 x + a0 ) = an x0n + an 1 x0n 1 + ... + a1 x0 + a0 = f ( x0 ).This implies that f is continuous at
pn ( x )
x = x0 . (2) Suppose f is a rational function with f ( x ) = qm ( x )
, wherepn ( x ) and qm ( x ) are polynomials
p (x) p (x )
such that qm ( x0 ) 6= 0 then limx! x0 f ( x ) = limx! x0 ( q n ( x) ) = q n ( x0 ) .This also shows that the rational
m m 0
functions f is continuous at x = x0 except at the zeros of qm .

Exercise 2.4
e2x 1 x2 3x +2 2
Evaluate the following limits (1) limx!0 e x 1 (2) limx !2 x 2 (3) limx! 4 16x+4x (4) limx! 2 3x2 +
2 +2
x 10.(5) limx! 3 4x2 5x + 6.(6) limx!∞ 3xx 2 +5
.Use 2 δ argument to justify limits of problems
2-5 and use 2 X argument to justify problem 6.

32
1

1 Provided by Departments of Mathematics, UNIJOS and PLASU - 2013

33
Chapter 3

The Derivative

3.1 Objectives

In this module, we expose students to the concept of differentiation emphasizing its dependence
on limits. We discuss rules for differentiation of different combinations of functions. We explore
the applications of derivatives to illustrate the use of calculus in the solution of real life problems.
Worked examples are provided to aid understanding of the material introduced. Group assign-
ments, take home tests and laboratory sessions will complement power point presentations in
tutorial type exchanges. All these activities expose the students to the concepts and techniques
and lead them to find out when the different techniques are most appropriate for use.

3.2 Learning Outcomes

At the end of this chapter, students should be able to:


Appreciate the dependence of the derivative on limits.
Apply the product, quotient and chain rules.
Differentiate implicit, exponential, logarithmic and hyperbolic functions.
Apply L’Hopital rule to evaluate limits of indeterminate forms.
Use the first and second derivative tests to investigate the maxima and minima of functions.
Investigate the Rolle’s and the Lagrange mean value theorems.

3.3 Learning Activities

Students should:

34
1. Explore notes and exercises, individually and in groups;
2. Explore and use related materials on the Intranet, especially the e-granary and MIT open
course ware;
3. Explore and use relevant materials from the MIT open course ware 18.01 Calculus;
4. Solve relevant questions from past MTH103 Examinations.

3.4 Introduction

The slope of a curve at any point a is the slope of the tangent to the curve at a. Let P( a, f ( a)) be
any point on the graph of a continuous function f and Q( a + h, f ( a + h)) be another point on
the curve close to P, where h is the difference between the x coordinates of Q and P.

Fig. 3.1 1st Principles Derivative

f ( a+h) f ( a)
The line PQ above is the secant line whose slope m PQ = h while the line L is the
tangent line at P.The tangent line at P is the limiting value of m PQ as Q approaches P.Hence,

35
f ( a+h) f ( a)
the slope m of the tangent of line L is given by: m = limh!0 h = f 0( a). The symbol
0
f ( a) read as f prime of a and it was introduced by Lagrange (1736-1813). The idea is that as
Q changes along the curve PQ, PQ becomes shorter secant line, and the lines through PQ, in the
limiting situation becomes the tangent line L.

Definition 3.1 : Let f be a function that is defined on an interval containing a.The slope m of the
f ( a+h) f ( a)
tangent line to the graph of f at the point P( a, f ( a)) is m = limh!0 h = f / ( a),provided the
limit exists. Thus, the derivative is the value of the slope m of the curve at point P.The concept of the
derivative of a function at a point, a as given above was invented by Isaac Newton (1642-1727). This
limit m, if it exists, is a function. This is the case since as a changes , so does f ( a) and each point in the
domain of f has a unique image which we call f 0 ( a). The new function is refered to as the derived function
f 0 ( x ).When we find f 0 ( x ) using this definition and the limiting process, we say that we are finding the
derivative f / ( x ) of f ( x ) from first principles.

Definition 3.2 : (Rate of change of a function). Given y as a function of x (as x changes, y also
changes), the rate of change of the function y is the change in y divided by the corresponding change in
x. That is, if xand x + ∆x are the values of x, with ∆x 6= 0; and y = f ( x ) and y + ∆y = f ( x +
∆x ) are the corresponding values of y, then the rate of change of the function as x changes from x to
f ( x +∆x ) f ( x ) ∆y
x + ∆x is given by ∆x = ∆x . The derivative of the function y = f ( x ) is therefore given by
f ( x +∆x ) f ( x ) ∆y dy
lim∆x!0 ∆x = lim∆x!0 ∆x .We write this last limiting value if it exists as dx and we call it a
differential coefficient or differential operator of y = f ( x ).

dy ∆y dy
Remark: Note that dx = lim∆x!0 ∆x is a limit and not a quotient. The differential operator dx
was first used by Liebnitz to denote the derivative of a function. This notion of derivatives as
the rate of change is applicable in a variety of disciplines including Biological, Social, Physical
sciences and Engineering.
In particular, if a body moves with its displacement given by s(t), the velocity v(t) of the body
s(t+∆t) s(t)
at time t is given by v(t) = ds
dt = lim∆t!0 ∆t . The derivative of the velocity v, dv
dt is the
dv d2 s
acceleration a, of the body given by a(t) = dt = dt2
.

3.5 Differentiability of a function at a point

Definition 3.3 : Let f be a function that is defined on an open interval containing a .The derivative of f
f ( a+h) f ( a)
at x = a written as f 0 ( a) is given by f 0 ( a) = lim h , provided f 0 ( a) exists and we say that f is
h !0
differentiable at a .If f is differentiable at a then f 0 ( a) is the slope of the tangent line to the graph of f at
the point ( a, f ( a)).

We explore the derivative of f ( x ) = x n for specific and general values of n.

36
f ( x +h) f ( x ) h
For the case where n = 1, f ( x ) = x, f ( x + h) = x + h, and limh!0 h = limh!0 h = 1,
d( x )
that is dx = 1.
f ( x +h) f ( x ) ( x + h )2 x 2
The case when n = 2,i.e. f ( x ) = x2 , f ( x + h) = ( x + h)2 , and limh!0 h = limh!0 h =
x2 +2xh+h2 x2 2xh+h2
limh!0 h = limh!0 h = limh!0 (2x + h) = 2x.
f ( x +h) f ( x )
Also the case when n = 3,we have f ( x ) = x3 , f ( x + h) = ( x + h)3 , and limh!0 h =
( x + h )3 x3 3x2 h+3xh2
limh!0 h = .lim h = limh!0 (3x2 + 3xh) = 3x2 .
h !0

Fnally, we explore the general case for any n 6= 1, f ( x ) = x n , f ( x + h) = ( x + h)n ,and


f ( x +h) f ( x ) ( x + h)n x n
limh!0 h = lim h ! 0 h = . limh!0 (nx n 1 + C2n x n 2 h + C3n x n 3 h2 + ... + hn 1 ) =
nx n 1 ,where n Cr = (n n!r)!r! . Taking the limit as h ! 0, we have all the remaining terms
C2n x n 2 h + C n x n 3 h2
3 + ... + hn 1 all go to zero.
We can show that when a function is differentiable at a point it is continuous there. This follows
f ( x ) f ( a)
since limx!a [ f ( x ) f ( a)] = limx! a [ x a .( x a)] = limx!a [ f (x)x f ( a)]
a ]. limx!a [( x
a)] = f / ( a).0 = 0, =) limx! a f ( x ) = f ( a). The converse of the result above is not true in
general. Show that it is not true by examining the function f ( x ) = j x j .
We can show that sin x is differentiable with derivative cos x. Also cos x is differentiable with
derivative sin x.
1
Exercise 3.1: Examine the differentiability of (i) j x 2j at x = 2 (ii) j2x 1j + 3x 2 at x = 2 (ii)
1
x at x = 0.
3

3.6 Methods of Differentiation

We shall in this section introduce some rules that simplify the task of finding derivatives of
different classes of functions and display their use.

3.6.1 Differentiation of Sums ( Difference) and Powers

We introduce some rules that will enable us to differentiate any polynomial.


We can show by using the first principles above that:
(i ). If f ( x ) = c = constant then f / ( x ) = 0, ( derivative of a constant is always zero).
(ii ).If f ( x ) = cg( x ) and g( x ) is differentiable with derivative g/ ( x ) then f / ( x ) = [cg( x )]/ =
cg/ ( x ).
(iii ).If f ( x ) and g( x ) are two differentiable functions with derivatives f / ( x ) and g/ ( x ) respec-
tively then [ f ( x ) g( x )]/ = f 0 ( x ) g0 ( x ).

37
How do we use the rules above to show that every polynomial is differentiable?
Exercise 3.1. Find the derivative of the functions below using the first principles or the methods
introduced above:

Example 3.1 : (i ). 4x100 ; ( Ans.400x99 )


(ii ).x4 + 2x.( Ans.4x3 + 2)
(iii ).( x4 + 2).( x4 2).( Ans.8x7 )
(iv).( x2 + 2)3 .( Ans : 6x ( x2 + 2)2 ).

Remark: The rule used above states that the derivative of a sum (difference) of differentiable
functions is the sum (difference) of their derivatives.

3.6.2 The Product Rule

This rule enables us to differentiate the product of two or more functions when we know the
derivatives of each one of them. It can be written in several ways and we present two below.
h
d[ f ( x ).g( x )] / = lim f ( x +h).g( x +h) f ( x ).g( x )] f ( x +h)f g( x +h) f ( x +h) g( x )g+ f ( x +h)
dx = [ f ( x ) .g ( x )] h !0 [ h = lim h !0 h
g( x +h) g( x )
limh!0 f ( x + h) h + g( x ) f (x+hh) f (x)

g( x +h) g( x ) f ( x +h) f ( x )
= limh!0 f ( x + h) limh!0 h + limh!0 g( x ) limh!0 h

Since f is differentiable and therefore continuous at x, we have limh!0 f ( x + h) = f ( x ) and the


limit becomes f ( x ) g0 ( x ) + g( x ) f 0 ( x ), i.e.

d[ f ( x ).g( x )] dg( x ) df
= f (x) + g( x )
dx dx dx

Example 3.2 : Suppose k( x ) = ( x2 2)(3x3 + 5x 2), we find k0 ( x ) using the product rule.

f 0 ( x ) = 2x and g0 ( x ) = 9x2 + 5.Thus k0 ( x ) = (3x3 + 5x 2)(2x ) + ( x2 2)(9x2 + 5).This is the


result.
We simplify for convenience only and we get: k0 ( x ) = 6x4 + 10x2 4x + 9x4 + 5x2 18x2 10 =
15x4 3x2 4x 10.
We note that we can express the product as a single polynomial and differentiate term by term.

38
3.6.3 The Quotient Rule

This rule facilitates the differentiation of the quotient of two differentiable functions. We present
it in two equivalent statements below.
f (x)
Suppose f and g are two differentiable functions of x then the quotient y = g( x )
is differentiable
and the derivative y0( x ) is
( )
f ( x +h) f (x)
y( x +h) y( x ) f ( x +h) g( x ) g( x +h) f ( x )
y0 ( x ) = limh!0 h = limh!0 g( x +h)
h
g( x )
= limh!0 hg( x ) g( x +h)
.We add
g( x ) f ( x +h) g( x ) f ( x )+ g( x ) f ( x ) f ( x ) g( x +h)
and subtract g( x ) f ( x ) to the numerator to get limh!0 hg( x +h) g( x )
=
g( x )[ f ( x +h) f ( x )] f ( x )[ g( x +h) g( x )]
g( x ) f 0 ( x ) f ( x ) g0 ( x )
limh!0 h
g( x +h) g( x )
h
.Taking the limits give y0 ( x ) = [ g( x )]2
; g( x ) 6=
0

4x3 2x2
Example 3.3 : Using the quotient rule we find y0 ( x ) if we are given y( x ) = 5x 3 .

We let f ( x ) = 4x3 2x2 =) f 0 ( x ) = 12x2 4x and g( x ) = 5x 3 ) g0 ( x ) = 5.


(5x 3)(12x2 4x ) (4x3 2x2 )(5) 40x3 46x2 +12x
Then y0 ( x ) = [5x 3]2
= (5x 3)2
.

h i/
1 [ g( x )]/
Corollary 3.1 : Let g be a function with g ( x ) 6= 0. Then = .
g( x ) [ g( x )]2

h i
1 g( x ).0 1g/ ( x )
We establish the above by letting f ( x ) = 1 in the quotient rule. Then 0= =
g( x ) [ g( x )]2
g/ ( x )
.
[ g( x )]2

Exercise 3.2:Differentiate the functions below and solve the other problems using the apropriate
methods.
(1) y( x ) = 14 x + 3x2 6x4 .
(2) y(r ) = r3 (4r3 8r2 + 2r ).
(3) y( x ) = (8x + 5x )(12x2 6).
(4) h ( x ) = 1
1 x + 1x2x .
(5) g(z) = z2 (3z2 4z + 1)(6z2 8).
( 35 t 1)
(6) f ( t ) = ( 22 +7)
.
t

(7) Find an equation of the tangent line to the graph of y = 3x2 + 4x 6 that is parallel to the
line y = 5x 2y 1 = 0.

39
(8) A ball rolls down an inclined plane such that the distance in (cm) it rolls in t seconds is
given by s(t) = 3t3 + 2t2 + 4 where 0 t 3.
(i ) find its velocity at t = 3, (ii ) find the time when the velocity is 35cm/ sec .
(9) An object was thrown upward such that its distance s(t) above the ground during the
first 15seconds is given by s(t) = 8 6t + t2 ,where s(t) is in metres and t in seconds. Find,
(i ) the velocity of the object at t = 2 and t = 4, (ii ) the velocity when the object is 50m above
the ground.

3.6.4 The Chain Rule

The rules of differentiation discussed so far are limited as they can only be used for sums, differ-
ences,products and quotients of differentiable functions. They cannot be used for differentiating
composite functions or function of functions such as ( x2 + 2x )3 . The chain rule gives us the way
for differentiating composite functions. Let the function be defined as y = f ( x ) = h( g( x )) with
h( x ) and g( x ) differentiable. The chain rule states that y0 ( x ) = f 0 ( x ) = h0 ( g( x )) g0 ( x ).A sketch
of the proof of the rule follows.
dy
Let f ( x ) = h ( g ( x )), and let u = g( x ) and y = h(u), then y = h(u), dx = f 0 ( x ) = h0 ( g ( x )) g0 ( x ) =
dy
h0 (u) g0 ( x ) = du . du
dx .
dy dy du
Thus, the chain rule becomes dx = du . dx .

Example 3.4 : Using the chain rule differentiate the functions below and discuss whether their deriva-
tives can be found in any other way.:

5
3x 4
(i ) y = cos( x2 + 3x ), (ii ) y = ( x3 4x2 + 6x )12 , (iii ) y = x 6 , (iv) y = 6x2 (5x2 7)3 .

Solutions:
du dy dy dy du
(i ). Let u = x2 + 3x, then y = cos u and dx = 2x + 3, du = sin u.Thus, dx = du dx =
(2x + 3) sin u = (2x + 3) sin( x2 + 3x )
dy dy dy
(ii ). Let u = x3 4x2 + 6x, then y = u12 and du
dx = 3x
2 8x + 6, du = 12u11 .Thus, dx = du
du
dx =
(3x2 8x + 6)12u11 = 12(3x2 8x + 6)( x3 4x2 + 6x )11
( x 6)3 (3x 4) dy
(iii ). Let u = 3x 4
x 6 , then y = u5 , and applying the quotient rule on u we have du
dx = ( x 6)2
, du =
5u4 .
dy dy ( x 6)3 (3x 4) dy ( x 6)3 (3x 4) 4
du 3x 4
Thus, dx = du dx = ( x 6)2
5u4 ) dx =5 ( x 6)2 x 6 .

(iv). Let u = 6x2 and v = (5x2 7)3 .


dy
Applying the product rule, we have that dx = v du dv
dx + u dx with
du
dx = 12x,

40
dk dv
To differentiate v we apply the chain rule, thus let k = 5x2 7, then v = k3 . dx = 10x and dk = 3k2
dv dk dv
dx = dx dk = (10x )3k2 = 30x (5x2 7)2 .
dy
Substituting t gives dx = (5x2 7)3 (12x ) + 6x2 (30x (5x2 7)) = 12x (5x2 7)3 + 180x3 (5x2 7)
Exercise 3.3: Find the derivatives of the functions below using the appropriate methods.
2
(i ) y = 5x3 6x +2
8
6x3 7
(ii ) y = x +5

(iii ) y = 6x3 cos(6x 4)


q
(iv) y = 6x7x5 2 5

(v) y = (8x4 5x3 + 3x2 )(3x8 + 2x3 5)3


3
sin 4x
(vi ) y = 1 cosx

3.6.5 Higher Derivatives and the Leibnitz Rule

We have seen that if f ( x ) is differentiable, we can find its derivative, a new function f / ( x ).If
f / ( x ) is itself differentiable, we can form its derivative, called the second derivative of f , and
denote by f // ( x ). As long as we have differentiability, we can continue in this way to get the
third f /// ( x ), fourth f 4 ( x ) and higher derivatives. We do not use dashes for more than the third
derivative.

Example 3.5 : Find the following derivatives.


(i ). f // ( x ) for f ( x ) = 7x8 6x5 .
(ii ). f 0 ( x ) for f ( x ) = ax4 .
(iii ). f 3 ( x ) for f ( x ) = (7 + 2x )2 .
ax +b
(iv). f ” ( x ) for f ( x ) = cx +d .

(v). f 10 ( x ) for f ( x ) = 7x8 6x5 .

2c(bc ad)
Solution(i) f 00 ( x ) = 392x6 120x3 (ii) f 0( x ) = 4ax3 (iii) f3 ( x ) = 0, (iv) f ( x ) = (cx +d)3
, (v)
f 10 ( x ) = 0
Leibniz Rule: Suppose u and v are functions whose nth derivatives at the point x exist. Then the
nth derivative of the product y = u( x )v( x ) exists and it is given by
dn y
dx n = D n (uv) = D n y( x ) = D n (u)v + nC1 D n 1 (u) D (v) + nC2 D n 2 ( u ) D 2 ( v ) + ... + nC D n r ( u ) D r ( v )
r
d dn n!
... + nCn uD n (v);where D = dx , D n = dx n , nCr = ( n r ) !r! .

41
Example 3.6 : (i) Find the fourth derivative of the function y = x5 (2x4 + 4x2 6x ) (ii) Find the third
derivative of y = x2 cos x,using the Leibnitz rule.

(i) Let u = x5 and v = 2x4 + 4x2 6x


du d2 u d3 u d4 u
dx = 5x4 , dx2
= 20x3 , dx3
= 60x2 , dx4
= 120x and
dv 2
d v d v 3 d v 4
dx = 8x3 + 8x 6, dx 2
2 = 24x + 8, dx3 = 48x, dx4 = 48.

Substituting the results into the formula gives


d4 y
dx4
= 120x (2x4 + 4x2 6x ) + 4(60x2 )(8x3 + 8x 6) + 6(20x3 )(24x2 + 8) + 4(5x4 )(48x ) + 48( x5 )

= 120x (2x4 + 4x2 6x ) + 240x2 (8x3 + 8x 6) + 120x3 (24x2 + 8) + 20x4 (48x ) + 48x5 =
240x5 + 480x3 720x2 + 1920x5 + 1920x3 1440x2 + 2880x5 + 960x3 + 960x5 + 48x5
= 6048x5 + 3360x3 2160x2 .
(ii) Let u = x2 and v = cos x
d3 y d3 u 3 C d2 u dv du d2 v 3
d v
dx3
= dx 3 v + 1 dx2 dx +3 C2 dx dx2 + u dx 3

du 2 3 2 3
dx = 2x, ddxu2 = 2, ddxu3 = 0, and dv
dx
d v
= sin x, dx 2 =
d v
cos x, dx 3 = sin x, substituting into the
formula gives
d3 y
dx3
= 6 sin x 6x cos x sin x = 5 sin x 6x cos x
Exercise 3.4: Find the nth derivatives of the following functions , as indicated, using the Leibniz
rule.
(i ). Fourth derivative of [ x5 ( x2 + 5x )]
(ii ).Third derivative of [ x4 (9x2 x )]
(iii ).Second derivative of (sin x cos x )
(iv) Fourth derivative of [cos 3x ( x2 + 5x )]
(v). Fifth derivative of ( x5 4x )( x2 + 5x )

3.6.6 Implicit Differentiation

A function can be defined in two ways; either explicitly or implicitly. When a function is ex-
pressed solely in terms of the independent variable, such a function is called an explicit func-
tion. An example of an explicit function is f ( x ) = x2 + 3x + 2 sin x. When a function is not
expressed solely in terms of the independent variable, such a function is said to be implicit. In
other words, an implicit function is one which does not present an explicit formula for its val-
ues, but rather defines it by giving conditions that it satisfies. Thus, its values must be inferred
as consequences of the definition. An example of an implicit function is 3x2 + y3 + xy = 3xy2 .
In this section, we shall consider implicit functions and how to differentiate them. We take the

42
derivative of implicit functions just as we take that of the explicit functions. This is done by
taking the derivative of both sides of the equation, applying the rules of differentiation, remem-
bering to treat the independent variable as a function of the dependent variable, and solving
for the derivative. Another alternative method is to solve for the dependent variable and then
differentiate the resulting equation. We illustrate this with examples.

p 3
Example 3.7 : Suppose (i) 8xy 16y2 = x2 ,(ii) 4x2 y2 + 3y3 + 4x2 = 16xy2 ;(iii) x y + 1 = xy 2 +
dy
1;(iv) cos xy = y2 + x2 , find dx .
(v) Obtain the derivative of the function 3x2 + xy3 = 4x3 y at the point ( 1, 0).

Solution:
(i) To solve this problem, differentiate both sides of the equation.
d d d
8 dx ( xy) 16 dx ( y2 ) = 2
dx ( x ).Use a combination of chain rule and product rule to get,
dy dy dy dy
8[ x dx + y] 16(2y dx ) = 2x =) 8x dx + 8y 32y dx = 2x.
Rearrange and collect like terms together.
dy dy dy
8x dx 32y dx = 2x 8y =) (8x 32y) dx = 2x 8y
dy 2x 8y dy 2( x 4y)
Therefore, dx = 8x 32y =) dx = 8( x 4y)
= 14 .

Alternative method. Rearrange 8xy 16y2 = x2 to give ,


0 = x2 8xy + 16y2 = ( x 4y)2 .Taking the square root, we have,
x
0=x 4y ) y = 4
dy
Therefore, dx = 14 .
d d d d
(ii) Differentiate both sides 4 dx ( x2 y2 ) + 3 dx (y3 ) + 4 dx ( x2 ) = 16 dx ( xy2 )
Using product and chain rule, we have
dy dy dy
4[2xy2 + x2 (2y dx )] + 3(3y2 ) dx + 4(2x ) = 16[y2 + 2xy dx ]
dy dy dy
8xy2 + 8x2 y dx + 9y2 dx + 8x = 16y2 + 32xy dx
dy dy dy
8x2 y dx + 9y2 dx 32xy dx = 16y2 8xy2 8x
dy
(8x2 y + 9y2 32xy) dx = 16y2 8xy2 8x
dy 16y2 8xy2 8x
Therefore, dx = 8x2 y+9y2 32xy

d
p d 3 d
(iii) Differentiate across the entire equation implicitly. dx ( x y + 1) = dx ( xy 2 ) + dx (1)
d
p p d d 3 3 d
By product rule we have, x dx ( y + 1) + y + 1 dx ( x ) = x dx (y 2 ) + y 2 dx (x)

43
p p
dy 3x y dy 3
px + y+1 = 2 dx + y2
2 y+1 dx
p p
3x y dy 3
This implies that ( px 2 ) dx = y2 y+1
2 y +1

3 p 3 p 3 p p
y2 y +1 y +1
dy y2 y +1 y2 y +1
Therefore, dx = p
3x y
= p
3x y(y+1)
= p .
px px p x 3x y ( y +1)
2 y +1 2 2 y +1 2 y +1

(iv) Differentiate both sides of the equation implicitly,


dy dy dy dy
(sin( xy)) y + x dx = 2y dx + 2x =) x sin xy dx 2y dx = 2x + y sin xy
dy dy 2x +y sin xy 2x +y sin xy
factorise the derivative, to get ( x sin xy 2y) dx = 2x + y sin xy ) dx = x sin xy 2y = ( x sin xy+2y)
.

(v) Differentiate across the entire equation, to have,


d d d dy dy
3 dx ( x2 ) + 3
dx ( xy ) = 4 dx ( x3 y) =) 6x + 3xy dx + y3 = 4x3 dx + 12x2 y
dy dy dy dy 12x2 y 6x
3xy dx 4x3 dx = 12x2 y 6x =) (3xy 4x3 ) dx = 12x2 y 6x =) dx = 3xy 4x3
.
dy 12x2 y 6x
At ( 1, 0) , dx j( 1,0)
= 3xy 4x3 (
j 1,0)
= 32 .

3.7 Derivatives of Logarithmic Functions

Let f : < ! < be a real-valued function, then f is said to be a logarithmic function if f ( x ) =


loga u( x ), where u( x ) > 0, is a function of x and a is the base. If a = e, then we have the natural
logarithmic function and we sometimes denote it as ln u. When u( x ) = x, we have f ( x ) = ln x.
In this section, we explore the derivatives of the logarithmic functions. Suppose that the function
f is given by f ( x ) = ln x. We find the derivative of the function using first principles. Given
that f ( x ) = ln x.
f ( x +h) f ( x )
From the definition of derivative, f 0 ( x ) = lim h = lim ln(x+hh) ln x
= lim 1h ln( x+x h )
h !0 h !0 h !0
h
= lim 1h ln(1 + x ).(From the laws of logarithm, i.e. we recall the definitions of the exponential
h !0
and logarithm functions and the relationship between them.)
Letting z = 1x , we have f 0 ( x ) = lim 1h ln(1 + zh).
h !0

Taking the exponential of both sides we have,


1 1
0 h n 0 (x)
ef (x) = lim eln(1+zh) = lim (1 + zh) h .If we set n = 1
h , then lim (1 + nz ) = ez , hence e f = ez .
h !0 h !0 n!∞
Since they have the same base, it means that f 0 ( x ) = z;
But then z = 1x , ) f 0 ( x ) = 1x . Therefore, the derivative of f ( x ) = ln x is 1x .

44
An alternative method of finding the derivative of f ( x ) = ln x is to use the concept of implicit
differentiation. We proceed as follows:
Let y = ln x. Then taking the exponential of both sides we have, ey = eln x ) ey = x
dy dy
Differentiate implicitly with respect to x to get ey dx = 1.Making dx the subject of the formula,
we have
dy 1 dy
dx = ey but ey = x.Therefore dx = 1x .
dy 1 du
In general, if u a is differentiable function of x then dx = u( x ) dx
.
dy
Let y = ln u( x ),then ey = eln u( x) . This means that ey = u( x ). By implicit differentiation,ey dx =
du dy 1 du dy 1 du
dx ,hence dx = ey dx . But ey = u( x ) ) dx = u( x ) dx
.

Example 3.8 : (i) Find the derivative of the function f ( x ) = ln( x2 + 3); (ii) Differentiate ln[( x2
1)( x3 + 2)];
p
(iii) Find the derivative of f ( x ) = ln x;(iv) Differentiate log3 (2x3 3x + 1);(v) Differentiate
log2 x.
Solution:
du
(i) Let y = f ( x ) and u( x ) = x2 + 3.This means that y = ln u and dx = 2x.
dy 1 du dy 1 d 2x
Since dx = u( x ) dx
it means that dx = x2 +3 dx
x2 + 3 = x 2 +3

(ii) Let y = ln[( x2 1)( x3 + 2)] then y = ln( x2 1) + ln( x3 + 2)


Differentiating the function with respect to x we have
dy 1 1 2x ( x3 +2)+3x ( x2 1) 2x4 +3x2 x
dx = ( x2 1)
(2x ) + ( x 3 +2)
(3x ) = ( x2 1)( x3 +2)
= ( x2 1)( x3 +2)
.
p 1
(iii) f 0 ( x ) = p1 d ( x ) = p1 . 1 x = 1
2x .
2
x dx x 2

The change of base formula allows us to rewrite a logarithm of any base in terms of natural
loge u( x ) dy d loge u( x )
logarithm. For instance loga u( x ) = loge a . The derivative of y = loga u( x ) is dx = dx ( loge a )
1 d 1 du
= loge a dx (loge u( x )) = u( x ) loge a dx
. Note that ln u( x ) is the same as loge u( x ).
du dy
(iv) Let y = log3 u, where u( x ) = 2x3 3x + 1. This means that dx = 6x2 3, and so dx =
1
(2x3 3x +1) ln 3
(6x2 3).
ln x
(v) Let y = log2 x = ln 2 .
dy d ln x 1 d 1
Therefore dx = dx ( ln 2 ) = ln 2 dx (ln x ) = x ln 2

The function log x is a monotonic function which assumes all values between ∞ and +∞ as
the independent variable x ranges over the continuum of positive numbers.

45
Logarithmic differentiation is a technique that is employed to take the derivatives of functions
involving complicated combinations of products, powers and especially quotients. To carry out
differentiation using this technique, we carry out the following steps.
(1) Take natural logarithms of both sides of the equation y = f ( x ).
(2) Differentiate implicitly with respect to x.
dy
(3) Solve for dx , replacing y by f ( x ) every where it occurs in the final result.

1
( x 2) 2
Example 3.9 : (i) Differentiate the following y = xsin x ;(ii) y= 3 ;(iii) y = ln(sec x +
( x3 +2) 4 ( x6 +3x 1)2
ln x
tan x ); (iv) w = 1+ x 2
.

Solution:
1 dy 1
(i) y = xsin x ) ln y = ln xsin x ) ln y = sin x ln x ) y dx = x sin x + ln x cos x
dy dy
dx = y( 1x sin x + ln x cos x ) =) dx = xsin x ( 1x sin x + ln x cos x ).
(ii) Taking the logarithm to base e of both sides, we have
1
( x 2) 2
ln y = ln 3 = 12 ln ( x 2) 3
4 ln x3 + 2 2 ln x6 + 3x 1
( x3 +2) 4 ( x6 +3x 1)2

1 3 y0 1 9 x2 +3 5
ln y = 2 ln ( x 2) 4 ln x3 + 2 2 ln x6 + 3x 1 =) y = 2( x 2) 4 x 3 +2 2 x66x
+3x 1

1
9 x2 5
+3 ( x 2) 2 9 x2
Example 3.10 : y0 = y( 2( x1 2) 4 x 3 +2 2 x66x
+3x 1
); y 0 = 3 ( 2(x1 2) 4 x 3 +2
( x3 +2) 4 ( x6 +3x 1)2
5 +3
2 x66x
+3x 1
)
1
y0 = p p 7 3
55x9 + 45x4 109x3 + 92x6 + 36x 92 114x8 + 18x2 192x5
4
4 ( x 2) ( x 3 +2) ( x6 +3x 1)

(iii) Taking the logarithm of both sides to base e, we have


ln y = ln ln(sec x + tan x ).Differentiate implicitly to have
y0 sec x tan x +1+tan2 x x tan x +1+tan x 2
y = (sec x +tan x ) ln(sec x +tan x )
=) y0 = y (sec sec
x +tan x ) ln(sec x +tan x )

x tan x +1+tan x 2 sec x tan x + sec2 x sec x (tan x + sec x )


y0 = ln(sec x + tan x ) (sec sec
x +tan x ) ln(sec x +tan x )
=) y0 = (sec x +tan x )
= (sec x +tan x )
= sec x
(iv) Taking the logarithm to base e of both sides, we have
ln x w0
ln w = ln 1+ x 2
= ln (ln x ) ln 1 + x2 =) w = 1
x ln x 2 1+xx2

1 x2 +2x2 ln x
w0 = w 1
x ln x 2 1+xx2 = ln x
1+ x 2
1
x ln x 2 1+xx2 = 2
(1+ x 2 ) x

46
3.8 Differentiation of Exponential Functions

Suppose a > 0, the function f : < ! <+ , defined by f ( x ) = a x , ,(where x 2 <) is called an
exponential function.
x+h x ( ah h
f ( x +h) f ( x ) ax 1)
Using first principles, f 0 ( x ) = lim h = lim a h = lim a h = a x lim (a h
1)
h !0 h !0 h !0 h !0

( a h 1)
But lim is a constant depending on the value of the base a. By power series expansion,
h !0 h

ah = 1 + (ln a) h + 1
2 ln2 a h2 + 1
6 ln3 a h3 + 1
24 ln4 a h4 + ...

( ah 1) = (ln a) h + 1
2 ln2 a h2 + 1
6 ln3 a h3 + 1
24 ln4 a h4 + ...

1 h
h (a 1) = (ln a) + 1
2 ln2 a h + 1
6 ln3 a h2 + 1
24 ln4 a h3 + ...

lim 1h ( ah 1) = lim (ln a) + lim 1


2 ln2 a h + lim 1
6 ln3 a h2 + ...
h !0 h !0 h !0 h !0

Therefore, lim 1h ( ah 1) = (ln a) .


h !0

Now returning to our definition, we have that f 0 ( x ) = a x ln a. A special case of this occurs when
dy
a = e = 2.71828... That is, if y = e x , then dx = e x .

In general, if u is a differentiable function of x, and y = f ( x ) such that y = au( x) then the


following results hold:
dy
(a) dx = au(x) ln a du
dx
dy
(b) dx = eu(x) du
dx (In this case y = e
u( x ) ).

Example 3.11 : Differentiate the following functions.

2 +4x +1 3 +2 3 +2)
(i) (a) y = a3x (b) f ( x ) = 4x (c) f ( x ) = 5sin( x
(ii) Differentiate the following functions. (a) e3x (b) e 4x ( x3 + 1)
(iii) A radioactive substance decays according to the formula q(t) = q0 e ct ,where q0 is the ini-
tial amount of substance, c is a positive constant and q(t) is the amount remaining after time
t. Show that the rate at which the substance decays is proportional to q(t).
Solution:
2 +4x +1
(i) (a) Given that y = a3x . Here u( x ) = 3x2 + 4x + 1. This means that du
dx = 6x + 4. Therefore,
dy dy 2 +4x +1 2 +4x +1 2 +4x +1
dx = au(x) ln a du
dx = dx = ( a3x ) ln a(6x + 4) = 2( a3x )(3x + 2) ln a = 2 ln a( a3x )(3x +
2)

47
3 +2 dy d x 3 +2 dy 3 +2
(b) Let y = f ( x ) = 4x , then dx = dx (4 ). This implies that dx = (4 x ) ln 4(3x2 ) =
3
(3x2 )(4x +2 ) ln 4.
3
(c) Let y = 5sin( x +2) . Then taking the natural logarithm of both sides, we have ln y = sin( x3 +
dy d
2) ln 5. Now differentiating both sides implicitly, we have 1y dx = cos( x3 + 2) dx ( x3 + 2) ln 5
dy 3 +2)
dx = y[3x ln 5 cos( x3 + 2)] = 5sin(x [3x ln 5 cos( x3 + 2)].
dy d dy
(ii) (a) Let y = e3x , then dx = e3x dx (3x ). This means that dx = 3e3x .
(b) Let y = e 4x ( x3 + 1). This particular problem requires the usage of product rule and expo-
nential differentiation.
dy 4x d ( x3 d
dx =e dx + 1) + ( x3 + 1) dx (e 4x ).

dy
dx = 3x2 e 4x + ( 4e 4x )( x3 + 1) = 3x2 e 4x 4e 4x ( x3 + 1) = (3x2 4( x3 + 1))e 4x = (4x3
3x2 4))e 4x .

(iii) To obtain the rate at which the substance is decaying, is equivalent to finding the first deriv-
ative of the function q(t) = q0 e ct with respect to the independent variable t.
dq d ct ) dq ct d ( dq ct
dt = dt ( q0 e ) dt = q0 e dt ct) ) dt = cq0 e
Exercise 3.5: Find the derivative of the following functions:
r
p 4x4 3
1. (i) y = ln( x6 + 3x2 + 1) (ii) y = ln 2x x2 + 4 (iii) ln (5x3 +2) 3 (iv) y = log10 ( x2
p e +3x +1 x6 2
x) (v) y = ln( cos 3x ln 4x2
) (vi) ln(sec x + tan x ) (vii) ln(cos x )
4
( x2 3x 1) (cos2 x)
2. Using logarithmic differentiation, differentiate with respect to x: (i) 3 (ii)
p ( x6 3x2 ) 2
x
3 4x (iii) 1ln
+ x2
(iv) cos( x )sin x
p
e x x 5 x 3 +2
(v) ln ln( x ) (vi) x y = y x . (vii) 5 3
( x +2) 2 ( x 2 +3)

3. Find the derivative of following functions with respect to the independent variable indicated.
2 2 3
1
(a) e x + e1x (b) (e3x e 3x )4 (c) e3x
sin x
(cos 3x ) (d) sec2 (e 3x ) (e) e ( x 1) (f) ecos x (ln x3 1 2
).
2
x +1
4. If a drug is injected into the bloodstream, then its a concentration t minutes later is given by
c(t) = α bt at )
a b (e e for positive constants a, b and α.
(a) At what time does the maximum concentration occur?
(b) What can be said about the concentration after a long period of time?

48
3.9 Derivatives of Hyperbolic Functions

Recall that the six hyperbolic functions are:


ex e x ex + e x sinh x ex e x cosh x ex + e x
sinh x 2 , cosh x 2 , tanh x cosh x = x
e + e x , coth x sinh x = ex e x
,
1 2 1 2
sechx cosh x = ex + e x cos echx sinh x = ex e x .
The following identities are true of the hyperbolic functions.
cosh2 x sinh2 y = 1, sech2 x + tanh2 x = 1 and coth2 y cos ech2 y = 1.
The derivatives of the hyperbolic functions can be obtained easily when written in terms of
exponentials. Since the derivative of e x is still e x and the derivative of e x is e x , we can
differentiate the above equations to have:
d(sinh x ) d ex e x d d e x +e x
dx = dx 2 = 21 (e x + e x) = cosh x and dx cosh x = dx 2 = 12 (e x e x) =
sinh x.
Notice that the derivative of hyperbolic trigonometric sinh x is cosh x and the derivative of
cosh x is sinh x.
This is unlike the trigonometric version, where the derivative of cos x is sin x and the deriva-
tive of sin x is cos x.
The derivatives of the remaining four functions can be obtained in a similar manner. That is
d d
d tanh x d ex e x cosh x dx (sinh x ) sinh x dx (cosh x ) d tanh x cosh2 x sinh2 y
= e x +e x
= 2 (quotient rule). i.e. = =
dx dx [cosh x ] dx cosh2 x
2
1 tanh x = 12 = sech2 x.
cosh x
d d
d coth x sinh x dx (cosh x ) cosh x dx (sinh x ) sinh2 y cosh2 x (cosh2 x sinh2 y) 1
= [sinh x ]2
(quotient rule) = = = =
dx sinh2 x sinh2 x sinh2 x
2
cosech x.
d d
d sec hx cosh x dx (1) (1) dx (cosh x ) sinh x
dx = [cosh x ]2
= [cosh x ]2
= sechx tanh x.
d d
d cos echx sinh x dx (1) (1) dx (sinh x ) cosh x
dx = [sinh x ]2
= [sinh x ]2
= coth x cosechx.

In general, if u is a differentiable function of x, then


d sinh u( x ) d cosh u( x ) d tanh u( x ) d coth u( x ) d sec hu( x )
dx = cosh u dudx , dx = sinh u du
dx , dx = sech2 u du
dx , dx = cosech2 u du
dx , dx
d cos echu ( x )
sech(u) tanh u du
dx , dx = coth u csch(u) du dx .

Example 3.12 : Differentiate the following hyperbolic functions.(i) sinh(4x2 ), (ii ) cosh (e x + x ) (iii ) tanh

Solution:
d dy
(i) Let y = sinh(4x2 ) and u( x ) = 4x2 , then dx = cosh(4x2 ) dx (4x2 ) = cosh(4x2 )(8x ) = 8x cosh(4x2 ).

49
dy d
(ii) Let y = cosh (e x + x ) and u( x ) = e x + x, then dx = sinh(e x + x ) dx (e x + x ) = (e x +
1) sinh(e x + x ).
dy d
(iii) Let y = tanh(3x ) and u ( x ) = 3x then dx = sech2 (3x ) dx (3x ) = 3 sech2 (3x ).

3.10 Derivatives of Inverse Trigonometric Functions

The inverse trigonometric functions are:


1 π
(i) y = sin x if x = sin y,and 2 < y < π2 .
(ii) y = cos 1 x if x = cos y, and 0 < y < π.
(iii) y = tan 1 x if x = tan y, and π
< y < π2 .
2

(iv) y = cot 1 x if x = cot y, and 0 < y < π.


(v) y = sec 1 x if x = sec y, and 0 y< π
and π
<y π.
2 2

(vi) y = csc 1 x if x = csc y, and π


y < 0 and 0 < y π
2 2.
To obtain the derivative of the six inverse trigonometric functions, we proceed as follows.
1
(i) Let y = sin x,where x 2 [ 1, 1]. Rearrange the expression to have
dy dy
x = sin y. Differentiate implicitly to have 1 = cos y dx . This means that dx = cos1 y .
q
dy
But cos y = 1 sin2 y. Thus dx = cos1 y = p 1 2 = p 1 2 , (since sin2 y = x2 ).
1 sin y 1 x

(ii) Let y = cos 1 x where x 2 [ 1, 1]. Rearrange the expression to have


x = cos y. Differentiate implicitly to have
dy dy 1
p
1= sin y dx . This means that dx = sin y . But sin y = 1 cos2 y
dy 1 1
Thus = = p = p 1 ,(since cos2 y = x2 ).
dx sin y 1 cos2 y 1 x2

(iii) Let y = tan 1 x where x 2 ( ∞, ∞). Rearrange the expression to have


x = tan y. Differentiate implicitly to have
dy dy 1
1 = sec2 y dx . This means that dx = sec2 y
. But sec2 y = 1 + tan2 y
dy 1 1 1
Thus dx = sec2 y
= 1+tan2 y
= 1+ x 2
,(since tan2 y = x2 ).

(iv) Let y = cot 1 x where x 2 ( ∞, ∞). Rearrange the expression to have


x = cot y. Differentiate implicitly to have

50
dy dy 1
1= csc2 y dx . This means that dx = csc2 y
. But csc2 y = 1 + cot2 y
dy 1 1 1
Thus dx = csc2 y
= 1+cot2 y
= 1+ x 2
, (since cot2 y = x2 ).

(v) Let y = sec 1 x where j x j> 1. Rearrange the expression to have


x = sec y. Differentiate implicitly to have
dy dy +p
1
1 = sec y tan y dx . This means that dx = sec y tan y . But tan y = sec2 y 1
dy 1 1
Thus = = p = p1 , (since sec2 y = x2 ).
dx cos y sec2 y 1 sec y (x x 2 1)

dy
= p1 , j x j> 1.
dx j x j x2 1

(vi) Let y = csc 1 x where j x j> 1. Rearrange the expression to have


x = csc y. Differentiate implicitly to have
dy dy +p
1
1= csc y cot y dx . This means that dx = csc y cot y . But cot y = csc2 y 1

dy +
1 1 p1
Thus dx = csc y cot y = p = , (since csc2 y = x2 ).
csc2 y 1 csc y (j xj x 2 1)

Generally, if u is a differentiable function of x,then


d 1 p 1 du d 1 u) p 1 du
dx (sin u) =
1 u2 dx
, dx (cos =
1 u2 dx
,
d 1 u) 1 du d 1 u) 1 du
dx (tan = 1+ u2 dx
, dx (cot = 1+ u2 dx
,
d 1 u) p1 du d 1 u) p1 du
dx (sec =
juj u2 1 dx
, dx (csc =
juj u2 1 dx

We shall attempt to prove the first two and leave the rest as exercises.
1
(i) Let y = sin u, where u is a function of x. Then u = sin y. By implicit differentiation, we
have
du dy dy
dx = cos y dx . Making dx ,the subject we have
dy 1 du 1 du du
= =p = p 1 ,(since u2 = sin2 y).
dx cos y dx 1 sin2 y dx 1 u2 dx

This gives the formula


d 1 p 1 du
dx (sin u) =
1 u2 dx
.

(ii) Let y = cos 1 u, where u is a function of x.Then


u = cos y. By implicit differentiation, we have
du dy dy
dx = sin y dx . Making dx , the subject we have

51
dy 1 du 1 du du
= = p = p 1 ,(since u2 = cos2 y).
dx sin y dx 1 cos2 y dx 1 u2 dx

This gives the formula


d 1 u) p 1 du
dx (cos =
1 u2 dx
.
Differentiate the following inverse trigonometric functions.
4
1 1+tan 1 x 1
(a) y = x3 cos 1 (4x3 1) (b) y = e x sin ( x2 2) (c) y = 4 5 tan 1 x
(d) y = sin ( x3 ) .

Solution:
(a) Let y = x3 cos 1 (4x 3 1). Apply the product rule. Then
dy d d 1 d
dx = x3 dx cos 1 (4x 3 1) + cos 1 (4x3 1) dx ( x3 ) = x3 p dx 4x3 1 +
1 (4x3 1)2
3x2 cos 1 (4x 3 1)

1 12x5
= x3 p 12x2 + 3x2 cos 1 (4x3 1) = p + 3x2 cos 1 (4x3 1)
1 (4x3 1)2 1 (4x3 1)2
q
4x3 (cos 1 (4x3 1)) ( 4x4 +2x )+2x2
= 3x2 cos 1 (4x3 1) p = 3x2 q
1 (4x3 1)2 ( 4x4 +2x )

1
(b) Let y = e x sin ( x2 2) . Apply the product rule. Then
dy d 1 1 d 1 d 1
dx = e x dx sin ( x2 2) + sin ( x2 2) dx (e x ) = ex p dx x2 2 + sin ( x2
1 ( x 2 2)2
2) e x
q
1
2x ex 1 2x +(sin ( x2 2)) ( 3 x4 +4x2 )
=p + sin ( x2 2) e x . = e x q .
1 ( x 2 2)2 ( 3 x4 +4x2 )

1+tan 1 x
(c) Let y = 4 5 tan 1 x
. Apply the quotient rule. Then
1 1 1
dy (4 5 tan 1 d
x ) dx (1+tan 1 x) d
dx (4 5 tan 1 x) (4 5 tan x)
1+ x 2
(1+tan 1 x) ( 5)
1+ x 2
dx = 2 = 2 =
(4 5 tan 1 x) (4 5 tan 1 x)
9
2 .
(1+ x 2 ) ( 4+5 tan 1 x)

4
1
(d) Let y = sin ( x3 ) . Apply the chain rule. Then

dy 3 3
1 d 1 1 1 d
dx = 4 sin ( x3 ) dx sin ( x3 ) = 4 sin ( x3 ) p x3
1 ( x3 )2 dx

3 12x2 (sin 1 ( x3 ) 3
1 p 3x
2 )
= 4 sin ( x3 ) = p .
1 x6 1 x6
Exercise 3.6:
1. Differentiate the following inverse trigonometric functions with respect to x.

52
1 1 (2x 3 e x ) 2x
(a) sin (4x3 3) (b) cos 1 (sin x ) (c) cos (d) tan 1
1 x
1 x ,show d2 y dy
2. If y = etan that 1 + x2 dx2
+ 2x (1 + x2 ) dx y = 0.
1
dy
3 If y= 1 x2 2
cos 1 x, prove that 1 x2 dx + xy + 1 x2 = 0.
3 x2 1
4. Differentiate (a) y = x2 tan 1 x (b) y = tan 1 e (c) y = x arctan x
dy 1 (ln x )
5. Find dx if y = exp x cos .

3.11 Derivatives of Inverse Hyperbolic Functions

The inverse hyperbolic functions can be differentiated by the following technique.


1) Write the relationship in such a way that the independent variable becomes the dependent
variable. For example if y = sinh 1 x, then rewrite as x = sinh y.
2) Differentiate the equation implicitly
dy
3) Rearrange to make derivative (in this case dx ),the subject of the formula.
4) Using an appropriate identity, write the expression in terms of the independent variable.

1 1 1
Example 3.13 : Differentiate the following (a) y = cosh x (b) y = sinh x (c) y = tanh x

Solution:
1
(a) Let y = cosh x. This means that x = cosh y
dy
Differentiate implicitly to have 1 = sinh y dx
ey +e y d 1 d
Since cosh y = 2 and dx (cosh y ) = 2 dx ( e
y +e y) = 12 (ey e y) = sinh y
dy
Therefore dx = 1
sinh y . But cosh2 y sinh2 y = 1
q p dy
) sinh2 y = cosh2 y 1 and sinh y = cosh2 y 1= x2 1, Therefore dx = p 1 .
x2 1
1
(b) Given that y = sinh x. Then x = sinh y
dy
Differentiating implicitly, we have 1 = cosh y dx
1 dy
This implies that dx = cosh y
q p
But cosh y = 1 + sin2 y = 1 + x2 , since sinh2 y = x2 .
1
(c) Let y = tanh x, 1 < x < 1.

53
dy
Then x = tanh y,and 1 = sec h2 y dx .
dy 1 1 1
Thus = sec h2 y
= = , x2 < 1.
dx 1 tanh2 y 1 x2

Generally, if u is a differentiable function of x the derivative of the inverse hyperbolic func-


tions can be obtained as follows:
1 du dy
(i) Let y = sinh u , then u = sinh y. Using implicit differentiation dx = cosh y dx .
dy 1 du 1 du du
Hence, = =p = p 1 .
dx cosh y dx 1+sinh2 y dx 1+u2 dx

This gives the formula


d 1 p 1 du
dx (sinh u) =
1+u2 dx
.
1
(ii) Similarly, let y = cosh u , then u = cosh y. Now using implicit differentiation, we have
du dy
dx = sinh y dx .
dy 1 du 1 du du
Hence, = =p = p 1 .
dx sinh y dx cosh2 y 1 dx u2 1 dx

This gives the formula


d 1 p 1 du
dx (cosh u) =
u2 1 dx
, u > 1.
1 du dy
(iii) Let y = tanh u then u = tanh y, and dx = sec h2 y dx .
dy 1 du 1 du 1 du
Rearrange to have = sec h2 y dx
= = .
dx 1 tanh2 y dx 1 u2 dx

This gives the formula


d 1 1 du
dx (tanh u) = 1 u2 dx
, j u j< 1.
1
(iv) Let y = coth u, then u = coth y. This means that
dy dy
du
dx = csch2 y dx = 1 coth2 (y) dx ; Since coth2 y cosech2 y = 1, we have
dy 1 du
dx = 1 u2 dx
. This gives the formula
d 1 1 du
dx (coth u) = 1 u2 dx
, j u j> 1
dy
v) Let y = sech 1u then u = sechy. Differentiate implicitly to have du
dx = sechy tanh y dx . Rearran
to have
dy 1 du du du
dx = sec hy tanh y dx = p1 dx = p1 . This gives the formula
sec hy 1 sec h2 u u 1 u2 dx

d 1 p1 du
dx (sec h u ) =
u 1 u2 dx
, 0 < u < 1.
Finally, using similar arguments, it can be proved that

54
d 1 p1 du
(vi) dx (csc h u ) =
juj 1+u2 dx
, u 6= 1.

Example 3.14 : Find the derivative of the following p


inverse hyperbolic functions with respect to x.(i)
1 1 3
2
cosh (2x + 3) (ii) sinh ( x ) (iii) sinh ( x2 1) (iv) tanh 1 ( 3x2x
1
+ x2
).(v)sech 1 ( x3 1).

d 1 p 1 du d 1 2 1 d
Solutions: dx (cosh u) = we have that dx (cosh (2x + 3)) = p (2x2 +
u2 1 dx (2x2 +3)2 1 dx
4x x
3) =p = 2q .
(2x2 +3)2 1 ( x4 +3x2 +2)
1 d 1 p 1 du
(ii) Let y = sinh ( x3 ), and u = x3 . Now from the relation dx (sinh u) = , we have
1+u2 dx
d 1 3 1 d 3x2 x2
dx (sinh ( x )) =p ( x3 ) =p = 3p .
1+( x3 )2 dx 1+( x3 )2 (1+ x 6 )
1
p p
(iii) Let y = sinh ( x2 1), and u = x2 1, then
p p 1 2
1
d 1 1 d 2 (x 1) 2 (2x ) 1
dx (sinh ( x2 1)) = p p dx ( x2 1) = p p =p .
1+( x2 1) 2 1+( x2 1)2 ( x 2 1)
1
(iv) Let y = tanh ( 3x2x
+ x2
), and u = 2x
3x + x2
, then from formula
d 1 1 du d 1 2x 1 d 2x
dx (tanh u) = 1 u2 dx
, we have dx (tanh ( 3x + x2 )) = 1 ( 2x
)2 dx ( 3x + x2 ).
3x + x2

d d
1 (3x + x2 ) dx (2x ) 2x dx (3x + x2 ) 1 2(3x + x2 ) 2x (3+2x )
= 1 ( 2x
)2 2 = 1 ( 2x
)2 2 =
3x + x2
(3x + x2 ) 3x + x2
(3x + x2 )
2
5+6x + x2
.
1 ( x3 d p1 du
(v) Let y = sech 1), and u = x3 1.Then from the formula dx (sech 1 u) = , we
u 1 u2 dx
have
d d 2
1 3 p1 3 p3x qx
dx (sech ( x 1)) =
( x3 1) ( x3 1)2 dx ( x 1) =
( x3 1) 1 ( x 3 1)2
= 3 .
1 ( x 3 1) ( x4 +2x )

Exercise 3.7:Explore the problems below: (a) Differentiate the following hyperbolic functions
with respect to x.
p
(i) cosh(3x ) (ii) cosh (e x ) (iii) e x sinh x2 + 3x (iv) tanh 3x3 1 (v) coth x3 x2 1 (vi) cosech
x2
sec h (3x2 1)2
du dv
(b) If u = e x cosh x and v = e x sinh x,show that dx = dx and give a relation between u and
v that is independent of x.
(c) Find the derivative of each of the following functions. (i) sinh 1 ( x4 2) (ii) sinh 1 (e 3x ) (iii)
sinh 1 (cosh( x4 2))
p p p
(iv) x cosh 1 x + 1 + x2 (v) x2 ln x2 1 x cosh 1 x (vii) sech 1 1 x2 (viii) x2 coth 1
x3
1
esinh x

55
3.12 Applications of Differentiation

3.12.1 Limit of Indeterminate Forms


f (x) f (x)
Definition 3.4 : If as x ! x0 , g( x )
takes the form 00 , we say g( x )
has an indeterminate form. This is
form ∞
f (x)
also the case when g( x) takes the ∞ as x ! x0 . The
limit of some functions with indeterminate form
can be evaluated using L’hospital’s rule.The rule states that if the functions f and g have derivatives on
an open interval ( x1 , x2 ) containing x0 , and g0 ( x0 ) 6= 0 and if g( x) has an indeterminate form 00 or ∞
f (x)
∞ as
x ! x0 , then

f (x) f 0 (x)
lim = lim 0
x ! x0 g ( x ) x ! x0 g ( x )

provided that
f 0 (x)
lim
x ! x0 g0 ( x )
f 0 (x) 0 ∞
exists. If the limit of g0 ( x )
is still of the form 0 or ∞, we differentiate f 0 and g0 again and evaluate
00
f (x)
the limit of the quotient g00 ( x) at x = x0 .

0 ∞ f (x)
This process is repeated until we no longer have expression of the form 0 or ∞ .But if lim =
x ! x0 g ( x )
f 0 (x) f 00 ( x ) f (x)
lim 0 = limx! x0 g00 ( x )
= ... = ∞, then lim does not exist.
x ! x0 g ( x ) x ! x0 g ( x )

Example 3.15 : (i) Let f : R+ f4g ! R be defined by f ( x ) = px 4 ,


x 2
find the limit of the function f
as x approaches 4.

x2 4
(ii) Let f : R f2g ! R be defined by f ( x ) = x +2 , find the limit of f ( x ) as x ! 2.
e x +e x
(iii) h : R f0g ! R be defined by h( x ) = 2x , find the limit of h at x = 0.
Solution: (i) lim f ( x ) = lim px 4 p4 4 = 0
= = 0
(indeterminate). Therefore lim f ( x ) =
x !4 x !4 x 2 4 2 2 2 0 x !4
p
2 x
lim px 4 = limx!4 1
1 = limx!4 ( 1 ) = 4.
x !4 x 2 1
2x
2

We can obtain the same result by rationalising the denominator to get:


p p
( x +2 ) ( x 4) p
lim f ( x ) = lim px 4 = lim ( pxx 42 . p xx+
+2
2
) = lim ( x 4 ) = lim x + 2 = 4.
x !4 x !4 x 2 x !4 x !4 x !4
x2 4 x2 4
(ii) limx! 2 x +2 = limx! 2 = 00 (indeterminate). Thus, limx! 2 x +2 = limx! 2x
2 1 = 2( 2) =
4.

56
We can evaluate this limit by factorising the numerator and then cancel the common factor for
x 6= 2 to get
x2 4
lim = lim ( x 2) = 2 2 = 4,
x! 2 x + 2 x! 2

x x x x x +e x x x
(iii) lim e e
2x = 00 (Indeterminate). Thus, lim e e
2x = lim e 2x = lim e 2
e
= 0
2 = 0.
x !0 x !0 x !0 x !0
2x
p
(iii) lim 1 = lim 2. ( x2 + 5) = 6.
x !2 x ( x 2 +5) 2 x !2

sin x ln x ex 1
Exercise 3.8: Evaluate the limits below if they exist (i) limπ 1 cos x (ii) lim x3 1 (iii) lim .
x! 2 x !1 x !∞ 3x 1

ex 3x
Example 3.16 : Evaluate the following limits if they exists: (i) lim ln x (ii) lim xe sin1x (iii) lim cot 2x
cot 3x .
x !∞ x !∞ x !0

Solution:
ex x
(i) lim e
= lim 1/x = lim xe x = ∞
x !∞ ln x x !∞ x !∞
e3x 1 3x 3x 27e3x
(ii) lim x sin x = lim 1 3ecos x = lim sin
9e
x = lim = ∞.
x !∞ x !∞ x !∞ x !∞ cos x

Hence, the limit does not exist.


cot 2x cos 2x sin 3x 2 sin 2x. sin 3x + cos 2x.3 cos 3x 3
(iii)lim cot
x !0 3x = lim sin 2x . cos 3x = lim 2 cos 2x. cos 3x
x !0 3 sin 2x.3 sin 3x =
x !0 2

Other Indeterminate Forms


f (x)
Suppose lim has the form 0.∞ or ∞.0, we say its form is also indeterminate. This form can
x ! x0 ( x )
g
0 ∞
be changed to the indeterminate form 0 or ∞ by writing
f (x) g( x )
f ( x ) g( x ) = 1/g( x )
or 1/ f ( x )
whichever is applicable before L’Hospital’s rule is applied.

Example 3.17 : (i) limx!0 x cot x (ii) limx!∞ ( x2 1) e x2 (iii) limx! π2 ( x π


2 ) tan x.

Solution:
x
(i) limx!0 x cot x = limx!0 tan x = limx!0 sec1 x = 1
x2 ( x 2 1) 2x 1
(ii) limx!∞ ( x2 1) e = limx!∞ 2 = limx!∞ 2 = limx!∞ 2 =0
ex 2xe x ex
( x π2 ) 1 1
(iii) limx! π2 ( x π
2 ) tan x = limx! π2 cot x = limx! π2 cos ec2 x
= limx! π2 1/ sin2 x
= limx! π2 sin2 x =
1

sin x ln x ex 1
Exercise 3.9: Evaluate the limits below if they exist i) limπ 1 cos x ii) lim x3 1 iii) lim .
x! 2 x !1 x !∞ 3x 1

57
3.12.2 Critical Point

The value x = c for which the derivative f / (c) = 0,is called the critical (or turning) point of f .
Geometrically this means that the tangent to the function f at (c, f (c)) is horizontal. The critical
point of a function f ( x )can therefore be found by solving for x,the equation f 0 ( x ) = 0.

Example 3.18 : Find the critical points of the functions below:(i) f ( x ) = x3 3x2 24x + 8; (ii)
y = 1 x12 .

Solution:
(i) f 0 ( x ) = 3x2 6x 24, f 0 ( x ) = 0 ) 3( x2 2x 8) = 3( x 4)( x + 2) = 0 =) x = 4,or
x = 2;
2
(ii) y0 = x3
,y 6= 0 for any value of x. So f has no critical point.
A function f ( x ) is said to have a local maximum f ( p) at the point p if f ( x ) f ( p) for all x near
p. This means the inequality holds in some interval containing the point p.
Similarly, f ( x ) has a local minimum f (q) at the point q if f (q) f ( x ) for all x near q. A local
maximum or minimum can be but it may not always a critical point.
A function f ( x ) is said to have an absolute maximum at r if f ( x ) f (r ) for all x in the domain
of f ( x ). Similarly, f ( x ) has an absolute minimum at s if f ( x ) f (s) for all x in the domain of f .
(i) The function y = x2 if ( 4 < x < 4) has no local maximum. There is no interval containing p
such that f ( x ) f ( p). The function however has a local minimum of 0 at x = 0. f (0) f ( x ) in
every interval containing 0.
(ii) However, the function y = x2 if ( 1 x 2) has local maximum of 1, taken at the point
x = 1 and 4 taken at x = 2, and an absolute maximum of 4, taken at x = 2. It has an absolute
minimum value of 0 taken at x = 0.
(iii)The function y = x2 if ( 1 x 1) has an absolute maximum at both 1 and 1.
If f ( x ) has a local maximum or a local minimum at p, then p need not be a critical point. Con-
versely, if p is a critical point, f ( p) need not be a local maximum or local minimum .
If, on the other hand, f ( x ) is differentiable at p and has a local maximum or minimum at p, then,
p is a critical point of f .

3.12.3 First Derivative Test

Suppose f 0 ( a) = 0and x is near a, then (i) If f 0 ( x ) 0for all x a and f 0 ( x ) 0for all x a, f
is a local minimum at a. The gradient of the curve goes from a negative value through zero to a
positive value. (ii) If f 0 ( x ) 0for all x a and f 0 ( x ) 0for all x a, f is a local maximum at a.
The gradient of the curve goes from a positive value through zero to a negative value.

58
If neither of these cases applies, then the critical point is neither a local maximum nor a local
minimum point.

Example 3.19 : Sketch the graph ofy = x2 2x 1 using the first derivative test.

dy
Since dx = 2x 2, we see that x = 1 is a critical point. Also 2x 2 0 if x 1and 2x 2 0 if
x 1.So the gradient of the curve goes from a negative value through zero to a positive value..
The function has a minimum y = 2 at x = 1..

Example 3.20 : Sketch the graph of f ( x ) = x3 3x + 1.

59
Note that f 0 ( x ) = 3x2 3 = 3( x 2 1).
Hence, x = 1 and x = 1 are critical points. For x 2 ( 1, 1), we have x2 < 1, so f 0 ( x ) =
3( x2 1) < 0. f 0 ( x ) = 3( x2 1) > 0 for x 2 ( ∞, 1) or x 2 (1, ∞). So f increases on these
intervals. Hence f (1) = 1 is a local minimum and f ( 1) = 3 is a local maximum. The graph
appears as shown above.

3.12.4 Maximum and Minimum Problems

The problem often takes the form of a word problem to find the maximum and minimum quan-
tity Q. It is helpful to draw and label its pertinent features. With the aid of the figure if possible,
find an equation giving quantity Q in terms of other variables. Use relations among the vari-
ables to obtain Q in terms of a single variable x : Q = Q( x ). Find the derivative,Q0 ( x ) . Set the
derivative equal to zero, i.e. Q0 ( x ) = 0 and solve to find critical points. The desired maximum
or minimum usually occurs at a critical point. Select the proper critical point and apply the test.
We have determined the value of x that either maximizes or minimizes Q. The desired value of
Q is obtained simply by finding Q = Q( x ) for this value of x.

Example 3.21 : Show that the square has the greatest area amongst all rectangles of a given perimeter.

60
Let P = the given perimeter, (a constant), x = width, and y = the length of the rectangle. Then the
area A = xy. Since 2x + 2y = P, y = P 22x ,so A( x ) = x P 22x = Px 2 x2 .
p
(Note that A( x ) only makes physical sense for 0 x P). We have,A0 ( x ) = 2 2x, and set it
p
equal to zero: 0 = 2 2x. Solving, we find x = P4 , y = P4 and the rectangle must be a square. It
is clear from the problem that the maximum area occurs at x = P4 . To show this mathematically,
p p
we note that if A0 ( x ) = 12 p 2x 0 if x 0 1
4 , A (x) = 2 p 2x 0 if x 4
p
Hence, A( x ) increases on[0, 4 ] and decreases on the the other set, which proves that the maxi-
p
mum occurs at x = 4 .

Example 3.22 : A man, 100 m away from the base of a flag pole, starts walking towards the base at 10
m/sec just as a flag at the top of the pole is lowered at the rate of 5 m/sec. If the pole is 7 cm tall, find how
the distance between the man and the flag is changing per unit of time at the end of 2 sec.

Call x the distance the man is from the base, y the height of the flag, and z the distance between
man and flag at any time t .
Then we are given
dx dy
dt = 10cm/ sec, dt = 5cm/ sec
(The minus sign is present because x and y are decreasing.) Moreover, x = 10t + 100 and
y = 5t + 70. Now, always we have z2 = x2 + y2 and differentiating this equation with respect
to time t gives
p
2z dz = 2x dx + 2y dy .We find that x = 80m, y = 60m then t = 2; and then z = x2 + y2 =
p dt dt dt
802 + 602 = 100m.
Hence, 100 dzdt = 80( 10) + 60( 5) = 1100. ) dz
dt = 11m/ sec .The minus sign says that the
distance t is decreasing at t = 2.
Exercise 3.10:We explore the problems below.
1. Find the critical points of
(i) f ( x ) = x3 3x + 7. (ii) y = 2x3 + 6x2 12x + 4.
2. Draw the graphs of the following functions:
1 x
i) f (x) = x 2 ii) f (x) = x2 4

3. Show that f ( x ) = x3 + x + 1 has no critical points and is always increasing.


4. A farmer has 100m of fence and wishes to enclose a rectangular plot of land. The land
borders a river and no fence is required on that side. What should the dimensions of the rectangle
be in order that it includes the largest possible area?

61
3.12.5 Second Derivative Test

Suppose that a function f has a derivative on an open interval ( x1 , x2 ) containing p, and that
f 0 ( p) = 0. If f 00 ( p) < 0, then f ( x ) has a local maximum at the point ( p, f ( p)).
If f 00 ( p) > 0, then f ( x ) has a local minimum at the point ( p, f ( p)). The notion of concavity of
the graph of f can be used to illustrate conditions above as follows.
If f 0 ( p) = 0, the tangent line to the graph at ( p, f ( p)) is horizontal. In addition if f 00 ( p) < 0,
then the graph is concave down at p and hence there is an open interval ( x1 , x2 ) containing p
such that the graph lies below the tangent line at p. It then follows that f ( p) is a relative (local)
maximum. If f 0 ( p) = 0, the tangent line to the graph at ( p. f ( p)) is also horizontal. If in addition
, the graph is concave up at p and hence there is an open interval ( x1 , x2 ) containing p such that
the graph lies above the tangent at p. Then f ( p) is a relative (local) minimum.
If f 0 ( p) = 0, the second derivative test is not employed in that case, the first derivative test is
employed x = p is called a flex and ( p. f ( p)) point of inflexion.

Example 3.23 : Use concavity and second derivative test to examine the following curves for minima,
maxima, flex point.

(i). x5 2x3 + x 12.( ii). x ln x.


(i) f ( x ) = x5 2x3 + x 12 =) f 0 ( p) = 5x4 6x2 + 1 = (5x2 1)( x2 1)=) f 0 ( p) =
(5p2 1)( p2 1) = 0
p= p1 ; p = 1 or p = 1.
5

. f 00 ( x ) = 20x3 12x.For p = 1, f 00 (1) = 8 which is positive.


This implies that (1, f (1)) is a local minimum and the graph is concave up at p = 1.For p = 1,
f 00 ( 1) = 8 which is negative.
Therefore, f ( 1) is a local maximum and the graph is concave down at ( 1, 12).
For p = p1 = 0.45,, f 00 ( 0.45) = 3.58 which is positive.
5

Therefore, f ( 0.45) is a local minimum and the graph is concave upward at ( 0.45, 12.46).
For p = p1 = 0.45, f 00 (0.45) = 3.58which is negative.
5

Therefore f (0.45) is a local maximum and the graph is concave downward at (0.45, 11.54).
For flex point, f 00 ( p) = 0 and this implies that 20p2 12p = 0 ) 4p[5p2 3] = 0.So either p = 0
or p = 3/5 = 0.77
Therefore, the points of inflection are
r !
q q q q q
3 3 3 3 3 3
(0, f (0)) , 5, f ( 5 ) and 5, f ( 5 ) or (0, 12) , 5, 12.12 and 5, 12.5

62
(ii) f ( x ) = x ln x
1 1
f 0 ( x ) = ln x + 1 f 0 ( p) = ln p + 1 = 0 implies that: ln p = 1 and p = e 1 . f 00 ( p ) = p = e 1 =

e.Hence f 00 (e 1) = e, is positive.
Therefore, f (e 1) is a local minimum, and the graph is concave up at p = e 1.

For flex point, f 00 ( p) = 0implies that 1


p = 0 ) 1 = 0 which is a contradiction, and so p = ∞.
Hence there is no flex point.

Example 3.24 : Use the concavity and second derivative test to examine the following curves for mini-
mum, maximum and flex point.

i) 12 + 2x2 x4 , ii) x5 5x3 , iii) 1 x1/3


Solution:
(i) f ( x ) = 12 + 2x2 x4 =) f 0 ( x ) = 4x 4x3 = 4x [1 x2 ] =) f 0 ( p) = 4p[1 p2 ] = 0 =)

p = 0 or p = 1.
f 00 ( p) = 4 12p2 .For p = 0, f 00 (0) = 4,is positive. Therefore, f (0) is a local minimum and the
graph is concave upward at (0, 12).
For p = 1, f 00 (1) = 8is negative. Therefore, f (1) is a relative (local) maximum, and the graph
is concave down at (1, 13).
For p = 1, f 00 (1) = 8 is negative. Therefore, f ( 1) is a local maximum, and the graph is
concave down at ( 1, 13).
1 p1
For the flex point f 0 ( p) = 0 and this implies that 4 12p2 = 0, p2 = 3 )p= = 0.58
3

Therefore, the points of inflection are ( 0.58, 12.56) and (0.58, 12.56).
(ii) 5x3 =) f 0 ( x ) = 5x4
f ( x ) = x5 15x2 = 5x2 [ x2 3] =) f 0 ( p) = 5p2 [ p2 3] = 0
p
) p = 0 and p = 3 = 1.73
f 00 ( p) = 20p3 30p.For p = 0, f 00 (0) = 0.Therefore the second derivative test is not ap-
plicable.
For p = 1.73
f 00 (1.73) = 46.36 is positive. Therefore, f (1.73) is a local minimum and the graph is concave
up at (1.73, 10.37)
For p = 1.73, f 00 ( 1.73) = 51.65 for positive.
Therefore, f ( 1.73) is a local maximum, and the graph is concave down at ( 1.73, 10.37) .
For the flex point f 00 ( p ) = 0.

63
q
3
Therefore 20p3 30p = 0. ) 10p[2p2 3] = 0 ) p = 0 or p = 2 = 1.22.So the points of

inflection are (0, 0), ( 1.22, 6.38)and(1.22, 6.38).


Exercise 3.11 Explore the problems below.
(i) x4 6x2 + 18x + 10, (ii) ( x + 5)4 (2x 3)3 , (iii) x3 + 2x2 + x + 1.

Definition 3.5 : :Suppose the function f is defined at p and for all values of x near p. Then the function
is said to be continuous at p provided that lim f ( x ) = f ( p) .
x! p

Example 3.25 : : Let us define f ( x ) = sinx x if x 6= 0. This definition of f ( x ) has no meaning if


x = 0, since division by 0 is undefined. However, let us make the additional definition f (0) = 1. With
this definition f ( x ) is continuous at x = 0.

Forlim sinx x = 1.
x !0

We have based the concept of continuity directly upon the concept of a limit. A condition for
continuity of a function may be given directly in terms of inequalities, just as we defined a limit
in terms of inequalities.

Definition 3.6 : Thus, if f is defined throughout some interval containing p and all points near p, f
is continuous at p if to each positive ε > 0 corresponds some positive δ > 0 such that j f ( x ) f (b) j
< ε whenever j x p j < δ. This form of the condition for continuity is equivalent to the original

definition.

Theorem: Let f be a continuous function on a closed interval [ x1 , x2 ], and differentiable on an

open interval ( x1 , x2 ), and f ( x1 ) = f ( x2 ). Then there exists at least one number p in ( x1 , x2 ) such
that f 0 ( p) = 0. This is called Rolle’s theorem.
(i) f ( x ) = f ( x1 ), (ii) f ( x ) < f ( x1 ) and (iii) f ( x ) > f ( x1 )
Now if (i) f ( x ) = f ( x1 ), it implies that f ( x ) is a constant function and hence f 0 (x) = 0, for
every p and for every point p in ( x1 , x2 ), f 0 ( p) = 0.
(ii) f ( x ) < f ( x1 ) for some x in ( x1 , x2 ), this implies that the minimum value of f ( x ) on [ x1 , x2 ] is
less than f ( x1 ) and therefore this must occur at some number p in ( x1 , x2 ). Since the function is
continuous, the derivative exists throughout ( x1 , x2 ), this implies that f 0 ( p) = 0.
(iii) If f ( x ) > f ( x1 ), the argument is similar to (ii).

Example 3.26 : Show that the curve 3x2 12x + 11 satisfies the hypothesis of Rolle’s theorem on the
interval [0, 4] and find all number(s) p in (0, 4) such that f 0 ( p) = 0.

64
Since f ( x ) = 3x2 12x + 11 is a polynomial it is continuous and differentiable on every interval,
therefore f ( x ) is continuous on [0, 4] and differentiable on (0, 4) and f (0) = f (4) = 11. Also,
f 0 ( x ) = 6x 12 =) f 0 ( p) = 6p 12 = 0 =)) p = 2 =) f 0 (2) = 0 and 0 < 2 < 4.
Hence, f ( x ) satisfies the hypothesis of Rolle’s theorem on [0, 4].

Example 3.27 : Verify that the function x4 + 4x2 + 1 satisfies the hypothesis of Rolle’s theorem on the
interval [ 3, 3] and find all number(s) p in ( 3, 3) such that f 0 ( p) = 0.

f ( x ) being a polynomial, it is continuous on [ 3, 3], and differentiable on ( 3, 3). And f ( 3) =


f (3) = 118.
p
f 0 ( x ) = 4x3 + 8x ) f 0 ( p) = 4p3 + 8p = 4p( p2 + 2) = 0 ) p = 0 and ( p = i 2) which
you can neglect.
So f 0 (0) = 0 and 3 < 0 < 3.Hence f ( x ) satisfies the hypothesis of Rolle’s theorem.
Exercise 3.12: Determine whether the following functions satisfy the hypothesis of Rolle’s theo-
rem on the interval [ x1 , x2 ]. If so, find all number(s) p in ( x1 , x2 ) for which f 0( x ) = 0.
i) f ( x ) = 4x2 20x + 29,[1, 4],(ii) f ( x ) = 5 12x 2x2 , [ 7, 1]; (iii) f ( x ) = x3 x,
[ 1, 1].
Theorem: (Lagrange’s Mean Value Theorem):Let f be a function which is continuous at each
point of the closed interval a x b, and let it have a derivative at each point of the open
interval a < x < b. Then there is a point x = p in the open interval ( a < p < b) such that
f (b) f ( a) = (b a) f 0 ( p) ...(1)
The theorem has a geometrical interpretation. Represent the function graphically by the curve
y = f ( x ), and let A, B be points on the curve corresponding to x = a, x = b, respectively. The
formula (1) states that there is some point on the curve, with the abscissa x = p, at which the
tangent line is parallel to the line AB. There may be more than one suitable value of p; the
essential thing is that there is always at least one.

Remark 3.1 : It is worth noting that (1) remains true if we exchange a and b, for both sides merely change
signs when this is done. Thus, suppose x1 , x2 are the endpoints of an interval on which the conditions of
the mean value theorem are satisfied, then we can write

f ( x2 ) f ( x1 ) = ( x2 x1 ) f 0 (ξ ) where x = ξ is some point between x1 and x2 . In writing this


formula we do not need to know which of the numbers x1 , x2 is larger.

Example 3.28 : Suppose f ( x ) = x3 , find a suitable value for p in formula (1) when a = 1 and b = 2.

b3 a3 = (b a)3p2 in the present case. With a = 1, b = 2 we have 8 ( 1)3 = (2


( 1)).3p2 = (2 + 1)3p2 =) 9 = 9p2 or p2 = 1.
Solving, we find p = 1. We want a value of p such that a < p < b, i.e. 1 < p < 2. Hence,
p = 1 is the suitable value in question.

65
a+b
Example 3.29 : If f ( x ) = x2 , show that the suitable value of p in the mean value theorem is p = 2 .

a+b
b2 a2 = ( b a).2p, or p = 2 . Where is this point located in relation to a and b?


Example 3.30 : If f ( x ) = sin x and x1 = 0, x2 = 6 , find ξ such that x1 < ξ < x2 and formula (2)
holds.

sin 5π
6 sin 0 = 5π
6 cos ξ =) or 1
2 = 5π
6 cos ξ =) cos ξ = 3
5π = 0.19099.
5π 1 (0.1909)
since 0 < ξ < 6 , we find ξ = cos = 1.37863.

Example 3.31 : Show that 0 < log 1.5 < 1

1
We take b = 1.5,a = 1.0 and f ( x ) = ln x so that f 0 ( x ) = x .Then by the mean value theorem,
(1.5 1) 0.5
log 1.5 log 1 = p = p ,where 1 < p < 1.5.
0.5 0.5 0.5
It follows that 1.5 < p < 1 .This gives the desired result.

log x log a
Example 3.32 : Use the mean value theorem to show that x < x + 1a if 0 < a < x.

log x log a x a x a 1 1
Suppose a < p < x then x = x + p ,Now, x . p < p < 1a .Hence the desired result.

Exercise 3.13:
1. Compute the point p in (0, 3) for which Rolle’s theorem is satisfied, where f ( x ) = 1 6x + 2x2 .
2. Find the value(s) of p for which the function f ( x ) = cos 3x
2 , [0, π ] satisfy the Lagrange mean
value theorem.
1

1 Provided by Departments of Mathematics, UNIJOS and PLASU - 2013

66
Chapter 4

Integration and the Indefinite Integral

4.1 Objectives

In this module we introduce with examples, the concept of integration as an anti-derivative.


This is followed by worked examples and exercises to facilitate consolidation of appreciation of
the concept. Various methods of evaluating integrals are treated with relevant worked examples
for their easy understanding. Group assignments, take home tests and laboratory sessions will
complement power point presentations in tutorial type exchanges. All these activities expose the
students to the concepts and techniques and lead them to find out when the different techniques
are most appropriate for use.

4.2 Learning Outcomes:

At the end of this module students should be able to decide when the different rules best apply
and to:
evaluate integrals using the antiderivative property;
evaluate integrals using substitution, integration by parts, partial fractions and reduction meth-
ods.

4.3 Learning Activities:

Students should:
explore notes and exercises, individually and in groups;
explore and use related materials on the Intranet, especially the e-granary and MIT open course
ware; explore and use related materials on the OLI Calculus course on the Internet(http://www.cmu

67
solve relevant questions from past MTH103 Examinations.

4.4 Introduction

Integration can be considered from two perspectives:


(i) to find a function whose derivative is given . In this sense, integration is referred to as finding
the anti-derivative.
(ii) to give the sum or total . In this sense integration measures the area bounded by curves.
We introduce and study the indefinite integral from the anti derivative perspective. The definite
integral is introduced in the next module as area under a curve.

4.4.1 The Indefinite Integral


dy
Suppose we have the derivative dx of a function y = F ( x ) with
dy
dx = f ( x ); a < x < b and we are asked to find y = F ( x ). We illustrate how to do so following
the process below.
dy
Let dx= f ( x ) = 2x; from our knowledge of derivatives, we can find one answer, y = F ( x ) = x2 .
dy p
This is so because if y = x2 , dx = 2x. But this is not the only answer; y = x2 + 2, y = x2 3, y =
x2 + 5π are all valid answers.

Hence y = x2 + C is an answer where C is an arbitrary constant.


Thus if we differentiate a function F ( x ) with respect to x and obtain another function f ( x ), that
d
is dx F ( x ) = f ( x ), we say that F ( x ) is an integral of f ( x ) with respect to x. If F ( x ) is an integral
of f ( x ) with respect to x, then F ( x ) + C is also such an integral where C is an arbitrary constant.
This is true since dx d
[ F ( x ) + C ] = dFdx(x) + dC
dx = f ( x ) + 0 = f ( x ).
d
R R
Thus if dx [RF ( x ) + C ] = f ( x ), then f ( x )dx = F ( x ) + C,where the symbol is called an “integral
sign” and f ( x )dx is called “the integral of f ( x ) with respect to x.
We state some standard formulae for integrals using standard formulae for derivatives.
d( x ) R
dx = 1 =) 1dx = x + c.
d( xn ) R n +1
dx = nx n =) x n dx = xn+1 + c, n 6= 1
1

d(ln x ) 1
R 1
dx = x =) x dx = ln x + c.
d(sin x ) R
dx = cos x =) cos xdx = sin x + c.
d(cos x ) R
dx = sin x =) sin xdx = cos x + c.

68
d(tan x ) R
dx = sec2 x =) sec2 xdx = tan x + c.
d(sin 1 x ) 1
R
dx =p =) p 1 2 dx = sin 1
x + c.
(1 x 2 ) (1 x )
d(e x ) R
dx = e x =) e x dx = e x + c.
d(cosh x ) R
dx = sinh x =) sinh xdx = cosh x + c.

d(tan 1 x ) 1
R 1 1
dx = 1+ x 2
=) 1+ x 2
dx = tan x + c.
We use the combination rules ( sum and difference ) for the derivatives to get similar rules for
integrals and illustrate them below.

Example 4.1 : Evaluate the following integrals:

R
(i) (5x x2 + 2)dx.
R 1
(ii) x 2 dx.
R
(iii) (3x + 2e x + cos x )dx.
R
(iv) (3e x + 4 sinh x )dx.
Solutions:
R R R R 5x2 x3
(i) (5x x2 + 2)dx = 5xdx x2 dx + 2dx = 2 3 + 2x + c.
R 1
1
x 2 +1 x2
3
3
(ii) x 2 dx = 1 + C = 23 x 2 + C.
+C = 3
2 +1 2
R R R R R R R
(iii) (3x + 2e x + cos x )dx = 3xdx + 2e x dx + cos xdx = 3 xdx + 2 e x dx + cos dx =
3x2 x
2 + 2e + sin x + C.
R R R
(iv) (3e x + 4 sinh x )dx = 3 e x dx + 4 sinh xdx = 3e x + 4 cosh x + C
Exercise 4.1: Evaluate the following integrals.
R
(1) (2x + 3) dx.
R 2 p
(2) x x dx.
R
(3) (5e x + sin x ) dx.
R 1
(4) 3 cosh x + x 2 dx.
R
(5) x4 e x + cos x + sinh x dx.

69
4.5 Methods of Integration

The integrals evaluated so far have been quite direct and simple. We could easily figure out the
function differentiated to obtain the integral. For more complicated functions we need to intro-
duce rules and guidelines for easy operations. These are the various methods for the evaluation
of integrals.

4.5.1 Integration by substitution or change of variable

The method of substitution is used to evaluate integrals of composite functions. It is derived


from the results below, which are generalizations of the list of integrals given earlier. The method
of substitution is the converse of the chain rule (i.e. differentiating a function of a function). The
list below is the standard formulae for the derivatives of some composite functions and there
integrals.
d( ax +b)n R ( ax +b)n+1
dx = na( ax + b)n 1 =) ( ax + b)n dx = ( n +1) a
+ c; n 6= 1.
d(sin ax ) R 1
dx = a cos ax =) cos axdx = a sin ax + c.
d(tan ax ) R
dx = a sec2 x =) sec2 axdx = 1a tan ax + c.
d(sin 1 xa ) R
dx =p 1
=) p 21 2 dx = sin 1 xa + c.
( a2 x 2 ) (a x )

d(e ax ) R
dx = ae ax =) e ax dx = 1a e ax + c.
d(cosh ax ) R 1
dx = a sinh ax =) sinh axdx = a cosh ax + c.

d(tan 1 xa ) a
R
dx = =) 1+1x2 dx = 1a tan 1 xa + c.
a2 + x 2
R dI dI dI dx
Let us consider I = ϕ( x )dx, where x = x (t).By definition of I, dx = ϕ( x ) and dt = dx . dt =
dx
ϕ( x ) dt .
R R
Thus ϕ( x )dx = ϕ( x ) dx dt dt. This is the substitution method.
R R
More explicitly it can be written as ϕ( x [t])dx = ϕ( x [t]) dx
dt dt.
We now illustrate the use of these generalizations and the related method of substitution by
solving some examples.

Example 4.2 : Evaluate the following integrals.

Rp
(i) (2x + 1)dx.
R
(ii) (3x 1)234 dx.

70
R
(iii) p3rdr 2 .
(1 r )
R
(iv) sin2 x cos xdx.
R
(v) e5x dx.
R
(vi) tan 3xdx.
Solutions:
Rp 1 2(2x +1) 2
3
3
(i) ((2x ) + 1)dx = 2 3 + C = 13 (2x + 1) 2 + C directly.
1
Alternatively, let ϕ( x ) = (2x + 1) 2 with 2x + 1 = t and 2 dx dt = 1.
R R R 1 1 3 3
ϕ( x )dx = ϕ( x ) dx
dt dt = t 2 . 2 dt = 13 t 2 + C = 13 (2x + 1) 2 + C.
R (3x 1)235
(ii) (3x 1)234 dx = 13 235 + C directly.
234
Alternatively, let ϕ( x ) = (3x 1) with 3x 1 = t and 3 dx dt = 1.
R R R 234 1 (3x 1)235
ϕ( x )dx = ϕ( x ) dx
dt dt = t . 3 dt = 13 . 235
1 235
t + C = 13 . 235 + C
R p
(iii) p3rdr 2 = 3 (1 r2 ) + C.
(1 r )
R sin3 x
(iv) sin2 x cos xdx = 3 + C directly.
Alternatively let ϕ( x ) = sin2 x cos x with sin x = t and cos x dx
dt = 1.
R R R R R
ϕ( x )dx = ϕ( x ) dx
dt dt = sin2 x cos x cos1 x dt = sin2 xdt = t2 dt
= 13 t3 + C = 13 sin3 x + C.
R
(v) e5x dx = 15 e5x + C.
sin 3x
(vi) tan 3x = cos 3x .
sin 3x dx
Let ϕ( x ) = cos 3x with sin 3x = t and 3 cos 3x dt = 1.
R R R sin 3x 1 R sin 3x R
ϕ( x )dx = ϕ( x ) dxdt dt = cos 3x 3 cos 3x dt = 1
3 1 sin2 3x
dt = 1
3
t
1 t2
dt

= 13 ln(1 t2 ). 2
1
+C = 6
1
ln cos2 3x + C = 3
1
ln jcos 3x j + C.
Exercise 4.2:Explore the evaluation of the following integrals.
R 2p
(i) x ( x3 + 5)dx.
R cos x
(ii) e sin xdx.
R cos x
(iii) sin x dx.
R
(iv) sinh x cosh xdx.
R
(v) (1 + cos θ )3 sin θdθ.

71
R tdt
(vi) p .
(4t2 1)
R xdx
(vii) 4x2 +1
.
R
(viii) p2vdv 2 .
(1 v )

4.5.2 Integration by Parts

Integration by parts is a consequence of the product rule for derivatives and the anti derivative
concept.
d dv
R d R
The product rule states that dx (uv) = u dx + v du
dx , =) dx ( uv ) = dv
u dx + v du
dx .
R dv R dv R du
By using the antiderivative concept we have uv = (u dx + v du
dx ) dx =) uv = u dx dx + v dx dx.
R dv R du
Hence u dx dx = uv v dx dx which is the formula for integration by parts. In using this
method, care must be taken in the choice of u and v as we illustrate in the following examples.

R R R R
Example 4.3 : Evaluate the following integrals: (i) xe x dx (ii) ln xdx (iii) tan 1 xdx (iv) x2 e x dx.

Solution:
dv
(i) Choose u = x and dx = e x ;then du x
dx = 1 and v = e .
R R x
Thus xe x dx = x.e x e dx = xe x e x + C = e x ( x 1) + C.
dv
It is important to note that the alternative choice of u and dx does not lead to an easier integral
and a solution.
dv
(ii) Let u = ln x and dx = 1. du 1
dx = x and v = x.
R R 1 R
Thus ln xdx = x ln x x x dx = x ln x dx = x ln x x + C = x (ln x 1) + C.
dv
(iii) Let u = tan 1 x and dx = 1; du 1
dx = 1+ x2 and v = x;
R R x 1
Thus tan 1 xdx = x tan 1 x 1+ x 2
dx = x tan 1 x 2 ln(1 + x2 ) + C.
The next example illustrates the repeated use of the method of integration by parts.
dv
(iv) Let u = x2 and dx = e x ; du x
dx = 2x and v = e .
R R
Thus x2 e x dx = x2 e x 2 xe x dx
R
We evaluated xe x dx in (i) above using the same method and can recall that result to conclude
the solution as;
R 2 x R
x e dx = x2 e x 2 xe x dx = x2 e x 2[e x ( x 1)] + C.
Exercise 4.3: Explore the evaluation of the following integrals:

72
R
(i) x ln xdx
R
(ii) x sin axdx
R
(iii) x cos(2x + 1)dx
R
(iv) x2 cos axdx
R 1
(v) sin axdx
R
(vi) x2 tan 1 xdx

4.5.3 Integration by partial fractions


2 3 2( x 3)+3( x +1) 5x 3
We recall that x +1 + x 3 = ( x +1)( x 3)
= x2 2x 3

The reverse process of finding constants A and B such that


5x 3 5x 3 A B
= x2 2x 3 ( x +1)( x 3)
= x +1 + x 3 ,where A and B are constants is called resolution
into partial fractions.
In the particular example above, we simplify both sides and explore the identity that results
from the numerators.
Either we consider the identity for special values of x such as x = 3 and x = 1 or we equate
coefficients of like powers to get A = 2 and B = 3

f (x)
Remark 4.1 : Generally, to write a rational function g( x )
in terms of its partial fractions, the following
guidelines should be observed.

(1) The degree of the numerator f ( x ) should be less than the degree of the denominator g( x ). If
not, we first perform a long division and get a remainder r ( x ) with degree less than the degree
of g( x ).
(2) The factors of g( x ) should be known and the steps below followed.
(a) Let x r be a linear factor of g( x ). Suppose m is the highest power of x r that divides g( x ).
Then assign the sum of m partial fractions to this factor thus:

A1 A2 Am
+ + ... +
x r (x r) 2 ( x r )m

This should be done for each distinct linear factor of g( x ).


(b) Let x2 + px + q be a quadratic factor of g( x ). Suppose n is the highest power of the quadratic
factor ( x2 + px + q) that divides g( x ). Then, to this factor, assign the sum of n partial fractions
B1 x +C1 B2 x +C2 Bn x +Cn
x2 + px +q
+ ( x2 + px +q)2
+ ... + ( x2 + px +q)n

73
This should be done for each distinct quadratic factor of g( x ).
f (x)
(c) Put the original rational function g( x) equal to the sum of all these partial fractions. Simplify
both sides and examine the numerators.
(d) Equate the coefficients of the corresponding powers of x or use particular values on the
identities to get the unknown coefficients.
By resolving rational functions into partial fractions we have simpler functions that are much
easier to integrate. We illustrate the use of the method of integrating by partial fractions in the
examples below.

f (x)
+4
R R x +4
Example 4.4 : (i) Express g( x) = ( x2 +12x
)( x 1)2
and find 4 2x
( x2 +1)( x 1)2
dx, Evaluate (ii) x3 +3x2 10x
dx,
R dx R x5 x4 3x +5
(iii) 1 x2 , (iv) x4 2x3 +2x2 2x +1
dx.

Solutions:(i) The degree of f ( x ) is less than that of g( x ) which is already written as a product
of linear and quadratic factors. So let
2x +4 Ax + B C D
( x2 +1)( x 1)2
= x 2 +1
+ x 1 + ( x 1)2
,then,

2x + 4 = ( Ax + B)( x 1)2 + C ( x 1)( x2 + 1) + D ( x2 + 1) = ( A + C ) x2 + ( 2A + B C+


D ) x2 + ( A 2B + C ) x + ( B C + D ).
Equating coefficients, we have
A+C = 0
2A + B C+D =0
A 2B + C = 2
B C+D =4
Solving these equations simultaneously we have
A = 2, B = 1, C = 2, D = 1.
) 2x +4
( x2 +1)( x 1)2
= 2x +1
x 2 +1
+ 2
x 1 + 1
( x 1)2
.

Alternatively we could use the identity


4 2x = ( Ax + B)( x 1)2 + C ( x2 + 1)( x 1) + D ( x2 + 1) which is true for all values of x. When
x = 1 gives 2 = 2D and D = 1. We have to equate coefficients to get A, B and C.
R R 2x+1 R 2 R 1 R 2x R 1 R 2
Thus ( x2 +41)(2x
x 1 ) 2 dx = x 2 +1 dx + x 1 dx + ( x 1 ) 2 dx = x 2 +1 dx + x 2 +1 dx + x 1 dx +
R 1
( x 1)2
dx
1
= ln( x2 + 1) + tan 1 x + 2 ln( x 1) x 1 + C.

(ii) The degree of f ( x ) = x + 4 is less than the degree of g( x ) = x3 + 3x2 10x and factors of
g( x ) are x ( x 2)( x + 5).

74
Thus,
x +4 x +4 A B C
x3 +3x2 10x
= x ( x 2)( x +5)
= x + x 2 + x +5 .

Solve simultaneously we obtain


2 3 1 x +4 2 3 1
R 2
A = 5 ,B = 7 and C = 35 . So x ( x 2)( x +5)
= 5x + 7( x 2)
+ 35( x +5)
= 5x dx +
R 3
R 1
7( x 2)
dx + 35( x +5)
dx
R
) x +4
x ( x 2)( x +5)
dx = 5
2
ln j x j + 37 ln j x 2j + 1
35 ln j x + 5j + C.

1 1
(iii) 1 x2
= (1+ x )(1 x )
.
A
Let 1
(1+ x )(1 x )
= 1+ x + 1 B x =) 1 = A(1 x ) + B (1 + x )

1=A Ax + B + Bx = ( B A ) x + ( B + A ).
Equating coefficients yields
B A = 0 and B + A = 1
=) B = A = 21 .
R R R
) 1 1x2 dx = 2(11+x) dx + 1
2(1 x )
dx = 12 ln j1 + x j 1
2 ln j1 xj + C = 1
2 ln 1+ x
1 x +C

(iv) The degree of f ( x ) is greater than the degree of g( x ), so, we first do a long division to get
x5 x4 3x +5 2x +4
x4 2x3 +2x2 2x +1
= x+1+ x4 2x3 +2x2 2x +1

We factor the denominator of the remainder term to get ( x2 + 1)( x 1)2 . Thus
2x +4 2x +1 2 1
( x2 +1)( x 1)2
= x 2 +1 x 1 + ( x 1)2
R x5 x4 3x +5 +1
R
) = [ x + 1 + 2x
x4 2x3 +2x2 2x +1
dx
x 2 +1
2 1
x 1 + ( x 1)2 ] dx
R R R R R 2 R 1
= xdx + 1dx + x22x+1 dx + x21+1 dx x 1 dx + ( x 1)2
dx
x2 1
= 2 + x + ln( x2 + 1) + tan 1 x 2 ln j x 1j x 1 + C.
Exercise 4.4: Explore the evaluation of the following integrals:
R dθ
(i)
θ 2 +θ 2 2θ
R xdx
(ii) x2 +4x 5
R (x+1)dx
(iii) x2 +4x 5
R cos xdx
(iv) sin2 x +sin x 6
R dx
(v) x ( x 2 + x +1)

75
R dx
(vi) ( x +1)( x2 +1)

4.5.4 Integration by Reduction Formula

This method involves the reduction of higher powers of an integrand to lower powers in order
to facilitate the integration. We explain the process by solving some examples.
R R
Example 4.5 : (i) Obtain a reduction formula for In = tann xdx and hence evaluate tan4 xdx.
R R
(ii) Obtain a reduction formula for Jn = cosn xdx, then evaluate cos4 xdx.
R
(iii) Evaluate sinn ax cos axdx.
Solutions:
R R
(i) In = tann xdx = tann 2 x tan2 xdx, n 6= 1
R R R
= tann 2 x (sec2 x 1)dx = tann 2 x sec2 xdx tann 2 xdx
n 1 R
= tann 1 x tann 2 xdx = n 1 1 tann 1 x In 2.
R
tan4 xdx is evaluated as follows:
tan 3x
Substitute n = 4; I4 = 3 I2 .
R R
For n = 2; I2 = tan1 x I0 .But I0 = tan0 xdx = 1dx = x + C1 .
R
Thus I4 = tan4 xdx = 13 tan3 x [tan x x + C1 ]
= 13 tan3 x tan x + x + C,where C = C1 .
R R
(ii) Jn = cosn xdx = cosn 1x cos xdx
R
= cosn 1 x sin x + (n 1) sin2 x cosn
2 xdx,using integrating by parts.
R R
Jn = cosn xdx = cosn 1 x sin x + (n 1) (1 cos2 ) x cosn 2 xdx
R R
= cosn 1 x sin x + (n 1) cosn 2 xdx (n 1) cosn xdx
= cosn 1 x sin x + (n 1) Jn 2 (n 1) Jn
and [1 + (n 1)] Jn = nJn = cosn 1 x sin x + (n 1) Jn 2
R n 1
Hence Jn = cosn xdx = cos nx sin x + n n 1 Jn 2 .
R 3
With n = 4, J4 = cos4 xdx = cos x4 sin x + 34 J2 .
R R
where J2 = cos2 xdx = cos x2sin x + 12 dx
R 3
and J4 = cos4 xdx = cos x3 sin x + 34 ( cos x2sin x ) + 12 x + C.
R n+1 ax
(iii) sinn ax cos axdx = sin
( n +1) a
+ C, n 6= 1.Directly.

76
R R cos ax
For n = 1, cot axdx = sin ax dx = 1a ln jsin ax j + C.
Similarly,
R n+1 ax
cosn ax sin axdx = cos ( n +1) a
+ C, n 6= 1 but n = 1 gives
R R sin ax 1
tan axdx = cos ax dx = a ln jcos ax j + C.
Exercise 4.5: Explore the following problems:
R n 1 R R
sinn xdx = sin nx cos x + n n 1 sinn 2 xdx and use it to evaluate sin4 xdx.
R x m+1 (ln x )n n
R m R
(ii) Show that x m (ln x )n dx = m +1 m +1 x (ln x )n 1 dx;hence, evaluate x3 (ln x )2 dx.
R R R
(iii) Show that x n e x dx = x n e x n x n 1 e x dx, and hence evaluate x3 e x dx.
R n 2 R R
(iv) Show that secn xdx = sec n x1tan x + nn 21 secn 2 xdx, n 6= 1,and hence evaluate sec3 xdx.
1

1 Provided by Departments of Mathematics,UNIJOS and PLASU - 2013

77
Chapter 5

The Definite Integral and Its Applications

5.1 Objectives:

In this module, we shift our attention to the definite integral and its applications after having
studied the indefinite integral in the previous one. Students are stimulated through on-line,
classroom and laboratory activities, to discover that this introduction of the integral depends on
the limit concept, which is the fundamental basis of calculus.

5.2 Learning Outcomes

At the end of this module, students should be able to:


Appreciate the limit property of integrals and evaluate definite integrals;
Identify the definite integral with the area under a curve;
Identify the definite integral with length of an arc of a curve;
Identify the definite integral with volumes of a solid of revolution;
Use the definite integral to investigate problems in physical sciences, engineering and eco-
nomics;
Use the definite integral to solve differential equations.

5.3 Learning Activities

Students should:
explore notes and exercises, individually and in groups;

78
explore and use related materials on the Intranet, especially
the e-granary and MIT open course ware;
explore and use related materials on the OLI Calculus course
on the Internet(http://www.cmu.edu/oLI/courses/);
solve relevant questions from past MTH103 Examinations.

5.4 Introduction:

The Definite Integral as area under a curve


Let us consider a function y = f ( x ) which is positive and continuous in the range a x b. Let
the
interval I = [ a, b] be divided or partition into n parts by arbitrarily choosing points a = x0 , x1 , x2 , ,
b
on the x axis such that
a = x0 < x1 < x2 < < xn 1 < xn = b.
Suppose further that ξ r represents a point on the x axis lying between the (r 1)th and rth
points in the division of the interval ( a, b) with
xr 1 < ξ r < xr
and
δr = xr xr 1.
Then an approximation to the area of the rth strip is given by
f ( ξ r ) δr
The whole area bounded by the curve, the x axis and the lines x = a, x = b is approximately
represented by the sum Sn of n expressions and it is given by
n
Sn = ∑ f ( ξ r ) δr .
r =1

If the limit of this sum as n tends to infinity exists independently of the way of choosing xr and
ξ r and as δr tends to zero then this limit-sum is called the definite integral of f from x = a to
x = b and is written as Z
n b
lim
n!∞
∑ f ( ξ r ) δr =
a
f ( x )dx.
r =1

79
The relationship between this integral and the indefinite integral may be found by considering
the variation of the function f over a small interval x = ξ to x = ξ + δξ .
If f min and f max are respectively the least and greatest value of f ( x ) in this interval, the elemental
area δA (shown shaded) must satisfy the inequality

f min δξ < δA < f max δξ.

As δξ tends to zero, f min and f max both approach f (ξ ) and hence

dA
= f ( ξ ).

The area A(ξ ) under the curve from x = a to x = ξ is therefore an indefinite integral of f (ξ ).
Suppose now that F (ξ ) is any indefinite integral of f (ξ ); then

F (ξ ) = A(ξ ) + C

where C is an arbitrary constant. Since A( a) = 0, we have (with ξ = b)


Z b
f ( x )dx = F (b) F ( a ).
a

This is the formula for computing the definite integral where a and b are called the lower and
upper limits of the integral, respectively. The value of the definite integral is the same as the area
under the curve y = f ( x ) between the lines, x = a and x = b.
Equivalently we can explore the definition of the integral as area under the curve by defining
the lower and upper sums and hence the concept of integrability. This can be done in the special
case when the function is f ( x ) = x, x 2 [1, 3] by exploring the diagrams below. The student is
encouraged to independently do so before the lecture on this module is delivered.

80
R2 R π/2
Example 5.1 : Evaluate the integrals: (i) 1 3xdx, (ii) 0 cos xdx.
R
(i) We use our knowledge of indefinite integrals and do the evaluation using the limits. 3xdx =
3 2
2 x + C = F ( x ) + C.
R2 2
Therefore 1 3xdx = 3x2 j21 = F (2) + C [ F (1) + C ] = 6 32 = 92 .
R π/2
(ii) 0 cos xdx = F ( π2 ) F (0) = [sin x ]0π/2 = sin π/2 sin 0 = 1 0 = 1.

5.5 Properties of the definite Integral

If f ( x ) and g( x ) are integrable functions of x in [ a, b], then

Rb Ra Ra Rb
(a) a f ( x )dx = b f ( x ) dx since b f ( x ) dx = [ F ( a ) F ( b )] = F ( b ) F ( a ) = a f ( x ) dx.
Rc Rb Rb Rc Rb
(b) For a c b, a f ( x )dx + c f ( x )dx = a f ( x )dx. This follows since a f ( x )dx + c f ( x )dx =
[ F (c) F ( a)] + [ F (b) F (c)]
Rb
= F (b) F ( a) = a f ( x )dx

81
Rb Rb Rb Rb
(c)a f f ( x ) + g ( x )g dx = a f ( x ) dx + a g ( x ) dx since a f f ( x ) + g ( x )g dx = F ( b )R+ G ( b )
b
F ( a) G ( a) which when rearranged becomes f F (b) F ( a)g + f G (b) G ( a)g = a f ( x )dx +
Rb
a g ( x ) dx
Rb Rb
(d) If k is a constant, then a k f ( x )dx = k a f ( x )dx.
Ra Ra
(e) 0 f ( x )dx = 0 f ( a x )dx. We prove this by letting x = a y in the first integral and then
Ra R0 Ra
using property (a) we have 0 f ( x )dx = a f (a y)dy = 0 f ( a y)dy. Whether we
write y or x in the last integral is quite irrelevant, since the value of the integral depends
only on the form of the integrand and the values of the limits of integration.

(f) If x is a variable
Rx upper limit of integration, a is a constant, and f ( x ) is a continuous function,
d
then dx a f (t)dt = f ( x ). This follows from the definition of the definite integral, for
d
Rx d
dx a f ( t ) dt = dx f F ( x ) F ( a)g = f ( x ).

R2 R π
d
Rx R4
Example 5.2 : Evaluate the following (1) 1 3xdx, (2) 2
0 cos xdx, (3) dx π sin2 tdt, (4) 1 ( x2 +
4
x3 )dx,
(5) Express the area under the curve, y = 9 x2 between the lines x = 0 and x = 1 as a definite
integral and then calculate it. The student is encouraged to sketch a graph of the curves for insight.

R2 R2 2
Solutions:(i) 1 3xdx = 3 1 xdx = 32 x2 1 = 32 22 12 = 29 .
Rπ π
(2) 02 cos xdx = [sin x ]02 = sin π2 sin 0 = 1 0 = 1.
d
Rx 2 2 d
Rx
(3) dx π sin tdt = sin x. This follows from property (f), that is dx f (t)dt = f ( x ).
4 a

R4 h i4 h i h i
x3 x4 43 44 13 14 64 1
(4) 1 ( x2 + x3 )dx = 3 + 4 1 = 3 + 4 3 + 4 = 3 + 64 3 + 14
256 7 1024 7 339
= 3 12 = 12 = 4 = 84 34 .
(5) The area bounded by the curve y = f ( x ), the x axis and the ordinates x = a, x = b is given
Rb Rb
by a ydx = a f ( x )dx.Thus the area bounded by the curve
R1 R1 h i
3 1
y = 9 x2 between x = 0 and x = 1 is 0 (9 x2 )dx. 0 (9 x2 )dx = 9x x3 = 9 13 = 26 3 = 0
8 23 square units.

Example 5.3 : (i) Calculate the area bounded by the curve y = x2 and the x axis and between the lines
x = 1and x = 3.
(ii) Calculate the area between the curve y = 3x ( x 4) and the x axis
(iii) Find the area bounded by the curve y = cos x, the x axis and the ordinates x = 0 and x = π.

82
Solutions:(i) The student is encouraged to sketch the curves and display the areas under inves-
tigation.
R3 R3 h 3 i3 3 13
The required area is the definite integral 1 x2 dx. ) A = 1 x2 dx = x3 = 33 26
3 = 3 square
1
units.
(ii)
Z 4
Area = ydx
0
Z 4
= 3x ( x 4) dx
0
Z 4
= (3x2 12x )dx
0
h i4
= x3 6x2
0
= 34 square units.

The negative sign should be discarded (i.e. Area A = 34 square units). The negative sign only
explains the fact that the curve lies below the x-axis for the range of x under consideration.
Rπ π
(iii) Area AOB = 02 cos xdx = [sin x ]02 = sin π2 sin 0 = 1square unit.

Area BCD = π cos xdx = [sin x ]ππ = sin π sin π2 = 1 square unit. We discard the negative
2 2
sign as it only explains that the area is below the x-axis. Area BCD = 1. The area bounded by
the curve y = cos x, the x axis and the ordinates x = 0 and x = π is equal to the Area AOB+
Area BCD = 1square unit+1square unit = 2 square units
Rπ Rπ Rπ π
Note that Area required = 02 cos xdx π cos xdx. But note that
0 cos xdx = [sin x ]0 =
2
sin π sin 0 = 0.

The area required is not given by the definite integral, 0 cos xdx.

5.6 Area between two curves

Given two curves y = f ( x ) and y = g( x ) such that f ( x ) > g( x ), where f ( x ) and g( x ) are non
negative, then the area between them bounded by the ordinates x = a and x = b, is
Z b Z b Z b
( f (x) g( x )) dx = f ( x )dx g( x )dx.
a a a

Example 5.4 : Calculate the area between the curve y = x2 and the line y = x.

Solution:Sketch the diagram for insight :

83
The curves y = x2 and y = x intersect at the origin (0, 0) and at the point P(1, 1).
R1 R1 R1 2 1 1 1
Therefore the area required is given by 0 ( x x2 )dx = 0 xdx 0 x dx = 2 3 = 6 square
unit.
Exercises 5.1 Explore the following exercises.
(1) Calculate the area enclosed by the curve y = x2 (1 x ) ,the axis of x and the ordinate x = 0
and x = 1
(2) Find the area enclosed by the axis of x and that part of the curve y = 5x 6 x2 for which y
is positive.
p
(3) Find the area enclosed between the curve y = x3 and the straight line y = 2x.
(4) Find the area enclosed by y2 = 4x and x2 = 4y
(5) Show that the area contained by the curve y = a + bx + cx2 + dx3 the x axis and the ordi-
h
nates at x = h is equal to h (y1 + y2 ) where y1 , y2 are the values when x = 3

5.7 Arc Length

Consider the function y = f ( x ) whose curve passes through the point P and Q with the ordinates
x = a and x = b respectively.
The length of the arc PQ is given by
Z bh i1
0 2 2
L( PQ) = 1 + f (x) dx.
a

3
Example 5.5 : Find the length of the arc of the curve y = 32 x 2 between the ordinates x = 3 and x = 8.

3 2 3 12 1 1
Solution:Now f ( x ) = 32 x 2 ) f 0 ( x ) = 3 2x = x 2 . Take a = 3 and b = 8. f 0 ( x ) = x 2 . Thus, the
1 8
R8 1 2 2 R8 1
3
2(1+ x ) 2
3
2(1+8) 2
3
2(1+3) 2
length of the arc is L = 3 1+ x 2 dx = 3 [1 + x ] dx =
2
3 = 3 3 =
3
16 units.

5.8 Volumes of Solids of Revolution

(A) Volume formed by rotating a curve y = f ( x ) around the x axis.


Suppose a region below a curve y = f ( x ) of a continuous function f between x = a and x = b is
rotated about the x axis, it generates a solid figure called a solid of revolution. Because of the

84
symmetrical way in which it is generated (by a rotation of a plane figure) it is easy to compute
its volume. Such a region will be shown in class.
Let [ a, b] be divided (partition) into n subintervals of equal lengths. Consider the contribution
to the solid of revolution given by revolving the strip about the x-axis. The strip sweeps out a
slab , where the minimum radius is mi = f (ti 1 ) and the maximum radius is Mi = f (ti ).The
thickness of the slab is b n a = δx. The volume of a circular slab of radius r and thickness h is of
course πr2 h.Since the radius of the slab varies from mi to Mi , its volume Vslab satisfies
πm2i ( b n a ) Vslab πMi2 ( b n a ). Thus for the volume V of the whole solid of revolution, we have
n n Rb
( b n a ) ∑ πm2i V ( b n a ) ∑ πMi2 for all n.The extremes of the inequality approach π f ( x )2 dx
i =1 i =1 a
as n ! ∞, and the desired volume V is given by
Z b Z b
V = lim
δx !0
∑ πy 2
δx =
a
2
πy dx = π
a
y2 dx

This is the formula for the volume of solid of revolution around the x axis.
(B) Volume of solid by rotating of curve around the y axis.
Using similar argument outlined above, the volume of revolution about the y-axis can also be
Rb
derived and it is given by V = a πx2 dy.

Example 5.6 : (i) The area enclosed by the curve x = 3(y2 1) and the line x = 0, x = 24 is rotated
through 4 right angles around the x axis. Find the volume of the solid generated. (ii) The portion of the
curve y = x2 + 1 between the ordinates x = 1 and x = 3 is rotated through 360 around the y axis,
calculate the volume of revolution.

R 24
Solution: (i) Here y2 = 1 + 31 x and the required volume is V = π 0 1 + 31 x dx = π 24 + 576
6 =
120π cubic units.
(ii) Now y = x2 + 1, a = y ( x = 1) = 12 + 1 = 2 b = y( x = 3) = 32 + 1 = 10
p
But y = x2 + 1 =) y 1 = x2 ) x = y 1.
R 10 R 10 p 2 R 10 y2
10
102 ( 22 )
)V = 2 πx2 dx = 2 π y 1 dy = π 2 (y 1) dy = π 2 y =π 2 10 ( 2
2
40π cubic units.

5.9 Mean or Average Value of a Function

Suppose that y is an integrable function of x over [ a, b] .Suppose that the range AB from x = a
and x = b is divided (partition) into n equal subrange each of width δx. Let y1 , y2, , yn be the
values of y of the middle point of each subrange. The arithmetic mean of these n values of y is
1
n ( y1 + y2 + + yn ) and since δx = b n a , this can be written y1 +yb2 + a +yn

85
If this expression has a limiting value as δx ! 0 ,then this limiting value is
Z b
1
y=ydx
b a a
and this is called the mean value of y over the range of [ a, b].

Example 5.7 : Find the mean value of sin x over the range x = 0 and x = 12 π.

Solution: Here y = sin x, a = 0, b = 12 π and from the formula, the mean value ȳ of y is
R π2 π
ȳ = 1 1 2 2 2
0 sin xdx = π [ cos x ]0 = π = 0.637unit
2π 0

Exercise 5.2: Explore the problems below drawing their appropriate diagrams.
R1 R2p
(1) Evaluate the integrals (a) 0 x (1 x )5 dx (b) 0 1 + 2x2 dx
R Π
(2) Show that 2
0 xsin2 xdx = 1
16 Π2 + 4
R2 p
(3) By means of substitution x2 = 1
show that pdx = 1
19 2
u 1 x2 5x2 1 2

(4) Calculate the area between the curve y = 4 + 2x x2 and line y = 4


(5) Sketch the curve y = x ( x 1) ( x 2) and find the area enclosed by the curve and axis of x
between x = 0 and x = 2
(6) Sketch the curve y = x (3 x ) and find the area contained between the curve X axis and
ordinates x = 0, x = 5
(7) Find the mean value of the ordinate of the curve y = 4 x2 over the range 2 x 2
(8) Find the mean value of x varied from 0 to π of (a) sinx2 (b) xsinx
(9) The pressure P kilograms per square centimeter and the volume V cubic centimeter of quan-
tity of gas are related by the laws PV 1.2 = 1000. Fond the mean pressure as the gas increases
from 3cm3 to 8cm3 .
(10) The portion of the curve xy = 8 from x = 2 to x = 4 is rotated about the x-axis. find the
volume generated.
(11) Find the volume generated if the area enclosed by y = 3x2 x3 is rotated around the x axis
(13) The area bounded by the curve y2 = 20x and the line x=0 and the line x=10 is rotated about
the axis of Y show that the volume of the solid obtained is 50Π and find the volume of the solid
obtained by rotating the same area around the axis of x.

5.10 Moment of Inertia

If m1 , m2 , , mn are masses of the system of particles situated at points p1 , p2 , , pn whose


perpendicular distance from a straight line are r1 , r2, , rn , the sum of the product of each mass

86
and the square of its distance from the line is called moment of inertia of the system with respect
to the given line.

Moment of Inertia

This moment sometimes also called the second moment and it is conventionally denoted by I
so that
I = m1 r12 + m2 r22 + 2
. + mn rn. .
If we imagine the total mass M = m1 + m2 + + mn to be concentrated at a point at distance
k from the given line such that this single mass has the same moment of inertia about the given
line as the system of particles, then

Mk2 = I = m1 r12 + m2 r22 + . + mn rn2 .

The distance k calculated from this equation is known as the radius of gyration of the system
around the given line. The moment of inertia of a rigid body about an axis is required in dy-
namics to express the kinetic energy of the body and it is useful to set up two general theorems
before discussing the moment of inertia of a specific rigid body. These theorems are as follows:
If the moment of inertia of a system of particles of the total mass M about an axis through the
centre of mass is Mk2 , the moment of inertia of the system about a parallel axis distance a from
the first is M k2 + a2 .

87
Suppose the particles of masses m1 , m2 , ...., mn and are at the distance r1 , r2 , ..., rn from the axis
through the centre of mass and distances R1 , R2 , ...., Rn from the second (parallel) axis. Let the
two axes meet a plane perpendicular to them in G and O respectively. If this plane passes the
point P1 occupied by first particle . Since OG = a, we shall have R21 = r12 + a2 2ar1 cosθ.
And similar relations between R and r hold for all the n particles of the system.

Parallel Axis Theorem

The moment of inertia I of the system about the axis through O is given by I = ∑ mR2 =
∑ m rr + a2 2ar cos θ where ∑ denote summation over all the particles of the system and this
can be written I = ∑ mr2 + a2 ∑ m 2a ∑ mr cos θ. Now ∑ m = M, ∑ mr2 = Mk2 ( the momentof
inertia about the axis through G) and ∑ mrcosθ = 0 for this latter sum when divided by M gives
the distance in the direction G of the centre of mass from point G. Hence I = Mk2 + Ma2 and

the theorem is proved.


(ii) The moment of inertia of the system of particles lying in the plane about two perpendicular
axes in the plane meeting a point O are A and B respectively. The moment of inertia of the

88
system about an axis through O perpendicular to the plane is A + B.

Perpendicular Axis Theorem

From the diagram, we can see that, OX and OY are the given perpendicular axes in the plane and
OZ and OY are the given perpendicular axes in the plane and OZ is the axis perpendicular to
the plane. P1 , P2 , ...., Pn are the points ( x1 , y1 ), ( x2 , y2 ), ....( xn , yn ) occupied by particles of masses
m1 , m2 , ....., mn. Since the moment of inertia about OY, OX are respectively A and B, A = m1 x12 +
m2 x22 + .... + mn xn2 and by addition, A + B = m1 x12 + y21 + m2 x22 + y22 + ... + mn xn2 + y2n . If
OP1 = r1 , OP2 = r2 , ..., OPn = rn , it is clear from the diagram that r12 = x12 + y21 , r22 = x22 + y22 ,
...,rn2 = xn2 + y2n , hence A + B = m1 r12 + m2 r22 + ... + mn rn2 and this is the moment of inertia of the
system around OZ.
An extension from system of particles to continuous body can be obtained by replacing the
particle by elements of the body and using integral in place of summation. We give below the
methods of calculating the moment of inertia about specific axes of a thin rod, a circular disc and
a sphere, with uniform density ρ in all cases.
An extension from system of particles to continuous body can be obtained by replacing the
particle by elements of the body and using integral in place of summation. We give below the
calculation for the moment of inertia about specific axes of a thin rod, a circular disc and a sphere,
with uniform density ρ in all cases.

89
5.10.1 Thin Rod

Suppose AB is a thin rod of length 2l lying along the axis OX with its midpoint at the origin O,
the mass of an element PQ of length δx at the distance x from O is ρδx and its moment of inertia
about the axis of y is ρδx x2 .

Fig. 4.14 Thin Rod

Hence the moment of inertia I0 of the whole rod about a line through its centre and perpendicular
to its length is give by
Z l l
1 2
I0 = ρx2 δx = p x3 = ρl 3 .
l 3 l 3
Since the total mass M of the rod is 2ρl, then the moment of inertia can be written in the form
I0 = 31 Ml 3 , showing that the radius of gyration k is given by k2 = 13 l 3 . If the moment of inertia
I A about an axis perpendicular to the length of the rod and passing through the end of A is
required, the parallel axis theorem gives I A = I0 + Ml 3 = 34 Ml 3

5.10.2 Circular Disc

The moment of inertia of a circular disc about an axis through its centre O perpendicular to the
plane of the disc can be found by considering the ring bounded by circles of radii r and r + δr

90
. The mass of the element is approximately 2πρrδr and its moment of inertia about the axis is
2πρrδr r2 .

Fig. 4.15
Hence if a is the radius of the disc, the total moment of inertia I0 is
Z a a
1 4 1
I0 = 2πρr3 dr = 2πρ r = πρa4 .
0 4 0 2

Since the total mass M of the disc about a diameter is A, then by symmetry, the moment of
inertia about a perpendicular diameter is also A. Therefore, by second of the general theorems,
the moment of inertia about an axis perpendicular to the plane of the disc is 2A and we have
2A = 21 ma2 given A = 14 ma2 .

91
5.10.3 Sphere

Let the sphere be of radius r with its centre at the origin O. Consider an element in the form of a
circular disc of radius y, thickness δx at distance x from the axis OY.

Sphere

The mass of this element is approximately πρy2 δx and by the theorem (ii) above, the moment of
inertia about OX of the whole sphere is therefore,

Z r Z
r 2
1 4 1
πρy d x = πρ r2 x2 d x, since x2 + y2 = r2
r 2 2 r
1 2 2 3 1 5 r
= πρ r4 r x + x
2 3 5 r
8 5
= πr .
15

92
Since the mass M of the sphere is 34 πρr3 , then the moment of inertia of the sphere can be written
as 25 Mr2 .

Example 5.8 : A uniform lamina is bounded by the curve y = 8x3 , the axis of x and an ordinate x = 1.
Find the radius of gyration about a line perpendicular to its plane through the origin of coordinates.

Solution: An illustrative figure can show the lamina and an element strip of height y and width
δx. If ρ is the surface density of the lamina, the mass of the strip is approximately ρyδx and its
moment of inertia about the y axis is ρyδx x2 . Hence the moment of inertia Iy of the whole
R1 2 R1 5 h i1
3 1 6
lamina about the y axis is given by, since y = 8x Iy = 0 ρx ydx = 8ρ 0 x dx = 8ρ 6 x =
0
4
moment of inertia of the strip about x axis is, using the result of thin rod, ρydx 13 y3 and
3 ρ.The
R1 R1 9 h i1
the moment of inertia about the x axis is Ix = 0 13 ρy3 = 5123 ρ 0 x dx = 512
3 ρ 1 10
10 x = 256
15 ρ.
0
By the general theorem 2, the moment of inertia about an axis through O perpendicular to the
256
plane of the lamina is Ix + Iy = 15 + 43 ρ = 92
5 ρ.

Exercises 5.5: Explore the problems below, sketching a diagram anytime you find it useful to do
so.
(1) The smaller of the two areas bounded by the curve y2 = 4x, the line y = 2 and the line y = 4
rotates through 4 right angles about the axispof x, prove that the volume so formed is 18π and
that its radius of gyration about the axis is 2 2
(2) Find by integration, the radius of gyration of a uniform semi circular-disc of radius a about
its bounding diameter. Deduce its radius of gyration about a parallel axis through the centroid
G of the disc assuming G is a distant 4a
3 π from the bounding diameter.
(3) Show that the moment of inertia about its axis of a uniform solid circular cone of mass M and
3
base radius r is 10 mr2 .
(4)A uniform thin hemispherical shell has mass M and radius r. Show that its moment of inertia
about the radius perpendicular to its base is 23 Mr2
(5) The area enclosed by the curve y2 = 4ax, the axis of x and the line x = h is rotated through
4 right angles about the axis of x to form a solid. Find the radius of gyration about the x-axis of
this solid.

5.11 Differential Equations

One area of application of integration is in the solutions of differential equations. An equation


dy ∂ f d2 y ∂ f ( x,y)
involving differential coefficients such as dx , ∂x , dx2 , ∂y and so on, is called a differential
dy d2 y
equation. When the equation involves ordinary differential coefficients, dx , dx2 ,e.t.c., it is re-
ferred to as an ordinary differential equation (ODE), but if it involves partial derivatives such

93
∂f ∂ f ( x,y) ∂2 f ( x,y)
as ∂x , ∂y , ∂y∂x ,e.t.c., the equation is called a partial differential equation (PDE). The or-
der of a differential equation is determined by the highest derivative appearing in the equa-
tion. The degree of a differential equation is the highest index (power) of the highest derivative
dy
in the equation. For example, 2 dx y = x, is a first order differential equation of degree 1,
d2 y dy 1 d y 3 2 dy
dx2
+ dx y = 0, is of second order, and degree 1, ( dx 2) dx + x = 0, is of second order, and
degree 3.
We shall solve first order equations using the method of separation of variables. A first order
differential equation with variables separable is an equation which can be put into the form
dy dy
dx = f ( x ) g ( y ).For example, consider the equation dx = xy. The variables can be separated thus:
1 dy dy R dy R x2 x2
y dx = x, ) y = xdx. ) y = xdx, ) ln y = 2 + c ) y = Ae 2 .This y obtained by

solving the differential equation is called the general solution of the equation. If in addition,
y = 1,when x = 0, that is y(0) = 1, called the initial value. In that case, we obtain A = 1. Hence
x2
y = e 2 . This is called particular solution of the equation. The equation together with the initial
values is said to be an initial value problem (IVP).

dy dy
Example 5.9 : Solve the following differential equations: (i) x dx = 3y 1, (ii) 2( x + 1)y dx = x,(iii)
dx dy dy xy+3x y 3
dt = e x sin t, (iv) dx + ty = y, y(1) = 3, (v) dx = xy 2x +4y 8 .

Solutions:
dy dy R dy R
(i) x dx = 3y 1 , ) xdy = (3y 1)dx, ) 3y 1 = dx
x . ) 3y 1 = dx
x , ) 1
3 ln j 3y 1 j= ln j
x j +k.
3 +3k e3k
) 3y 1 = eln j xj = j x j3 e3k =) ) y = 31 x3 e3k + 13 = Ax3 + 13 , where A = 3 .
dy
(ii) 2( x + 1)y dx = x, ) 2ydy = 1+x x dx = (1 1+1 x )dx.
R R R R dx
) 2ydy = (1 1+1 x )dx = dx 1+ x = x ln j 1 + x j +k
x x
) y2 = x ln j 1 + x j +k = ln j1e+ xj + k, ) y2 = ln j1e+ xj + k.
R x R
(iii) dx
dt = e x sin t, ) dx
e x = sin tdt ) e dx = sin tdt, ) e x = cos t + A
) e x= cos t + c, where c = A, or x = ln j cos t + c j .
dy R R
(iv) dt + ty = y, y(1) = 3, ) dy y = (1 t)dt
t 1 1
) ln j y j= t 1 2
2t + C, ) y = Aet(1 2) . But y(1) = 3, ) 3 = Ae 2 , ) A = 3e 2 .
1 t t 1
) y = 3e 2 e t (1 2) = 3et(1 2) 2 .
dy xy+3x y 3 dy ( x 1)(y+3)
(v) dx = xy 2x +4y 8 , ) dx = (y 2)( x +4)
.
R y 2 R
) (y 2)( x + 4)dy = ( x 1)(y + 3)dx, ) y+3 dy = x 1
x +4 dx.

94
R R 2 R x R dx
) y+y 3 dy y +3 dy = x +4 dx x +4
R R
) (1 y+3 3 )dy 2 ln j y + 3 j= (1 x+4 4 )dx ln j x + 4 j +C
R R 3 R R 4
) dy y +3 dy 2 ln j y + 3 j= dx x +4 dx ln j x + 4 j +C

) y 3 ln j y + 3 j 2 ln j y + 3 j= x 4 ln j x + 4 j ln j x + 4 j +C
)y 5 ln j y + 3 j= x 5 ln j x + 4 j +C
) 5(ln j x + 4 j ln j y + 3 j) = x y+C
) 5 ln j x +4
y +3 j= x y + C, )j x +4 5
y +3 j = Ae x y.

Exercises 5.6: Explore the solution of the following equations.


dy y dy dy
(i ) dx = 2x (ii ) dx + y = x2 y (iii ) (1 + x2 ) dx = 9 + y2 , y(0) = 0.
dy
(iv) ( x2 1) dx + 2y = 0, y(2) = 3. (v) 2 sin θ dθ
dr = cos θ sin θ.
dy xy 2y x 2 dy
(vi ) dx = xy 3y+ x 3 (vii ) (1 + x2 ) dx = 1 + y2 , y (2) = 3
1 1
(viii ) x (y2 1) 2 dx y( x2 1) 2 dy = 0.

5.12 Application of Integration to Problems in Physical Sci-


ences & Engineering.

5.12.1 Displacement/Distance and Velocity/Speed

If a body travels in a line with a constant velocity v, then the distance S, covered after a time t is
given by the product j v j t, where j v j represents the speed of the body. If a body is moving in
a line with a velocity v(t), starting from time t0 to time t, then the distance from time t0 to time t
is given by
Z t
S= jv(t)jdt.
t0

Example 5.10 : (1) If the velocity at any time t seconds of a body travelling on a line is v(t) = (2t +
t2 )m/s. Find how far the body travels from t = 2s to t = 4s. (2) Suppose the velocity at time t of a
body moving on a line is v = cos πt 2 m/s. Thus at time t = 0, the body is moving at 1m/s in the positive
direction, while at time t = 2, the body is moving in the negative direction at the speed of 1m/s. (3)
The velocity of a moving particle along the x axis is given by v(t) = (1 2t)(2 t). Given that when
t = 0, x = 1, find t when the velocity is a minimum, and also determine (i) the displacement of the particle
at this value of t, (ii) the total distance travelled and the displacement of the object when t = 3.

Solutions:

95
Rt R4 43 22 92 92
(1) S = t j v(t) j dt = 2 (2t + t2 )dt = (42 + 3) ( 22 + 3) = 3. Thus the body travels 3m
0
from t = 2s to t = 4s.
(2) Distance travelled from t = 0 to t = 2 is given by
R2 R1 R2
S = 0 j cos π2 t j dt = 0 cos π2 tdt + 1 ( cos π2 t)dt = π2 sin π2 (1) 2 π
π sin 2 (0)
2
π sin π2 (2) 2
π sin π2
2 2 4 4
π + π = π . Thus, the total distance covered from t = 0 to t = 2 is π m.
(3) Note that the velocity v, of an object at any point x is defined as the rate of change of the
displacement x with respect to (w.r.t.) time t. Hence v = dx dx
dt , while the magnitude of dt , i.e. j v j is
the speed. v is minimum if dv dv
dt = 0. i.e., dt = 2(2 t) (1 2t) = 4t 5 = 0. ) t = 54 . But
d2 v
dt2
= 4 > 0. Hence the velocity is a minimum when t = 45 .
R R R
(i) Note that v(t) = dx
dt = (1 2t)(2 t), ) dx = (1 2t)(2 t)dt = (2t2 5t + 2)dt;
3 2 3 2
) x = 2t3 5t
2 + 2t + c. x = 1, when t = 0, ) c = 1. ) x = 2t3 5t
2 + 2t + 1. This is the
5
expression for the displacement of the particle at t = 4 . The displacement at
5
t= 4 is x = 23 ( 54 )3 5 5 2
2(4) + 2( 54 ) + 1 = 43
48 .
(ii) Total distance travelled when t = 3 is
R1 R2 R3 R1
S = 02 (1 2t)(2 t)dt + 1 (2t 1)(2 t)dt + 2 (1 2t)(2 t)dt = 02 (2t2 5t + 2)dt +
R2 2
R3 22
1 ( 2t + 5t 2)dt + 2 (2t 5t + 2)dt
2
h i h i
= 23 ( 12 )3 52 ( 12 )2 + 2( 12 ) 0 + 23 (2)3 52 (2)2 + 2(2) 2 1 3
(
3 2 ) 5 1 2
(
2 2 ) + 2 ( 1
2 )
2 5 2 5 11
+ 3 (3)
3
2 (3)
2 + 2(3) 3 (2)
3
2 (2)
2 + 2(2) = 24 + 27 13
24 + 6 =
15
4

The displacement at t = 3 is x = 23 (3)3 5


2 (3)
2 + 2(3) + 1 = 52 m.

5.12.2 Work

If the force acting on a body in the direction of its motion along a line is the function F (s) of the
position s of the body, then the work done by the force in moving the body from position s = a
to position s = b is
Z b
W= F (s)ds.
a

The force F required to stretch (or compress) a coil spring is proportional to the distance x it is
stretched (or compressed) from its natural length. That is F = kx for some constant k, the spring
constant.

Example 5.11 : Suppose that a spring is such that the force required to stretch it one metre from its

96
natural length is 4N. For a spring with constant k = 4, find the work done in stretching the spring a
distance of 4m from its natural length.

R4 4
Solution: Let x be the original or natural length of the spring, then W = 0 4xdx = 2x2 0
=
2 (4)2 (0)2 = 32.Thus 32J of work is done.

5.13 Application of Integration to Laws of Growth and Decay

In nature, the rate of change of any given quantity of most physical phenomena is always pro-
portional to the amount of the quantity at any instance. For instance, if the rate of change of a
population of bacteria in a culture is proportional to the bacteria present in the culture at any
dp dp
time t, we have that dt _ p =) dt = kp, where k is the proportionality constant. Solving the
equation by separation of variables, we obtain p = Aekt . Given that the population at time t = 0
is p0 , we obtain p0 = Aek 0 = A. ) p = p0 ekt .

Example 5.12 : (1) The population of a colony doubles in 50 days. In how many days will the population
triple?
(2) The population of a country doubles in 50 years. If the present population is 20,000,000:
(a) when will its population reach 30,000,000?
(b) what will its population be in 10 years?

Solutions: (1) Let the population at any time t be p.


R dp R
) dp
dt = kp, ) p = ktdt ) ln p = kt + C, ) p = Aekt .

If the population at t = 0 is p0 , then p0 = Ae0 = A, ) p = p0 ekt . p = 2p0 when t = 50,


ln 2
) 2p0 = p0 e50k , ) ln 2 = 50k, ) k = ln 2
50 . ) when p = 3p0 , we have 3p0 = p0 e 50 , ) t = 50 ln 3
ln 2 '
79 days.
ln 2
(2) (a) 2p0 = p0 e50k , ) k = ln 2
50 . ) p = p0 e 50 , ) p = 20, 000, 000e0.01386t .
) when p = 30, 000, 000,
30, 000, 000 = 20, 000, 000e0.01386t , ) 1.5 = e0.01386t .
) t= ln 1.5
0.01386 ' 29 . Thus, the population will be 30, 000, 000 in 29 years.
(b) For t = 10, p = 20, 000, 000e0.01386 10 = 22, 970, 000.
Note: Disintegration of a substance is equivalent to decay, hence we obtain the
dQ
equation dt = kQ, where Q represents the quantity of the substance present at time t,

97
R R
and the negative sign indicates decay. ) dQ
Q = kdt, ) ln Q = kt + C.

) Q = Ae kt .

(3) Suppose the original quantity is Q0 , when t = 0, then Q0 = Ae0 = A.


) Q = Q0 e kt .

1 100k , )
10% decays in 100 days implies 10 Q0 = Q0 e ln 0.1 = 100k.
ln 10 ln 10
) k= ln 0.1
100 = ln 10
100 , hence Q = Q0 e 100 t . Thus, for half-life of the substance, 12 Q0 = Q0 e 100 t .
Note that the half-life of a substance is the time it takes to disintegrate
to half of its original size. ) ln 0.5 = ln100
10
t, ) t = 100 ln 0.5
ln 10 = 30. 103. Hence, the half-life of the
substance is approximately 30.1 days.
Exercises 5.7: Explore the solution of the problems below:

Example 5.13 : (i) The bacteria count in a culture is 100, 000. In 2 12 hours, the number has increased
by 10%.
(a) In how many hours will the count reach 200, 000? (b) What will the bacteria count be in 10 hours?
(ii) Assume that the half-life of the radium in a piece of lead is 1500 years. How much radium will
remain in the lead after 2500 years?
(iii) If 1.7% of a substance decomposes in 50 years, what percentage of the substance will remain after
100 years? How many years will be required
for 10% to decompose?
(iv) The bacteria count in a culture doubles in 3 hours. At the end of 15 hours, the count is 1, 000, 000.
How many bacteria were in the count initially?
(v) The rate of natural increase of the population of a certain city is proportional to the population . If the
population increases from 40, 000 to 60, 000 in
40 years, when will the population be 80, 000?

5.14 Application of Integration to Problems in Economics

Integration can be applied in Economics in determining the total cost of production of a com-
modity when the marginal production cost is known. Recall that the total cost of producing x
number of items, is given by C ( x ), therefore the marginal cost function is given by the rate of
change of the total cost with respect to the items produced. That is, dC /
dx or C ( x ). Thus the total
R /
cost function, C ( x ) = C ( x )dx.

98
Example 5.14 : (1) The cost of producing an article consists of £1000 fixed cost and £2 per unit variable
cost. Find the total cost function and the cost of producing 2500 units of the article. (2) Given that
the marginal cost in dollars, of producing x units of a certain commodity is M( x ) = 3x2 4x + 5,
determine the total cost function and hence find (i) the cost of producing 20 units, (ii) the cost of producing
the 11th through the 15th units inclusive.(3) The marginal revenue function for a certain commodity is
M ( x ) = (500 30x ), where x is the number of units produced. Determine the total revenue function
r ( x ), and hence, find the total revenue in naira, realized, when 20units of the commodity are produced
and sold.

R
Solutions: (1) Let the total cost be C ( x ). ) C ( x ) = 2dx = 2x + K, where K = 1000. Hence,
C ( x ) = 2x + 1000. ) the cost of producing 2500 units is C (2500) = 2(2500) + 1000 = £6000.
R
(2) Total cost function, C ( x ) = (3x2 4x + 5)dx = x3 2x2 + 5x + C.
) C (0) = 0 ) C = 0. Hence, C ( x ) = x3 2x2 + 5x.
(i)Thus the cost of producing 20 units is C (20) = 203 2(20)2 + 5(20) = $7300.
R 15 15
(ii) the cost of producing 11th through the15th units is 10 (3x2 4x + 5)dx = x3 2x2 + 5x 10 =
153 2(15)2 + 5(15) 103 2(10)2 + 5(10) = $2150.
R
(3) r ( x ) = (500 30x ) dx = 500x 15x2 + C. But r (0) = 0 ) C = 0. ) r ( x ) = 500x 15x2 .
Hence, r (20) = 500(20) 15(20)2 = N4000.

The examples show the wide range of applications of calculus to real life problems. it is also the
basic tool required for the study of more sophisticated mathematics.
1

1 Provided by Departments of Mathematics,UNIJOS and PLASU - 2013

99

You might also like